CFP tax planning quizzes

Pataasin ang iyong marka sa homework at exams ngayon gamit ang Quizwiz!

MSC, Inc. is a closely held C corporation that manufactures boilers. The company has been in business for over 45 years. MSC has active income this year of $250,000 and no passive or portfolio income. The company also leases equipment that generates passive losses of $120,000 per year. How much of the passive loss can the company use this year? A) $120,000 B) $0 C) $100,000 D) $25,000 Explanation The company can deduct the entire $120,000 because it is a closely held C corporation that is not a personal service corporation. Passive losses may be used to offset active income, but not portfolio income.

ans a

Under Section 1221, which of the following is a noncapital asset? An office building that is rented to others A painting bought from the artist Accounts receivable of a manufacturer Flowers for sale by a florist A) I, III, and IV B) II only C) I and II D) III only Explanation Creative works are noncapital assets in the hands of the creator, not the buyer (ACID).

ans a

In 2020, Floyd, age 15, is a dependent on his parents' income tax return. When Floyd was born, his parents established an UGMA with corporate bonds and have contributed a little to it every year since. This year, the account generated $5,000 of interest income. There were no distributions from the account this year. Floyd's parents file jointly and have taxable income of $175,000 and are in the 24% MFJ tax bracket. What is Floyd's income tax liability for the current year? A) $782 B) $1,029 C) $110 D) $260

ans a Floyd's liability is $782. This is computed as follows: $5,000($1,100)limited standard deduction($1,100)taxed at child's rate of 10%$1,100×10%=$110.00$2,800taxed at parents' rate of 24%=$672.00$782.00

Which one of the following is NOT subject to the Medicare contribution tax? A) Qualified Roth distributions B) Income from a nonperiodic distribution from an annuity C) Qualified dividends D) Long-term capital gains

ans a Qualified Roth distributions are not subject to the Medicare contribution tax. Only taxable items, such as net capital gains, net rental income, annuity income and dividends, for example, are subject to the Medicare contribution tax.

All of the following are exclusion items for the purposes of calculating alternative minimum taxable income (AMTI) except A) the bargain element on exercised incentive stock options. B) the standard deduction amount. C) miscellaneous itemized deductions. D) taxes taken as itemized deductions.

ans a The bargain element on exercised incentive stock options is a preference (deferral) item for AMT purposes and is not an exclusion item for calculating AMTI.

Bill's 2019 income tax return, which was for a full year, showed an AGI of $140,000 and an income tax liability of $32,100. He estimates his 2020 income tax to be $38,000 and his total wage withholding to be $5,000.What minimum amount of estimated tax payments must Bill pay (in equal quarterly installments) for 2020? A) $41,800 B) $27,100 C) $34,200 D) $32,100 Explanation The safe harbor for avoiding the underpayment penalty is the lesser of 90% of the current year tax liability or 100% of the prior year tax liability (110% if the prior year's AGI was over $150,000). 100% of the prior year tax liability is $32,100. 90% of the current year tax liability of $38,000 is $34,200. The smaller of these numbers, reduced by the $5,000 withholding, equals $27,100.

ans b

Lisa and William are married taxpayers who file as married filing separately (MFS) for income tax purposes as it is advantageous for their tax positions; although the couple does live together. Lisa has rental property she inherited from her uncle that will generate a loss this year of $14,000. Lisa meets the active participation standard. Lisa's AGI is $65,000 and William's AGI is $80,000. How much is Lisa's allowed passive activity loss this year? A) $12,500 B) $0 C) $5,000 D) $14,000 Explanation The rental real estate loss allowance is not available to taxpayers who file as MFS and have lived together at any time during the tax year.

ans b

Which of the following taxpayers is subject to the passive activity loss rules? Individuals Estates Trusts A) I only B) I, II, and III C) II and III D) I and II Explanation All of these taxpayers are subject to the passive activity loss rules.

ans b

In the current year, George invested $100,000 for a 20% partnership interest in an activity in which he is a material participant. The partnership reported a loss of $400,000 in the current year and a loss of $200,000 in the following year. George's share of the partnership's loss was $80,000 in the current year and $40,000 in the following year. How much of the loss from the partnership, if any, can George deduct in the current and following year? A) $80,000 in the current year and $40,000 in the following year B) $0 in both the current and the following year C) $80,000 in the current year and $20,000 in the following year D) $80,000 in the current year and $0 in the following year Explanation George's losses are deductible in both years because he is a material participant in the activity. However, the at-risk rules limit his total losses to $100,000. He can carry the remaining $20,000 loss forward until he has increased his basis in the partnership through subsequent partnership allocations or investment in the partnership. LO 7.1.2

ans c

Ken is a successful attorney with a full-time practice from which he earns a salary of nearly $500,000 a year. Which of the following investments would be appropriate in reducing Ken's income tax liability? Investment in an oil and gas working interest A real estate limited partnership A rental real estate general partnership in which Ken will not participate Active participation rental real estate A) I, III, and IV only B) III only C) I only D) II and III only Explanation Only a direct interest in oil and gas (the working interest) would generate any tax savings. The other activities would be passive activities and would not meet any deduction exceptions. An investment in an oil and gas working interest is an exception to the passive activity rules. The real estate partnerships are both passive and could only be used if Ken had passive income against which he could offset the losses. The active participation rental real estate is also passive, and Ken does not meet the AGI requirements for the $25,000 exception because his AGI is well in excess of $150,000 (the upper limit of the active participation requirement).

ans c

To which of the following do the passive activity loss rules apply? Individuals C corporations that are not closely held Closely held C corporations Estates A)I only B)I and III C)I, III, and IV D)II and IV Explanation Of the listed choices, option II is the only option to which the passive activity loss rules do not apply. The passive activity loss rules specifically do not apply to C corporations that are not closely held.

ans c

Alex established a 2503(c) trust for his daughter, Julie, when she entered college four years ago. Alex decided to name his attorney as trustee and give Julie the right to revoke the trust at age 23, when she finished college. Julie did not revoke the trust and chose to allow the trust to continue until she is age 30. Which of the following correctly identifies the taxpayer, if any, who must pay tax on the trust income? A) Alex, because this is required by law B) The attorney as trustee C) The trust, because it is irrevocable and a separate taxable entity D) Julie, because she allowed the trust to continue past age 23

ans c Because Julie waited past age 23 when she had the right to revoke the trust, she is responsible for taxes on the trust given to her.

The kiddie tax rules apply to the unearned income of a child under age 19 (under age 24 if a dependent full-time student providing less than 50% of own support). to the earned income of a child under age 19 (under age 24 if a dependent full-time student providing less than 50% of own support). A) Both I and II B) Neither I nor II C) I only D) II only

ans c Statement I is correct. The kiddie tax applies only to any unearned income (such as dividends and interest) of a child under age 19 (under age 24 if a dependent full-time student providing less than 50% of own support). For a dependent on another's income tax return at any age, the standard deduction for earned income is the amount of earned income plus $350, but limited to an amount no greater than the standard deduction for a single taxpayer ($12,400 in 2020).

How is a shareholder's share of an S corporation's income and/or loss reported to the shareholder? A) On the K-1 of Form 1065 B) None of these C) On Schedule C of Form 1040 D) On the K-1 of Form 1120S Explanation An S corporation files IRS Form 1120S and also provides each shareholder with a Schedule K-1 form indicating that shareholder's share of the corporation's income or loss.

ans d

Three years ago, Mort purchased equipment for use in his business at a cost of $26,000. He claimed a Section 179 deduction in the year of acquisition of $10,000, and has since claimed cost recovery deductions totaling $7,604. The equipment was sold for $18,000. What is the amount of cost recovery deductions that must be recaptured? A) $7,604 B) $8,396 C) $0 D) $9,604 Explanation The cost basis of the property, $26,000, would be reduced by the Section 179 and cost recovery deductions taken, $17,604. This leaves an adjusted basis of $8,396. When the property is subsequently sold for $18,000, the difference between the sales price ($18,000) and the adjusted basis of $8,396 is the gain realized of $9,604. The recapture is the lesser of the gain realized of $9,604 or the cost recovery deductions taken of $17,604.

ans d

Your client, Joe, has active income of $300,000 per year and substantial unused passive losses from a nonpublicly traded limited partnership. He would like to find an investment that would allow him to utilize his passive losses. Which of the following are the most appropriate investments for Joe? An active participation rental real estate activity generating income A master limited partnership (MLP) generating income Certificates of deposit generating portfolio income A nonpublicly traded limited partnership generating income A) I, II, and IV B) I and II C) III and IV D) I and IV Explanation Income from active participation rental activities is considered passive income. The nonpublicly traded limited partnership losses may not offset income from a publicly traded partnership (the MLP) or portfolio income.

ans d

Mark operates a sole proprietorship from his apartment. His gross income for the current tax year is $61,000. Business expenses not associated with his home office total $63,000. Expenses associated with the home office total $4,200. How much of the home office expense, if any, may Mark deduct for the current year? A) $2,000 B) $275 C) $4,200 D) $0

ans d The home office expense deduction is limited to the earned income from the business. In other words, the home office expense deduction can generally neither create nor add to a loss. In this situation, the $61,000 of gross income is reduced by the $63,000 of business expenses not associated with the home office, to leave no earned income. Thus, of the $4,200 of home office expenses, none would be deductible in the current year. Note that the entire $4,200 of home office expenses would be subject to a carryforward. LO 5.4.1

Lindsey is age 2 and she received $6,000 in municipal bond interest income and $900 in other interest income in 2020. What is the total federal income tax due on her income in 2020?

A) $1,400 B) $1,495 C) $90 D) $0 ans: d Explanation Lindsey owes no federal income taxes in 2020. Municipal bond interest income is not taxable. The $900 in other interest income is less than Lindsey's $1,100 standard deduction amount.

Which of the following is NOT a step in the tax calculation process?

A) Calculate federal tax on federal taxable income. B) Subtract adjustments to income from total income to get adjusted gross income. C) Subtract exclusions from AGI. D) Deduct the greater of itemized deductions or the standard deduction. ans: C Explanation The following are involved in the income tax computation: subtracting adjustments to income from total income to get AGI, and deducting the greater of itemized deductions or the standard deduction from AGI to arrive at taxable income. Subtracting exclusions from AGI is not a step in the tax calculation process. Excluded amounts simply do not show up as income on the return.

Which one of the following steps occurs in the tax calculation process?

A) Total tax liability equals refund or tax owed B) Total tax liability minus itemized deductions plus additional taxes owed, equals total tax liability C) Total withholding is adjusted on Form I-9 D) Tax liability minus tax credits equals refund or tax owed ans: d Explanation Tax liability minus tax credits equals total tax liability or refund.

Which one of the following reflects the CORRECT sequence of steps in the tax calculation process?

Total (gross) income minus adjustments to income equals adjusted gross income (AGI). AGI minus standard or itemized deduction(s) equals federal taxable income.

Which of the following is NOT an allowable itemized deduction against alternative minimum taxable income? A) State and local income taxes B) Qualified housing interest C) Charitable contributions D) Medical expenses greater than 7.5% of adjusted gross income

ans a State and local income taxes, as well as property taxes, are not allowable itemized deductions. A deduction is allowed for medical expenses in excess of 7.5% of adjusted gross income (for 2020). A deduction is allowed for charitable contributions. A deduction is allowed for qualified housing interest.

Gregg has an AGI of $150,000. He donated to the local church stock valued at $100,000 that was purchased eight years ago. His basis in this stock was $30,000. What is Gregg's maximum allowable charitable contribution deduction for the current year? A) $45,000 B) $100,000 C) $75,000 D) $30,000

ans a The gift of long-term capital gain property to a 50% organization is based on the FMV of the property, with a 30% of AGI limitation. 30% of $150,000 is $45,000. The excess of $55,000 is subject to a five-year carryforward.

John is a contractor who has just purchased a tractor for use in his business. John paid $25,000 plus $1,250 in sales tax for the tractor. The local municipality also imposes an annual personal property tax of $500 per year. The tractor has an expected useful life of five years. What is John's basis in the tractor for depreciation purposes? A) $25,000 B) $26,250 C) $26,750 D) $25,500 Explanation The $25,000 cost is increased by the capitalized cost (the sales tax) of $1,250. The payment of a personal property tax has no impact on the basis. LO 6.1.2

ans b

Patty has a $10,000 passive loss carryforward from Beta limited partnership, which is publicly traded. She also has a $15,000 passive loss carryforward from Alpha limited partnership, which is nonpublicly traded. In the current year, she has $6,000 of income from Beta. She also has $11,000 of income from Gamma LP. Gamma is not publicly traded. What is the total amount of passive losses that Patty may deduct during the current year? A) $25,000 B) $17,000 C) $11,000 D) $6,000 Explanation Of the $10,000 passive loss carryforward from the Beta limited partnership, only $6,000 may be utilized in the current year due to the $6,000 of current year passive income. A total of $11,000 in losses from the Alpha limited partnership may be utilized against the $11,000 of income from the Gamma limited partnership in the current year because both are nonpublicly traded. Thus, the total of passive losses that are allowed for the current year is $17,000.

ans b

Which of these statements is NOT correct regarding cash value life insurance products? A) Proceeds payable before death are taxable to the extent they exceed the insured's cost basis. B) A MEC is not a life insurance contract. C) Income earned on funds invested in cash value insurance accumulates on a tax-deferred basis. D) Insurance products are a type of tax shelter. Explanation A MEC is a life insurance contract—it must meet one of the two Internal Revenue Code tests for life insurance, and the state law definition. The MEC is a life insurance contract that fails to meet the seven-pay test.

ans b

Teddy, age 12, has interest income of $1,275. He also has earned income from an after-school job that totals $12,500. Teddy is eligible to be treated as a dependent on his parents' return. What is the amount of Teddy's standard deduction for 2020? A) $12,850 B) $12,400 C) $1,275 D) $1,100

ans b The standard deduction for an individual eligible to be claimed, or treated, as a dependent is the greater of the limited standard deduction of $1,100 or the amount of earned income plus $350, not to exceed the full standard deduction amount of $12,400 (for 2020). $12,500 + $350 = $12,850, but the deduction is limited to the full standard deduction amount of $12,400.

Upon the disposition of a passive activity interest by gift, the suspended losses are A) added to the basis of the interest. B) completely lost. C) deductible in full. D) deductible to the extent the losses exceed any increase in the fair market value of the activity. Explanation Upon the disposition of a passive activity by gift, the suspended losses are added to the basis of the activity.

and a

Which of these is an economic objective of the federal tax system? A) Providing low-income housing B) Raising revenue C) Stabilizing prices D) Encouraging charitable contributions Explanation An economic objective of the federal tax system is price stability. The three primary objectives of the federal tax system are raising revenue, social objectives, and economic objectives (price stability and economic growth).

ans c

How can passive activity losses be deductible from other taxable income? A) Passive activity losses are deductible against portfolio gains. B) Passive losses can only be carried forward against future passive gains. C) Passive losses can only be offset by passive gains. D) Passive losses can offset passive gains, and a phased-out $25,000 deduction ($12,500 for MFS) for rental real estate applies. Explanation Generally, passive activity losses can only be used to offset passive activity income. An exception for the deduction of passive losses is for rental real estate. A phased-out deduction of $25,000 ($12,500 for MFS) of rental real estate losses is allowed against a taxpayer's other nonpassive income. If AGI is greater than $100,000 ($50,000 for MFS), however, there is a reduction of 50 cents for each dollar over $100,000 ($50,000 for MFS), which then terminates at $150,000 ($75,000 for MFS) of AGI.

ans d

Dolores bought 200 shares in a mutual fund for $15 per share. Shortly after this purchase, the mutual fund went ex-distribution and declared a distribution of $.50 per share. Dolores elected to have dividend distributions from the fund reinvested to purchase additional shares at $15 per share. How much taxable gain will Dolores incur if she later sells all her shares for $16 per share? A) $207 B) $200 C) $107 D) $100 Explanation Gain is determined by the difference between the sales price of the shares held and the basis of the shares held. Basis is (200 × $15) + ($0.50 × 200), or $3,100. This represents the original cost of the 200 shares plus the reinvested dividends of $100. The number of shares owned must first be determined. The reinvested dividend of $100 is divided by the purchase price and added to the originally purchased shares. $100 ÷ $15 = 6.67 shares + 200 = 206.67 shares (these shares are then multiplied by the sales price of $16 to arrive at a total sales price of $3,307). $3,307 - $3,100 = $207 total gain on the sale.

ans a

During June 2020, Judy, a sole proprietor, purchased new equipment for her business at a cost of $640,000. She has net income (without regard to the Section 179 deduction) from the sole proprietorship of $206,000. She also has wages from a part-time clerking position of $14,000. What is the maximum Section 179 deduction that Judy may claim with respect to the equipment? A) $220,000 B) $640,000 C) $206,000 D) $510,000 Explanation The Section 179 deduction is subject to a taxable (earned) income limitation. However, for this purpose, wages received (even from a completely unrelated source) are considered to be from the active conduct of a trade or business. With only $220,000 of earned income, only $220,000 may be deducted under Section 179. The maximum Section 179 deduction, as a result of TCJA, is $1.04 million (2020), and the property placed in service limitation was increased to $2.59 million.

ans a

Which of these types of investors derives the greatest tax benefit from investing in preferred stocks? A) Mutual funds B) Corporate C) Government D) Nonprofit institutional Explanation Because 50% of the preferred dividends (and other dividends from stock) received by a corporation are exempt from federal income taxes, a corporation gains a tax advantage. The government and nonprofit organizations pay no income taxes. Mutual funds are also exempt from taxation. (TCJA reduced the dividend-received deduction to 50%.)

ans b

Which one of the following types of investors derives the greatest tax benefit from investing in preferred stocks? A) Nonprofit institutional investors B) Mutual funds investors C) Government investors D) Corporate investors

ans d Because 50% of the preferred dividends received by a corporation are exempt from federal income taxes, a corporation gains a tax advantage. The government and nonprofit organizations pay no income taxes. Mutual funds are also exempt from taxation. (The Tax Cuts and Jobs Act, or TCJA, reduced the dividend-received deduction to 50%.)

Sam has the following items of income: Self-employment earnings$45,000Interest income$ 4,000Gain on the sale of a capital asset$12,000 What is the amount of self-employment tax Sam owes? Round your answer to the nearest dollar. A) $8,619 B) $6,885 C) $6,923 D) $6,358 Explanation Self-employment income$45,000.00Less 7.65% of $45,000(3,442.50)$41,557.50Times tax rate15.3%Self-employment tax$6,358.30

ans d

Alisha, a CFP® certificant and fee-only financial planner, has assisted Roger, a self-employed physician, in income tax return preparation and investment planning during the year. On which of the following schedules may Alisha's fee be deductible by Roger on his federal income tax return? Schedule A—itemized deductions Schedule C—profit or loss from business Schedule D—capital gains and losses A) I, II, and III B) I only C) I and II D) II only

ans d The investment planning fees are not deductible (the Tax Cuts and Jobs Act, or TCJA, repealed the Tier II miscellaneous itemized deductions). This expense may not be added to the basis of securities. The tax preparation fee is partly nondeductible (the portion attributable to the i

Which one of the following reflects the CORRECT sequence of steps in the tax calculation process?

A) Calculate federal tax on total income. B) AGI minus adjustments to income equals federal taxable income. C) Total income minus adjustments to income equals AGI. D) Total income minus standard or itemized deduction(s) equals AGI. ans: c Explanation Total (gross) income minus adjustments to income equals adjusted gross income (AGI). AGI minus standard or itemized deduction(s) equals federal taxable income

Marvin has all of the following items. All of them are AMT preference items except A) percentage depletion in excess of adjusted basis on a mining property. B) tax-exempt income from a State of Iowa municipal revenue bond. C) exclusion of gain on the sale of certain qualified small business corporation stock. D) tax-exempt interest on certain private-activity bonds.

ans b Tax-exempt income from a municipal revenue bond is not a preference for AMT. All of the other items are preferences for AMT purposes.

In an attempt to curb overzealous use of tax-sheltered investments, the government has developed rules including at-risk requirements. passive-activity loss limitations. special certification requirements for real estate investors. A) I only B) I, II, and III C) I and II D) II and III Explanation Statement III is incorrect. No special certification is required for real estate investors.

ans c

Jacob has an apartment building in Atlanta that he would like to exchange. Which of the following assets could Jacob receive in a like-kind exchange? Farmland Interest in a low-income housing limited partnership Parking lot An apartment building in Tahiti A) II, III, and IV B) II and III C) I and III D) III and IV Explanation In a like-kind exchange, only real estate may be exchanged for real estate. The like-kind exchange rules specifically prohibit the exchange of U.S. realty for foreign realty.

ans c

Larry and Paula are a married couple who file their federal income tax returns separately. They are both over 65 and still provide full support for a son who has been blind since birth. They live together and do not itemize. They alternate listing their son as a dependent, and it is Paula's turn this year. Paula will be required to file a federal income tax return if her gross income is at least which of the following amounts in 2020?

B) 12400 The normal filing threshold for the MFS filing status is $12,400 in 2020. For married taxpayers over age 65, the threshold is raised by $1,300 per spouse. The additional blind deduction applies only to the taxpayers themselves, not their dependents. Tangentially, if the other MFS spouse itemizes, the filing threshold is reduced to $5. (IRS pub 501, 2020) Because Larry and Paula still live together, neither can file as head-of-household with a dependent.

Don and Paul are married. They adopted an infant daughter in December of last year. They have consulted you, a CFP® professional, for advice on how to proceed when filing their federal income tax return this year. What should you recommend as their filing status this year for their federal return?

B)Married filing jointly Explanation Having a dependent does not change the filing status for a married couple. LO 1.1.1

In the current year, Keith had passive losses of $19,000 from a real estate limited partnership purchased in 1985. He also had passive income of $7,000 from an oil and gas limited partnership. Both limited partnerships are not publicly traded. What is the total amount of passive losses that may be used to offset active, passive, and portfolio income in the current year? A) $9,400 B) $7,000 C) $8,200 D) $1,900 Explanation The passive loss ($19,000) is deductible, but only up to the amount of passive income ($7,000) in the same year; thus, the answer is $7,000.

ans b

Which of the following apply to the passive activity loss rules? Deductible passive losses are limited to the passive gains in other passive activities. Any unused passive losses may be carried forward against future passive gains. When the passive activity property is disposed of, any unused passive losses can be deducted against passive gains, portfolio, or active income. Passive loss rules apply only to real estate transactions. A) II, III, and IV B) I, II, and III C) I and III D) II and IV Explanation Passive loss rules apply to all passive activities, not just real estate transactions. Passive losses may be deducted against passive gains. If the investor has excess passive losses, the losses are carried forward and may be used in future years to offset future passive gains.

ans b

Which of the following forms of business may be classified as direct participation programs? General partnership Limited partnership S corporation Closely held C corporation A) IV only B) I, II, and III C) I and II D) III and IV Explanation The tax advantages provided by direct participation programs are founded upon the principle that most types of business organizations function as tax conduits; therefore, a closely held C corporation cannot qualify as there is no flow through of gains and losses.

ans b

Which of the following statements regarding married couples who file joint tax returns is NOT correct? A) Spouses may file a joint return even if one spouse has no income. B) When spouses file jointly, each spouse is liable for only one-half of the tax due. C) Spouses who file a joint return have joint and several liability for the payment of any tax due. D) The law provides for innocent spouse relief, which may excuse one spouse for the failure of the other spouse's tax obligation. Explanation Spouses who file a joint return have joint and several liability for the payment of any tax due. This means each spouse is responsible for the entire tax liability and not just one-half.

ans b

Which one of the following is a statute of limitations that restricts the IRS in auditing a return? A) Ten years for failure to file or if a fraudulent return is filed B) Six years if 25% of gross income is unreported C) Seven years from the filing date of the return or due date, if later D) Twelve years if 50% of gross income is unreported Explanation After the statute of limitations has passed, except in cases of fraud, the IRS cannot audit a return. The statutes to be aware of are as follows: three years from the filing date of the return or due date if later, six years if 25% of gross income is unreported, and no statute of limitations for failure to file or if a fraudulent return is filed.

ans b

With respect to the at-risk rules, qualified nonrecourse financing is a debt A) from a loan provided by a related person at "below market" terms. B) secured by specific real property. C) for which at least one individual is personally liable. D) that is convertible into an equity interest. Explanation A partner's share of nonrecourse financing established basis in the partnership, but is not treated as an amount at risk. Remember that nonrecourse financing is debt that is secured by the property, but for which no individual has personal liability.

ans b

Full imputed interest on a below-market loan (with the IRS providing accepted loan rates) will be paid with A) a gift loan of amounts more than $5,000. B) a gift loan of amounts more than $100,000. C) any gift loan given. D) a gift loan of amounts more than $10,000.

ans b In a gift loan, the amount of the imputed interest constitutes a gift from the lender to the borrower. For gift loans greater than $10,000 and less than or equal to $100,000, no interest is imputed if the borrower's investment income for the year does not exceed $1,000. For a gift loan of more than $100,000, the prevailing federal rate of interest will be imputed.

Bruce and Melissa Parish, married taxpayers filing jointly, have the following items related to their investments during the current tax year: Investment interest expense $5,000 Interest income $2,500 Short-term capital gains $1,000 Investment adviser's fees $1,250 Commissions paid on stock purchase $200 Adjusted gross income $60,000 What is the Parishes' allowable investment interest expense deduction for the current year? A) $3,250 B) $3,500 C) $3,450 D) $5,000

ans b Investment interest expense is limited to the taxpayer's net investment income of $3,500.

Which of the following is a tax preference item for the purpose of calculating the alternative minimum tax? Tax-exempt interest from a private-activity bond issued in 2008 Cash contributions to charitable organizations Cash flows from limited partnerships Personal-service income in excess of tax losses A) II only B) I only C) I, III, and IV D) I and II

ans b Items II, III, and IV are not tax preference items. However, other AMT tax preference items include the part of the deduction for certain depletion that is more than the adjusted basis of the property and the excluded gain on the sale of certain small business stock (Section 1202).

Max and his mother, Lucy, live together in his home. Lucy is bedridden, and Max must pay a caregiver to provide meals and other aid during the day so he can leave the house and work. He pays $5,000 annually for this service. His mother has no income, and he is her full support. What tax relief may be available to Max for the expenses of caring for his elderly mother? A greater standard deduction amount Child and dependent care credit A) I only B) Both I and II C) II only D) Neither I nor II

ans b Max is entitled to both the standard deduction amount for the head of household filing status and the child and dependent care tax credit, subject to the limits based on Max's AGI.

Jack and Emily are legally separated on December 31 this year. Jack earned $40,000 this year, and Emily earned $80,000. They live in a common law state and have no dependents. They have come to a tax preparer to determine how they must file their income taxes this year. What does the planner tell them? A) They may file as MFJ as they were not legally separated until the end of the year. B) They must file as single, each one reporting their own income. C) They must file as single, each reporting $60,000 (half of the total of $120,000) in income. D) They must file as MFS, each reporting their own income.

ans b Single (S) filing status is used by an unmarried, legally separated, or divorced individual who does not qualify for any other filing status.

Which of the following statements regarding alternative minimum tax (AMT) are true? AMT reduces the tax benefits from certain types of deductions and tax preferences allowable for regular tax purposes. Depreciation allowable for AMT can never be the same as that allowable for regular tax purposes. It may be advantageous to accelerate ordinary income into years when AMT will be paid. It is often advantageous to not accelerate the payment of state income and real estate taxes when AMT will be paid in the current year. A) I, II, and IV B) I, III, and IV C) I and II D) III and IV

ans b The common strategies related to income tax planning are often reversed when dealing with the AMT. Thus, rather than accelerating certain itemized deductions, we often would defer payment (if possible) of those items that are not deductible for AMT purposes. We often will accelerate income that would have been taxed at the highest marginal rates in future years into the current year to be taxed at the AMT rates of 26% or 28%.

Which of the following is necessary to include in a premarital agreement? A) Intentions to facilitate a divorce B) Financial resources and net worth of both parties C) Financial resources and net worth of the more wealthy party D) Financial resources and net worth of the less wealthy party

ans b To be valid, a premarital agreement must be in writing and contain a complete disclosure of each party's financial situation. It may not be used to regulate an award of alimony.

Alex owns a vacation home that he rents, on average, 98 days each year. Alex stays in the home for 6 weeks during the fall every year. Which category does this vacation home fall under? A) Because Alex stays in the home for 6 weeks in each year, the home is classified as his primary residence B) Rental use C) Mixed use D) Primarily personal use Explanation A vacation home is classified as mixed use if the vacation home is rented for at least 15 days a year and it is also used for personal use for more than the greater of 14 days per year or 10% of the rental days.

ans c

The gross profit percentage in an installment sale is calculated by A) dividing the sale price by the number of payments required to be made under the contract. B) multiplying the payments received by the contract price. C) dividing the gross profit by the contract price. D) multiplying the marginal income tax bracket by the payments received. Explanation The gross profit percentage in an installment sale is calculated by dividing the gross profit (the sale price minus the adjusted basis) by the contract price (typically the sale price). It is not determined by multiplying the marginal income tax bracket by the payments received.

ans c

Matthew Brady, age 47, purchased a deferred annuity in January 1982 for $50,000. In the current year, when the surrender value was $125,000, Matthew took a nonperiodic distribution of $75,000. Which one of the following statements correctly describes the income tax consequences of the distribution? A) $75,000 is tax free. B) $75,000 is taxable income. C) $50,000 is tax free, $25,000 is taxable. D) $50,000 is taxable, $25,000 is tax free.

ans c The pre-August 14, 1982, annuity retains first-in, first-out (FIFO) treatment. Thus, the basis of $50,000 is treated as being withdrawn first and is tax free. The remaining $25,000 is taxable. If this were a post-August 13, 1982, contract, it would be treated on a last-in, first-out (LIFO) basis.

Assume a taxpayer is faced with a tax deficiency of $10,000, along with interest on the deficiency of $4,200; the entire deficiency is the result of negligence from the taxpayer's 2018 return. What is the amount of the penalty? A) $10,650 B) $7,500 C) $2,840 D) $2,000 Explanation The negligence penalty is 20% of the amount of the deficiency attributable to negligence. Twenty percent of $10,000 is $2,000. The interest does not enter into the computation.

ans d

David and Kristen are married taxpayers filing jointly. They lived in their principal residence for six months, and sold the residence because Kristen was transferred to a job out of state. They have a realized gain of $145,000 on the residence. They have not used the Section 121 exclusion in the past. What is the maximum Section 121 exclusion, if any, that they may claim? A) $500,000 B) $0 C) $36,250 D) $125,000 Explanation They are entitled to a partial exclusion as they failed to meet the two-year test due to health, job, or unforeseen circumstances. The partial exclusion is six months of ownership and use divided by the required 24-month ownership and use period, multiplied by the full exclusion amount of $500,000. Thus, 25% of $500,000 = $125,000.

ans d

Lowell and Thelma Jordan are married and will file a joint return for the current tax year. They have provided you with the following information: Lowell's salary$140,000Thelma's salary$25,000Unemployment compensation$10,000Net capital loss$8,000 Based on the information given, what is Lowell and Thelma's adjusted gross income for the current tax year? A) $157,000 B) $162,000 C) $172,000 D) $175,000

ans: c The total salaries of $165,000 plus the unemployment compensation of $10,000 equals $175,000. Net capital losses of $3,000 per year are deductible, leaving $172,000.

Which of the following forms does a C corporation file? A) Form 1120 B) Form 1120-S C) Form 1120-C D) Form 1065

ans a

Michelle Will has interest income of $23,000 in the current tax year. She paid brokers' commissions of $2,000 on stock purchases and had $40,000 of investment interest expense. What amount, if any, of investment interest expense may be deducted as an itemized deduction? A) $23,000 B) $33,000 C) $21,000 D) $0

ans a

Which of the following activities is treated as a rental activity under the passive activity rules? Property rental where average customer use is 6 days Property rental where average customer use is more than 30 days and no significant services are provided Property rental where extraordinary services are provided on behalf of the owners Property rental where the property is customarily made available during defined business hours for the nonexclusive use of customers A) II only B) II, III, and IV C) II and III D) I only Explanation Only Statement II is treated as a rental activity according to IRS regulations. If a rental is 30 days or less and significant personal services are provided, the activity is a service rather than a rental activity.

ans a

To be considered a responsible person by the IRS, which one of the following is among the important factors? A) Impressive title B) In charge of hiring and firing employees C) Shareholder of a similar company D) No authority to sign checks Explanation The determination of whether someone is a responsible person is a test of facts and circumstances. Some of the questions the IRS usually asks in determining responsibility are as follows: Was the individual an officer or director? Was the individual a shareholder? Was the individual a member of a board of directors? Did the individual have the authority to sign checks? Was the individual responsible for hiring and firing employees? Did the individual have actual authority or merely an impressive title?

ans b

Which of the following is a tax preference item for the purpose of calculating individual AMT? Tax-exempt interest from a private-activity bond issued in 2006 Cash contributions to charitable organizations Cash flows from limited partnerships Capital losses A) II only B) I only C) II, III, and IV D) III and IV

ans b Tax-exempt interest on a private-activity bond issued in 2006 is a preference item.

Amy, age 12, is claimed as a dependent on her parents' income tax return. During 2020, she earned $2,200 from a summer job. She also earned $2,600 in interest and dividends from investments that were given to her by her grandfather five years ago. How much of Amy's income, if any, will be taxed to her in 2020 using her grandfather's marginal tax rate of 32%? A) $400 B) $0 C) $2,700 D) $2,300

ans b When applying the kiddie tax, the parents' marginal tax rate is always used (regardless of the source of unearned income). Therefore, none of the income is taxed to Amy using the grandfather's tax rate. $400 of income ($2,600 − $2,200) is taxed to Amy at her parents' marginal tax rate.

Beth's husband died in Year 1. Assume that Beth does not remarry and continues to maintain a home for herself and her dependent child during Year 2, Year 3, and Year 4, providing full support for her child throughout those years. For Year 4, Beth's filing status will be

head of household Beth's Year 4 filing status is head of household. Qualifying widow filing status is only available for 2 years following the death of a spouse (Year 2 and Year 3).

Which of the following statements correctly defines inside buildup as it refers to life insurance?

Accumulations of cash value within a life insurance policy grow on a tax-deferred basis during the insured's lifetime.

Which one of the following is the best description of an exclusion?

An exclusion is an item, such as a qualified Roth distribution or interest from a municipal bond, that is not subject to regular income tax and is not part of income on the Form 1040.

Which of the following forms of gifting to a charity will qualify for the income tax charitable deduction? Gift of stock to a 50% charity Gift of cash to a supporting foundation Gift of land to a 30% charity

All of these forms of gifting to a charity will qualify for the income tax charitable deduction.

Which of the following are includible in an individual's gross income for income tax purposes? Gambling winnings Inheritances Interest collected by the taxpayer on federal obligations Scholarships and fellowships in degree programs

Gambling winnings and interest on federal obligations are includible in an individual's gross income for income tax purposes. The other items are not subject to income taxation.

The effective tax rate is obtained by dividing the amount of tax paid by

The effective tax rate is found by dividing total tax by taxable income.

Mary created an irrevocable trust for her two minor sons. She named her bank as trustee. The trust property earned $75,000 in the first year and had taxable income of $68,000 after deducting expenses. This income was left to accumulate for future distributions to be made to each son equally when the youngest son attains age 18. To which of the following will the income of the trust be taxable?

The trust will pay the taxes since the trust is irrevocable and no distributions are allowable until the youngest son attains age 18.

Which of the following is a public pronouncement that contains official guidance about regulations or interpretations of the Internal Revenue Code (IRC)? A) Private Letter Ruling B) Notice C) Revenue Ruling D) Revenue Procedure Explanation A notice is a public pronouncement that contains official guidance about regulations or interpretations of the Code. The guidance is often substantial but again, ultimately it only points to higher regulations. Notices in and of themselves do not carry the weight of law.

ans b

Which of the following taxpayers may use the married filing jointly filing status? A married couple, even though one spouse did not have any income during the tax year A married couple that is legally separated on the last day of the tax year if they share custody of a dependent child A married couple that is legally separated on the last day of the tax year A married couple that is not legally separated on the last day of the tax year A) I, II, and III B) II and IV C) I and IV D) I, II, and IV

ans c A married couple may file a joint return even though one spouse has no income or deductions if they are not legally separated or divorced on the last day of the tax year.

Imputed loan interest is usually taxable and occurs when A) the loan is secured by specific real property. B) a company loan is provided to an employee. C) a loan is provided by a related person at below-market terms. D) bonds are gifted by an employer.

ans c Imputed loan interest is usually taxable and occurs when a loan is provided by a related person at below-market terms.

Imputed interest on a below-market loan (with the IRS providing accepted loan rates) will be paid, unless the gift loan is A) from a corporation to a shareholder. B) between friends. C) less than $10,000 and the gift loan recipient has less than $1,000 in interest income. D) less than $5,000 and the loan recipient has no interest income.

ans c In a gift loan, the amount of the imputed interest constitutes a gift from the lender to the borrower. For gift loans greater than $10,000 and less than or equal to $100,000, no interest is imputed if the borrower's investment income for the year does not exceed $1,000. For a gift loan of more than $100,000, the prevailing federal rate of interest will be imputed.

Caroline, age 48, has a filing status of single and she earned a salary of $55,000 in 2020. Her employer also paid $6,000 for health insurance premiums for Caroline. Caroline had a $3,000 capital loss during the year. What is Caroline's adjusted gross income (AGI) in 2020? A) $52,000 B) $49,000 C) $55,000 D) $61,000

ans: a Caroline's adjusted gross income (AGI) is $52,000 ($55,000 - $3,000). Caroline's $3,000 capital loss reduces her gross income by $3,000 to $52,000. The health insurance premiums were never included in her income, thus cannot be deducted.

Jane, age 35, whose filing status is single, earned a salary of $55,000 in 2020. She also made a $2,000 contribution to her Roth IRA for 2020. Jane had a capital loss of $3,000 during the year. Her uncle, Charles, gave her $100,000 in municipal bonds for which she earned interest of $3,500. In her employment as a sales representative for her company, Jane incurred $650 of unreimbursed business expenses. What is Jane's adjusted gross income (AGI)?

A) $56,800 B) $53,800 C) $52,000 D) $53,300 ans: c Explanation Jane's AGI is $52,000 ($55,000 ‒ $3,000). Jane's $3,000 capital loss is a deduction for calculating AGI. Roth IRA contributions are never deductible from gross income. Municipal bond interest is not included in income. The unreimbursed business expenses are not deductible.

Which of the following are adjustments to gross income (above-the-line deductions)? Medical expenses Capital losses Deductible IRA contributions

II and III Medical expenses are an itemized (below-the-line) deduction.

Which of the following are preference items or adjustments for purposes of the individual alternative minimum tax? Interest on qualified private-activity municipal bonds issued in 2008 Excess of percentage depletion over the property's adjusted basis Investment interest expense in excess of net investment income Qualified housing interest A) I and II B) I only C) I, II, III, and IV D) II and III

ans a By definition, investment interest expense in excess of net investment income and qualified housing interest are not preference items or adjustments for purposes of the alternative minimum tax. Remember that interest on qualified private-activity municipal bonds issued in 2009 and 2010 is not a preference item.

Which of the following is NOT an advantage of the cash basis method of accounting? A) Taxpayers have more control over each year's income and expenses. B) Constructive receipt serves to defer income. C) Income is reported when it is received. D) Taxpayers can keep simple records.

ans a Constructive receipt, which serves to accelerate income, is not considered an advantage of the cash method of accounting.

For two years, Lisa Carson was able to pay the premiums on her whole life policy without borrowing. For the past two years, she has borrowed from the cash value of her whole life policy to pay the premiums. Last year, she paid $95 of interest on the funds she borrowed. What are the tax implications in this situation?

A) The interest expense is not tax deductible because it does not exceed $100. B) The interest expense is not tax deductible. C) The interest expense is tax deductible because it does not exceed $100. D) The interest is deductible because Lisa is in the business of continuing her insurance and the interest is deductible business interest expense. ans: b Explanation The interest expense is not tax deductible because interest on a loan incurred to purchase personal life insurance protection is considered personal interest, which is not deductible. Personal loan interest is not tax deductible, regardless of whether the lender is a bank or a life insurance company.

All of the following statements regarding income tax filing status are CORRECT except

A) unmarried people who maintain a household for a qualifying child or relative may be eligible for head of household status. B) spouses may not file a joint return if one spouse has no income or deductions. C) it is usually advantageous for married couples to file a joint return. D) if a spouse dies during the tax year, the surviving spouse may use married filing jointly status for that year. ans: b Explanation Spouses may file a joint return even if one spouse has no income or deductions.

Kelly established an irrevocable trust for the benefit of her two young grandsons. She named her attorney as the trustee. The trust documents stipulate that the income from the trust be accumulated within the trust for distribution to each grandchild when the youngest reaches age 21. Which of the following parties will be currently taxed on the income from the trust?

Currently, until the youngest reaches age 21, income from the irrevocable trust will be taxed to the trust.

Mary's husband died in March of the current year. What filing status should Mary use in the current year?

Married filing jointly is allowed for a surviving spouse in the year of death. This filing status has the most favorable tax brackets and features the largest standard deduction. The surviving spouse should coordinate this election with the executor of the estate of the deceased spouse.

The marginal tax rate is obtained by

The marginal tax rate is found by finding the tax bracket that contains the taxable income amount; it is the amount at which all subsequent taxable amounts will be taxed (until entering the next tax bracket). The effective tax rate is calculated by dividing the calculated tax by total (gross) income, not taxable income.

Carl, June, and Diane are partners. There is one employee, Mark, who is part-time, working 250 hours per year, primarily in the busy season. Carl works 1,900 hours annually while June works 600 hours and Diane devotes 550 hours in the busy season. Who qualifies as a material participant in the partnership? A) Carl, June, and Diane B) Carl, Mark, June, and Diane C) Carl and June D) Carl only Explanation Carl, June, and Diane each participate more than 500 hours in the partnership and are considered material participants.

ans a

The partner's tax basis in his interest in a partnership A) is increased by his share of income reported by the partnership. B) is decreased if additional capital is contributed. C) remains unchanged until the interest is sold or otherwise disposed. D) remains unchanged unless additional capital is contributed or distributions are made. Explanation A partner's tax basis in a partnership interest is affected by items of income or loss, which are passed through to the partner on a proportionate basis. Items of income, as well as capital contributions, increase basis; deductions, as well as distributions, decrease basis.

ans a

Which of the following statements best describes the difference between portfolio and passive income or losses? A) Portfolio income/loss is derived from investment practices, while passive income/loss is derived from ownership of rental property and activities in which there is no material participation. B) Portfolio income/loss is derived from investment activities, while passive income/loss is derived from the active management of businesses. C) Portfolio income/loss is derived from a combination of investment assets and rental property, while passive income/loss is derived only from rental income. D) Portfolio income/loss is derived from ownership of rental property, while passive income/loss is purely derived from investment activities. Explanation Portfolio income or loss is characterized as interest, dividends, or non-passive investment property. Passive income or loss is derived from passive or rental activities. A passive activity is one in which the investor does not materially participate. The deductibility of passive losses is generally limited to the amount of passive income.

ans a

Alicia is age 16 and she received $6,000 in municipal bond interest income and $900 in other interest income in 2020. Her parents' marginal tax rate is 28%. What is the total federal income tax due on her income in 2020? A) $0 B) $1,472 C) $90 D) $735

ans a Alicia owes no federal income taxes in 2020. Municipal bond interest income is not taxable. The $900 in other interest income is less than Alicia's $1,100 standard deduction amount (for 2020).

Bob passed away during the current year. He had suspended losses from a passive activity of $15,000. Bob's basis in the partnership was $1,000, and the fair market value at the time of his death was $9,000. Bob originally purchased the partnership interest for $25,000. What amount of passive losses, if any, is deductible on Bob's final return? A) $1,000 B) $7,000 C) $0 D) $15,000 Explanation The suspended losses are "freed up" only to the extent that the losses exceed the step-up in basis. In this situation, the basis was stepped up by $8,000; subtracted from the $15,000 of losses leaves $7,000 of losses that are freed up

ans b

Frank Swanson anticipates adjusted gross income of $80,000 during the current tax year. He is considering making a gift of real estate to the public university he attended. Frank's adjusted basis in this real estate is $50,000. The real estate has a current fair market value of $70,000. Frank has owned the real estate for 19 months. If Frank donates the real estate, what is the maximum allowable charitable deduction Frank can receive for the current tax year? A) $24,000 B) $40,000 C) $50,000 D) $70,000

ans b If Frank makes a 50% election, he must utilize the basis of the property but may deduct up to 50% of AGI. This yields a $40,000 current-year deduction with a $10,000 carryforward. If no 50% election were made, the deduction would be based on the fair market value of the property but would be limited to 30% of AGI, which is $24,000, with a $46,000 carryforward.

Which of the following statements regarding the alternative minimum tax (AMT) or AMT planning are CORRECT? The AMT reduces the tax benefits from certain types of deductions and tax preferences allowable for regular income tax purposes. The starting point for determining alternative minimum taxable income (AMTI) is AGI as reported for regular income tax purposes. It is generally advantageous to defer the payment of real estate taxes to a future year when AMT will be paid in the current year. It is generally advantageous to accelerate ordinary income into years when AMT will be paid. A) I, II, and IV B) I, III, and IV C) I and III D) III and IV

ans b The IRS notes that the starting point for determining AMTI is taxable income as reported for regular income tax purposes on a taxpayer's IRS Form 1040. Because real estate taxes are not deductible for AMT purposes, it is generally advantageous to defer the payment of such taxes to a year when AMT will probably not be payable. Also, if AMT will be payable in the current year, it is generally advantageous to increase the amount of regular taxable income (e.g., by accelerating ordinary income into the current year) because the total tax payable will likely not be affected by doing so.

Nathan has a salary of $100,000, dividends of $4,000, and limited partnership income of $10,000. The limited partnership is publicly traded. During January of the current year, Nathan purchased an interest in a nonpublicly traded limited partnership that will generate a $12,000 passive loss during the current tax year. How much of this passive loss, if any, is deductible by Nathan during the current tax year? A) $10,000 B) $4,000 C) $0 D) $12,000 Explanation The general rule is that passive losses are deductible only against passive income. However, passive income from a publicly traded partnership cannot be offset by passive losses arising from any other source. Thus, the passive losses from the new partnership will not be deductible.

ans c

David owns property with a current fair market value (FMV) of $60,000 and an adjusted basis of $80,000. He has an AGI of $200,000. He plans to donate the property to charity. Which of the following statements regarding the income tax implications of David's plan is CORRECT? If David donates the property to charity, he cannot recognize a capital loss of $20,000. If David donates the property to charity, he can deduct $80,000 as a charitable deduction. If David sells the property for $60,000 to a third party in an arms-length transaction, he can recognize a capital loss of $20,000 and donate the $60,000 cash received to the charity. A) I, II, and III B) II and III C) I and III D) I and II

ans c Statements I and III are correct. Statement II is incorrect. If David donates the property to charity, he can deduct only the current FMV of $60,000 as a charitable deduction and would not be able to recognize the capital loss of $20,000.

Which of the following is subject to the self-employment tax? Distributive share of limited partnership operating income Flow-through of S corporation income Distributive share of general partnership operating income Interest or dividends from investments A) II and IV B) I and III C) III only D) I, II, III, and IV

ans c The general partnership operating income is self-employment income. By definition, the other items of income are not subject to the self-employment tax.

A taxpayer invested in a real estate limited partnership several years ago. There is a special allocation in effect in the partnership. He is concerned about the deductibility of the losses that are flowing from the partnership. Which of the following rules or doctrines may limit the availability of income tax benefits from his limited partnership investment? Direct participation program Special allocation rules At-risk rule Passive activity loss rule A) II and III B) I and IV C) I and III D) II, III, and IV Explanation Special allocation rules, such as the substantial economic effect doctrine, limit the ability to utilize special allocations in a partnership. The at-risk rule limits the ability to utilize leverage by attacking the use of nonrecourse financing. The passive activity loss rule limits the ability to deduct losses from activities in which the taxpayer does not materially participate. The direct participation program simply refers to a tax conduit.

ans d

All of these statements regarding recapture (the taxation of certain gain from the sale of the asset as ordinary income instead of capital gain) are correct except A) for depreciable tangible personal property, all depreciation claimed is generally subject to recapture as ordinary income upon the sale of the property, regardless of the depreciation method used. B) under MACRS, the amount of depreciation recaptured as ordinary income for residential property is only the amount of depreciation claimed in excess of the amount allowable under the straight- line method. C) for real property placed in service under MACRS, there is no recapture as ordinary income because it is depreciated under the straight-line method. D) for real property, all depreciation claimed is always subject to recapture at the ordinary tax rate income upon the sale of the property. Explanation There is no recapture of depreciation on the sale of Section 1231 real estate. The gain created by the straight-line depreciation is referred to as unrecaptured Section 1250 income, which is a type of long-term capital gain that is taxed at a maximum rate of 25%. If the taxpayer's marginal tax rate is less than 25%, the unrecaptured Section 1250 gain will be taxed at that lower rate.

ans d

During the current tax year, Riley, a single taxpayer, has a $10,000 short-term capital loss and a $10,000 long-term capital gain, both from the sales of securities. Riley also has a $15,000 long-term capital gain from the sale of collectibles. Riley is in a 37% marginal income tax bracket. What is the income tax result from these transactions? A) $5,000 of collectibles gain taxed at 25%, and $10,000 of long-term capital gain taxed at 15% B) $15,000 collectibles gain taxed at 25% C) $15,000 collectibles gain taxed at 28% D) $5,000 of collectibles gain taxed at 28%, and $10,000 of long-term capital gain taxed at 20% Explanation The $10,000 short-term capital loss is first used against the collectibles gain—the gain that would be taxed at the highest rate (28%). This leaves $5,000 of collectibles gain, taxed at 28%, and $10,000 of long-term capital gain from the sale of securities, taxed at 20%. The long-term gain from the securities is taxed at 20% because Riley is in the 37% marginal income tax bracket. That gives him taxable income in excess of the breakpoint for the 20% rate—taxable income in excess of $441,450 (for 2020).

ans d

On June 1, 2020, Hugh and Judy, both age 45, moved into a new house with a purchase price of $380,000. Hugh and Judy sold their previous home in January 2020. Their previous home sold for $650,000. Their basis in their old home was $305,000. They had lived in the old home for eight years. How much gain, if any, must Hugh and Judy recognize for tax purposes as a result of this sale? A) $20,000 B) $345,000 C) $270,000 D) $0 Explanation The realized gain is $345,000. The entire gain may be excluded under Section 121. They had owned and lived in the residence for eight years, easily meeting the two years of ownership and use requirement. Thus, the recognized gain (the amount subject to taxation) is $0. The exclusion is not affected by the age of the taxpayers or the purchase of the new residence.

ans d

Which of the following statements is accurate with respect to a like-kind exchange? A) No gain will be recognized on the exchange of inventory. B) Gain recognized is equal to the gain realized on the exchange plus the boot received. C) The amount of gain recognized will reduce the taxpayer's basis in the property received. D) No gain will be recognized unless the taxpayer receives boot. Explanation In a like-kind exchange, the gain recognized is always the lesser of the gain realized or the boot received. If there is no boot received, there is no gain recognized. Inventory is not eligible for like-kind exchange treatment—thus, gain would be recognized. The basis in the acquired property is the FMV of the acquired property, reduced by the gain realized but not recognized (the deferred gain).

ans d

Which one of the following is NOT a federal taxation function of the economic objective? A) Reduction of taxes during a recession to stimulate the economy B) Promoting full employment C) Restricting spending through greater taxation D) Charitable deduction Explanation The reduction of taxes in order to stimulate the economy is due to the economic objective, as are restricting spending and promoting full employment. Social objectives of the federal taxation system include the charitable deduction, excluding life insurance proceeds from taxation, and renovation of a historic home.

ans d

Which one of the following is NOT a main source of federal tax revenue? A) Payroll taxation B) Individual income taxation C) Corporate income taxation D) Taxation of taxpayers living abroad Explanation The three main sources of federal tax revenue are individual income taxes, corporate income taxes, and payroll taxes; taxation of taxpayers living abroad is not a main source of federal tax revenue.

ans d

Which one of the following statements is CORRECT regarding the effect of the taxpayer's death on suspended passive losses? A) They are fully deductible upon the death of the taxpayer. B) They are completely nondeductible upon the death of the taxpayer. C) The death of the taxpayer has no effect on the suspended passive losses. D) They are nondeductible up to the amount of the step-up in basis of the activity. Explanation They are nondeductible up to the amount of the step-up in basis of the activity.

ans d

Willis has an active participation rental real estate activity. Last year, he had losses of $15,000 from the active participation real estate, and his AGI was $225,000. The $15,000 of losses were suspended due to his AGI. In the current year, Willis has an AGI of $90,000 and $6,000 of current losses from his real estate rental activity. What amount of loss may Willis deduct in the current year? A) $25,000 B) $6,000 C) $15,000 D) $21,000 Explanation The suspended losses of $15,000 from a former year plus the current losses of $6,000 equals $21,000.

ans d

Which of the following are items of tax preference or adjustments for the individual alternative minimum tax? Deduction for gambling losses Excess intangible drilling costs Bargain element on exercise of an ISO Cost depletion deductions A) I, II, and IV B) II, III, and IV C) I and III D) II and III

ans d By definition, the excess intangible drilling costs (from an oil and gas activity) and the bargain element on exercise of an ISO are preference items or adjustments for the individual AMT. Cost depletion is not an AMT preference item, but percentage depletion is. Gambling losses (to the extent of gambling winnings) is an itemized deduction for both regular tax and AMT purposes.

Danielle created a revocable trust for her two minor sons. She named her bank as trustee. The trust property earned $30,000 in the first year and had taxable income of $28,000 after deducting expenses. This income was left to accumulate for future distributions to be made to each son equally when the youngest son attains age 18. To which of the following will the income of the trust be taxable? A) The oldest son after attaining age 18, then the sons equally after the youngest son attains age 18 B) The trust C) Both sons equally D) Danielle

ans d The trust income will be taxed to the grantor, as the trust is revocable. A revocable trust is treated as a grantor trust.

John is a former University of Georgia student. The Alumni Relations Office allows John to purchase season football tickets in advance. The price is the usual amount charged or $160 per seat. If John purchases two seats for this year's football season, how much of a tax deduction is he entitled to take on his tax return? A) $160 B) $320 or 50% of his adjusted gross income, whichever is less C) $320 or 30% of his adjusted gross income, whichever is less D) $0

ans d This is not a donation. It is a purchase. If the $320 was considered a charitable contribution, it would only be deductible to the extent that it exceeded the value of the product or service. In this case, it clearly states that John is allowed to purchase the tickets in advance, although he must pay the usual price for the tickets. There is no charitable gift in this situation.

All of the following are examples of self-employment income for purposes of the self-employment tax except A) income from a sole proprietorship. B) net Schedule F income. C) net Schedule C income. D) salary paid to an S corporation shareholder.

ans d Wages and salary paid to an S corporation shareholder are not self-employment income.

Which one of the following is NOT a deduction for adjusted gross income? A) Self-employed retirement plan (Keogh) contribution B) Sole proprietorship loss C) Standard deduction

ans: c Deductions for AGI are items deducted on the front page of the Form 1040. The standard deduction is taken on the back of the 1040, as a deduction from AGI. LO 1.3.1

Which one of the following items is NOT included in the computation of total income on the Form 1040? A) Tips received B) Partnership income C) Penalty on early withdrawal of savings D) Sole proprietorship loss

ans: c Tips received, partnership income, and a sole proprietorship loss are all included in arriving at total income. The penalty on an early withdrawal of savings is an adjustment to income.

Jacinta, an investor, has the following items related to her investments during the current tax year: Investment interest expense$4,000Dividend and interest income$3,500Investment adviser's fees$1,750Adjusted gross income$50,000 What is Jacinta's maximum allowable investment interest expense deduction for the current year? A) $3,500 B) $2,250 C) $1,750 D) $4,000 Explanation The investment interest expense deduction is limited to the taxpayer's net investment income of $3,500. Net investment income is simply the investment income of $3,500. The inclusion of the dividend income results in the largest investment interest expense deduction. However, including the dividends in investment income results in forgoing the potential preferential rates on the dividends. The investment adviser's fees are not deductible and do not enter into the calculation.

ans a

Tom Bell has investment income (interest) of $8,000 in the current year. He paid $1,200 in investment adviser fees and had $7,000 of investment interest expense. His AGI is $35,000. What amount of investment interest expense may be deducted in the current year as an itemized deduction? A) $7,000 B) $6,500 C) $6,800 D) $8,000

ans a Investment interest expense is deductible up to the amount of investment income. The investment income is the interest income of $8,000. However, the deduction cannot exceed the actual investment interest expense of $7,000. Historically, the adviser fees would impact the calculation, but the Tax Cuts and Jobs Act of 2017 (TCJA) eliminated the Tier II miscellaneous itemized deductions.

Samantha received the following dividends in 2020 from her portfolio: Ordinary dividends from HOT stock, a publicly traded company Dividends from Sky High Realty and Trust, a publicly traded REIT Life insurance dividends from her whole life policy Qualified dividends from BET stock, a publicly traded company Which of the above is NOT considered taxable? A) III only. B) IV only. C) II and IV. D) I and II.

ans a Life insurance dividends are considered a return of premium paid (provided the cumulative dividends received over the life of the policy do not exceed the basis in the policy) and thus are not taxable. The other choices listed are taxable. Qualified dividends are eligible for long term capital gains rates. REIT dividends may qualify for a QBI deduction but nonetheless will still be taxable.

Lindsey is age 2 and her total income was $6,000 in qualified dividends in 2020. What is the tax on the dividends at Lindsey's rate? A) $0 B) $30 C) $143 D) $95

ans a Lindsey is in the 10% marginal income tax bracket. She can use the long-term capital gains tax rate on qualified dividends received. At her income and filing status, that capital gain tax rate is 0%.

Laura donated $250 to her daughter's Girl Scout troop and also bought six raffle tickets at $5 each from the troop. She also performs bookkeeping services for her church monthly, reconciling bank statements and preparing financial statements. She is a certified public accountant and the services she performs would have cost the church $500 if she had charged them her normal fees. How much can Laura deduct as a charitable deduction (without regard to AGI limitations)? A) $250 B) $780 C) $280 D) $750

ans a Only the $250 donation is a deductible charitable donation. Raffle tickets and donation of services are nondeductible.

Cindy has adjusted gross income (AGI) of $350,000. Included in the AGI is passive income of $40,000 and passive losses of $55,000, $40,000 of which she uses to offset the passive income and $15,000 of which is subject to carryforward. Which one of the following activities has the greatest potential for reducing Cindy's tax liability? A) Investing in a real estate partnership in which she will not materially participate that is producing passive losses B) Investing in an oil and gas limited partnership that is generating losses C) None of these options will reduce Cindy's tax liability D) Investing in "active participation" rental real estate that is producing a loss Explanation The active participation deduction is eliminated at $150,000 of AGI. The oil and gas limited partnership and the equipment-leasing limited partnership would produce more passive losses that are nondeductible. Therefore, none of the options are viable.

ans c

Dave owns equipment that has an adjusted basis of $10,000 and a fair market value of $75,000. Through an exchange, he acquires new equipment from Rachel that has a fair market value of $60,000 and an adjusted basis of $35,000. In the exchange, Dave receives $15,000 from Rachel. What is the amount of gain or loss, if any, recognized by Dave in the exchange? A) $50,000 B) $10,000 C) $65,000 D) $15,000 Explanation In the exchange, Dave received new equipment with a fair market value of $60,000 and cash of $15,000. He gave up an adjusted basis in his property of $10,000. The difference between $75,000 and $10,000 is the gain realized, $65,000. Because this is not realty, it is not eligible for 1031 exchange treatment as determined by the 2017 Tax Cuts and Jobs Act). Thus, the transaction is treated as a sale and subsequent purchase. The entire gain of $65,000 is recognized and taxable.

ans c

Diana has asked Alfredo to sign a premarital agreement. Alfredo is a Canadian citizen. Which of the following are characteristics of a valid and enforceable premarital agreement? Once executed, it is binding in all 50 states and Canada. It is not binding without proper disclosure. It should be used with the intention of facilitating a divorce. There should be a written agreement with the willingly executed signatures of both parties. A) III and IV B) II and III C) II and IV D) I and II Explanation To be valid, a premarital agreement must be in writing and contain a complete disclosure of each party's financial situation. Enforceability requirements vary from state to state. It cannot be intentionally used to facilitate a divorce.

ans c

During the current tax year, Paul has the following items from his four investments: Passive income from a publicly traded limited partnership$15,000Passive loss from a publicly traded limited partnership$10,000Passive income from a nonpublicly traded limited partnership$8,000Passive loss from a nonpublicly traded limited partnership$16,000 What is the total amount, if any, of passive losses that may be deducted during the current year? A) $26,000 B) $10,000 C) $8,000 D) $16,000 Explanation Only passive losses from nonpublicly traded limited partnerships may be offset against income from other nonpublicly traded limited partnerships. Thus, $8,000 of the $16,000 loss may be deducted. The passive loss from a PTP must be held in suspense until that same activity generates income (or until a taxable disposition of the PTP).

ans c

Paul, age 16, is listed as a dependent on his parents' income tax return. During 2020, he earned $2,600 from a summer job. He also earned $2,600 in interest and dividends from investments that were given to him by his uncle five years ago. How much of Paul's income, if any, will be taxed to him in 2020 using his uncle's marginal tax rate of 32%? A) $2,600 B) $400 C) $0 D) $2,000

ans c When applying the kiddie tax, the parents' marginal tax rate is always used (regardless of the source of the property generating the unearned income). Therefore, none of the income is taxed to Paul using the uncle's tax rate. The $400 of income ($2,600 − $2,200) is taxed to Paul at his parents' marginal tax.

During the current tax year, Jim has $10,000 of passive income from a publicly traded limited partnership. He also has a nonpublicly traded limited partnership that will generate a $10,000 passive loss. How much of this passive loss, if any, is deductible by Jim during the current tax year? A) $6,500 B) $1,000 C) $10,000 D) $0 Explanation Income from a publicly traded limited partnership may not be offset by any other passive losses. Remember that income and losses from nonpublicly traded partnerships may be used to offset each other.

ans d

Kevin Riley anticipates adjusted gross income of $120,000 for the current tax year. He has made no charitable gifts during the year, but now he wants to give his church a stamp collection with a fair market value of $70,000. Kevin paid $38,000 for the collection five years ago. The collection is appreciated tangible personal property that is unrelated to the church's exempt function. What is the maximum allowable charitable deduction Kevin can receive during the current year if he makes an immediate gift of the stamp collection? A) $36,000 B) $60,000 C) $24,000 D) $38,000

ans d The answer is $38,000. Since this is use-unrelated property, the allowable deduction is limited to his basis.

Kurt and Allison Long are married and file a joint income tax return. Their adjusted gross income (AGI) is $180,000 per year. On last year's tax return, the Longs claimed a $1,200 credit for child care expenses. The Longs are in the 22% marginal income tax bracket. What amount of deductions for AGI would be required to equal the tax benefit of the $1,200 child care credit?

$1,200 divided by the 22% marginal income tax bracket gives us $5,455.

A small business with under $20 million in gross receipts and no inventory should most likely use which accounting method? A) LIFO B) Accrual C) Cash D) Hybrid

A typical small business would use the cash method of accounting. Due to cost and complexity, the accrual and hybrid methods would NOT be recommended. A business is not required to use the accrual method until it has at least $26 million in gross receipts (2020). A business that has no inventory would not use LIFO.

What percentage, if any, of a taxpayer's self-employment tax may be deductible as an adjustment to income? A) 0% B) 25% C) 50% D) 75%

ans c Only half (50%) of a taxpayer's self-employment tax liability is deductible as an adjustment to income.

All of the following statements regarding above-the-line deductions are CORRECT except A) these deductions are subtracted from gross income in determining adjusted gross income. B) these deductions are allowable regardless of whether the taxpayer claims itemized deductions. C) these deductions are subtracted from adjusted gross income in determining taxable income. D) some above-the-line deductions include deductible contributions to IRAs.

ans: c Deductions that are subtracted from adjusted gross income in determining taxable income are below-the-line deductions.

Jeff Munroe has an annual salary of $140,000 and is not an active participant in a company-maintained retirement plan. He had the following financial transactions during the current tax year: Received a $100,000 cash inheritance due to the death of his brother Received unemployment compensation of $2,000 Had a Schedule C loss of $10,000 (assume material participation) Made an IRA contribution of $6,000 Paid qualified student loan interest of $2,000 What is Jeff's total income for the current tax year?

A) $126,500 B) $142,000 C) $132,000 D) $124,500 ans: c Explanation The $140,000 salary is reduced by the $10,000 self-employment loss and increased by the unemployment compensation of $2,000. The inheritance is excluded. The IRA contribution is a potential adjustment to income, as is the student loan interest. Thus, those items do not affect the total income. Remember that total income is the figure approximately two-thirds of the way down the front of the 1040. It is the figure from which allowable adjustments to income are subtracted.

Claudia had her art collection appraised and discovered some of the artwork had declined in value since she inherited it from her mother. She wants to donate some of the pieces to charity. Can she take the cost of the appraisal fee as a deduction?

The costs of an appraisal are not deductible.

A taxpayer in a 35% marginal income tax bracket holds a limited partnership interest in a low-income housing activity. A deduction-equivalent tax credit of $22,000 flows through to the taxpayer. What is the amount of income tax that may be offset by this deduction-equivalent tax credit?

The deduction-equivalent tax credit of $22,000 may be used to offset the income tax on $22,000 of income. For a taxpayer in a 35% marginal income tax bracket (MITB), the credit may offset $7,700 ($22,000 × 35%).

All of the following statements regarding the installment method of reporting gain from a disposition of property are correct except A) the installment sale method may be used for securities sold in the secondary market. B) the payments received under an installment sale may each include capital gains, return of capital, and interest. C) an installment sale is a sale of property in which the seller receives at least one payment after the year of sale. D) the installment method permits the seller to spread out the taxable gain over more than one year. Explanation The installment sale method cannot be used for inventory or securities traded in the secondary market.

ans a

Which of the following are allowable itemized deductions for purposes of computing the alternative minimum tax? Charitable deductions Qualified housing interest Gambling losses to the extent of gambling winnings Property taxes A) II, III, and IV B) I, II, and III C) I and II D) I and III

and b Statement IV, property taxes, is the only itemized deduction listed that is not allowed for AMT purposes. State and local income taxes are also disallowed.

Amanda's share of general partnership net income is $60,000 in 2020. In addition, her distributive share of S corporation income is $10,000. What is Amanda's self-employment tax, if any, for 2020? Round your answer to the nearest dollar. A) $10,710 B) $9,891 C) $0 D) $8,478

ans d

John and Karen Postman will spend a total of $5,000 on day care for their two children (ages 9 and 10) in the current tax year. These expenses were incurred to allow both John and Karen to work outside the home. Their adjusted gross income is estimated at $138,000. What is the amount of child and dependent care credit, if any, to which they are entitled?

The maximum amount of qualifying expenditures on which the credit may be based is $3,000 per child, or $6,000 for two or more children. In this situation, they spent $5,000. This is multiplied by 20% for taxpayers with an AGI greater than $43,000. Thus, $5,000 × 20% = $1,000.

Sheila, a single taxpayer, has taxable income of $460,000. Included in the taxable income is $50,000 of qualified dividends. At what rate(s) will her qualified dividends be taxed? A) 25% B) 15% and 20% C) 15% only D) 20% only

ans b The qualified dividends straddle the $441,450 breakpoint (for 2020). Thus, a portion fall into the $40,001 to $441,450 range and are taxed at 15%. The dividends above the $441,450 breakpoint are taxed at 20%.

Which of the following statements regarding limited partnerships is CORRECT? A limited partner is subject to the passive activity rules when accounting for income and losses from the limited partnership. The limited partner is liable to the creditors of the partnership only to the extent of that partner's contributed or promised cash or property. A) Neither I nor II B) II only C) I only D) Both I and II

ans d

What is Bobby's total self-employment tax on his Schedule C net income of $50,000? Round your answer to the nearest dollar. A) $7,650 B) $3,825 C) $3,533 D) $7,065

ans d Bobby's total self-employment tax is $7,065 ($50,000 × 0.9235 = $46,175; $46,175 × 0.1530 = $7,065).

Several years ago, John established a 2503(c) trust for his niece, Cindy. John named his attorney as trustee and gave him broad investment authority. Cindy turned age 21 this year and acquired the right to revoke the trust, but she chose to allow the trust to continue until she is age 25. Which of the following correctly identifies the taxpayer, if any, who must pay tax on the trust income?

Because Cindy waited past her 21st birthday, when she had the right to revoke the trust, she is responsible for taxes on the trust given to her.

John owns a classic automobile that had a cost basis of $32,000. John paid $38,000 to have the automobile fully restored. John sells the automobile through an installment sale for $100,000. John is to receive a $25,000 down payment in the current year, and $15,000 per year for five years, beginning this year. What amount of gain must John recognize during the current year? A) $12,000 B) $4,500 C) $12,800 D) $7,500 Explanation The profit on the sale was $30,000 divided by the $100,000 contract price, which equals a 30% gross profit percentage. This is multiplied by the $40,000 of payments received during the year to calculate the amount of gain recognized, $12,000. The $38,000 of restoration costs are capitalized, added to basis, to give us the $70,000 basis.

ans a

Which of the following married couples may file their federal income tax return using the married filing jointly (MFJ) status? A) Terry and Edie divorced last year but did not move into separate homes until November of this year. B) Sara has income for the tax year but Jack does not. C) Mark and Beth are both self-employed and have different fiscal years to accommodate their businesses. D) Paul and Josie will be married next year on New Year's Day.

ans b Because Mark and Beth have different fiscal years to accommodate their businesses, their tax years do not begin on the same date, and they may not file as MFJ. In order to use the MFJ status, a couple must be legally married on the final day of the tax year.

Assume that married taxpayers filing jointly have a taxable income of $466,500. What is the taxpayers' effective tax rate? You will need to use the tax rate schedule found in your materials. A) 35% B) 22% C) 47% D) 24.2% Explanation The effective income tax rate is the amount of income tax ($112,865) divided by the taxable income of $466,500. This gives us 24.2%. Taxable income$466,500Less (from tax rate schedule)(414,700)Amount over $414,700$51,800Times (marginal rate, from tax rate schedule)35%Tax on amount over $414,700$18,130Plus (from tax rate schedule)94,735Total Tax$112,865

ans d

Christopher's wife, Sarah, died last year and he has been living alone in their home since then. What filing status should he use when filing his income tax return for this year? A) Married filing jointly B) Qualifying widower (surviving spouse) C) Head of household D) Single Explanation Christopher may only file as a single taxpayer. He has no dependent children or other dependents in his household and does not qualify for either qualifying widower or head of household status.

ans d

Frank, a taxi driver, inadvertently fails to report approximately $1,200 of his tips received during the tax year. If the IRS imposes a penalty due to the underreported income, it would most likely impose a penalty equal to A) 75% of the deficiency. B) 50% of the deficiency. C) 20% of the deficiency plus 50% of the interest. D) 20% of the deficiency. Explanation The negligence penalty is likely to be imposed, due to the failure to make a reasonable effort to comply with the requirements of the Code. This carries a penalty of 20% of the deficiency.

ans d

Which one of the following is NOT a group of the IRS Regulations? A) Legislative B) Procedural C) Interpretive D) Implementative Explanation Regulations can be classified into three groups: (1) legislative, (2) interpretive, and (3) procedural.

ans d

Molly's grandparents gifted her with substantial securities at her birth eight years ago. In 2020, she has dividends of $10,000 and brokers' fees of $800 on the activity in the account her parents manage for her. What is her net unearned income taxed at her parents' rate? A) $10,000 B) $9,200 C) $8,950 D) $7,800

ans d Some of Molly's unearned income is taxed at her parents' rate and is calculated as follows: $10,000 UI - $1,100 (standard deduction) - $1,100 (greater of $1,100 for 2020 or amount of allowable itemized deductions directly connected with the production of the unearned income) = $7,800

John and Mary West, married taxpayers filing jointly, have itemized deductions consisting of the following: Home mortgage interest$12,000 State income taxes$18,000 Property taxes$5,150 Charitable contributions$2,250 Unreimbursed employee business expenses$3,200 Medical expenses$14,000 Sales taxes paid$2,650 The Wests' AGI for 2020 is $400,000. What is the amount of allowable itemized deductions?

A) $14,250 B) $54,600 C) $37,400 D) $24,250 ans: d Explanation Unreimbursed employee expenses are no longer deductible since the Tax Cuts and Jobs Act (TCJA). The medical expenses are deductible only to the extent that they exceed 7.5% of AGI (for 2020), which they do not. The sales taxes would only be deductible in lieu of state income taxes. The overall deduction for taxes (state, local, and property) is limited to $10,000 as a result of TCJA. Home mortgage interest$12,000State, local, and property taxes$10,000Charitable contributions$2,250Total itemized deductions$24,250

Steve and Allison Parker, a married couple in their 40s, file a joint return. They earned combined salaries of $185,000. They received dividend and interest income of $860 from mutual funds. They have allowable itemized deductions of $14,000. They have net capital losses of $5,200. They have two children, ages 12 and 14. What is their taxable income for the 2020 tax year?

A) $168,860 B) $182,860 C) $156,660 D) $158,060 ans: d Explanation The salaries combined with the income from the investments total $185,860. This is reduced by the $3,000 net capital loss to leave an AGI of $182,860. Remember that only $3,000 of net capital loss may be deducted in a given tax year. The AGI is then reduced by the greater of the itemized deductions ($14,000) or the standard deduction ($24,800 in 2020). The deduction for personal and dependency exemptions was repealed by Tax Cuts and Jobs Act (TCJA).

Your client, Hal Meyer, will receive a deductible loss of $15,100 from a working oil and gas interest. Hal is in a 35% marginal income tax bracket and has asked you the approximate amount of tax savings that this will generate. What is the approximate amount, if any, of tax savings generated by this loss?

In a 35% tax bracket, a $15,100 loss deduction will save $5,285. Thus, $5,200 is the closest answer.

Which of the following is NOT a step in the tax calculation process?

The following are involved in the income tax computation: subtracting adjustments to income from total income to get adjusted gross income, and deducting the greater of itemized deductions or the standard deduction from AGI to arrive at taxable income. The calculation of federal tax is on federal taxable income.

Janet and Bruce Robinson, both age 43, are married taxpayers filing jointly. They have itemized deductions consisting of the following: Home mortgage interest$19,500 State income taxes$8,700 Property taxes$5,200 Charitable contributions$6,200 Tax return preparation fee$895 Unreimbursed employee business expenses$2,100 Unreimbursed medical expenses$18,460 Their AGI for 2020 is $466,000. What is the amount of their allowable itemized deductions?

The total itemized deduction amount is $35,700. Note that the tax preparation fee and the unreimbursed employee business expenses are not deductible. The medical expenses are deductible only to the extent that they exceed 7.5% of AGI for 2020, which they do not. The deduction for the state income taxes and the property taxes is capped at $10,000. Taxes of $10,000, mortgage interest of $19,500, and charitable contributions of $6,200 total $35,700.

Bradley, Inc., sold some land on June 30 of the current year for a $15,000 gain. The land was originally purchased three years ago and was classified as a Section 1231 asset. This was the only asset sale for this year. In the previous year, Bradley, Inc., had an $8,000 net Section 1231 loss. For the current year, Bradley's net Section 1231 gain is treated as A) a $7,000 long-term capital gain and a $8,000 ordinary gain. B) a $8,000 long-term capital gain and a $7,000 ordinary loss. C) a $15,000 ordinary gain. D) a $15,000 ordinary loss. Explanation When the taxpayer has a net Section 1231 gain for the year, the lookback rule may recapture some or all of the net gain as ordinary income. To the extent the lookback rule does not apply, the net gain is treated as a long-term capital gain. When determining how much of the gain is treated as capital gain and how much must be treated as ordinary gain, the taxpayer is looking back to the five previous taxable years, not including the current taxable year.

ans a

Ann Hamilton owns 500 shares in the XYZ S&P 500 Index Fund. The basis of her investment in this fund is $4,500, while the fair market value is only $2,000. She wants to sell her shares to "lock in" the $2,500 loss, but she is considering buying 500 shares of the GRC Small-Cap Index ETF the following week because she believes that the value is going to increase significantly over a longer period. As her planner, what can you accurately tell Ann about this scenario? A) If the loss were disallowed, the basis in the newly acquired shares would be decreased by the disallowed loss. B) The loss would be a fully deductible capital loss. C) She should wait a minimum of 61 days after the sale to repurchase the shares so that the loss may be recognized. D) The basis in the newly acquired shares would be the amount paid for those shares, increased by the $2,500 disallowed loss.

and b The wash sale rule disallows a loss if substantially identical securities are purchased prior to 30 days after the sale that resulted in the loss. The basis of the acquired securities is increased by the amount of the disallowed loss. The S&P 500 mutual fund should not be substantially identical to the small-cap ETF because the funds track very different indices and because of the difference in the way ETFs trade compared with mutual funds.

Which of the following is NOT a step in the tax calculation process?

A) Claim allowable tax credits. B) Calculate federal tax on total income. C) Deduct the greater of itemized deductions or the standard deduction from AGI to arrive at taxable income. D) Subtract adjustments to income from total income to get adjusted gross income. ans: b Explanation The following are involved in the income tax computation: subtracting adjustments to income from total income to get adjusted gross income, subtracting tax withholdings from total tax liability, and deducting the greater of itemized deductions or the standard deduction from AGI to arrive at taxable income. Credits are applied to tax liability. The calculation of federal tax is on federal taxable income.

Jack was divorced on March 30 of the current year and has not remarried as of the last day of the tax year. He lives alone in his condo. His ex-wife, Mary, has custody of their son Jack Jr. What is Jack's filing status for the current tax year?

A) Head of household B) Married filing jointly C) Single D) Married filing separately ans: c Explanation A taxpayer who is unmarried, legally separated, or divorced and does not qualify for any other filing status must use the single filing status. Jack does not have custody of his son and does not qualify for head of household status.

Which one of the following is allowable in the computation of total income?

A) Net capital losses of up to $5,000 B) Charitable contributions C) Loss from a sole proprietorship D) Tax credits ans: c Explanation Remember that the total income is the amount shown about two-thirds of the way down the front of the Form 1040. It is the amount before the deduction for adjustments to income. Certain deductions are allowed in the computation of total income, such as the deduction for sole proprietorship losses or net capital losses up to $3,000. Charitable contributions are an itemized deduction.

Which one of the following statements is true regarding self-employment taxes?

A) Net earnings from self-employment must be calculated under the accrual method of accounting. B) A taxpayer is allowed to deduct one-half of his self-employment tax liability as an adjustment to income. C) Self-employed taxpayers are subject to employer withholding. D) The wage base is not adjusted annually for cost of living increases. ans: b Explanation A taxpayer may deduct one-half of his self-employment tax liability as an "above the line" adjustment to income. The wage base is adjusted annually for cost of living increases. Net earnings from self-employment are determined under the same accounting method as that used for income tax purposes. Self-employed taxpayers are not subject to employer withholding.

Which of the following who do not maintain a household for a dependent must use the single filing status?

A)Legally separated taxpayer B)Divorced taxpayer C)All of these D)Unmarried taxpayer answer: c Explanation A taxpayer who is an unmarried, legally separated, or divorced individual and does not maintain a household for a dependent must use the single filing status.

Neil McElroy is an engineer for Causley Computer Inc. In addition, Neil operates a janitorial service that cleans several local office buildings. Neil was divorced in 2019, and his wife received custody of their two children. He has assembled the following information for preparation of his tax return for the current tax year. Neil's salary$71,500Interest income$9,500Monthly alimony paid to ex-spouse$1,500Monthly child support$500Purchase of equipment for use in janitorial service$10,000IRA contribution$6,000 Based on the information given, which of the following are fundamental methods of managing Neil's tax liability? Tax credit: Neil could take an investment tax credit for purchases of qualifying business equipment. Deductions for AGI: Neil may deduct alimony payments of $18,000 made to ex-spouse. Deductions for AGI: Neil may deduct child support payments of $6,000. Exclusions: Neil could have invested in municipal bonds to receive tax-free income.

Neil may not deduct alimony paid to his former spouse because the deduction is disallowed for alimony under divorce decrees in 2019 and thereafter. He may invest in municipal bonds to receive tax-free income. There is no investment tax credit for equipment purposes. Some students confuse this with the Section 179 expense election, but that provision provides a deduction, not a credit. Child support payments are specifically nondeductible.

Tim Jones is single, 21 years old, and in his third year of college. He has an AGI of $35,000 and receives no support from his parents. The college is a Title IV institution where students are eligible to receive federal financial aid, and Tim is pursuing an undergraduate degree in criminal justice. When Tim was 13, his parents established a Uniform Transfers to Minors Act (UTMA) for him, and funded it with EE savings bonds. When Tim was a freshman, he was convicted of a felony drug possession charge. Which one of the following is CORRECT regarding Tim's situation? A) Tim qualifies for the American Opportunity Tax Credit. B) Tim qualifies for the Lifetime Learning Credit. C) Tim could use both the American Opportunity Tax Credit and the Lifetime Learning Credit in the same year. D) Tim may redeem the EE bonds potentially tax free if the proceeds are used for his qualifying education expenses.

and b Tim qualifies for the Lifetime Learning Credit. His AGI is under the phaseout range. He is pursuing a degree at an eligible institution. The felony drug conviction would preclude the use of the American Opportunity Tax Credit but not the Lifetime Learning Credit. There is no exclusion available for EE bonds unless they are held by the individual who purchases the bonds or unless they are held jointly with a spouse. A bond that has been gifted to another taxpayer does not qualify for the exclusion. The American Opportunity Tax Credit and the Lifetime Learning Credit may not be claimed in the same year for the same student.

Which one of the following may enable a direct participation program to provide specific tax advantages to the investors? A) Passive activity loss rules B) Partnership basis rules C) Special allocations D) At-risk rules Explanation A special allocation is an allocation of one or more items of income, gain, losses, deductions, or credits that depart from the partner's general profit and loss sharing ratio. In other words, it may not be necessary to split all of these items pro rata among all of the owners. Certain deductions may be allocated to certain individuals rather than dividing it among all participants.

and c

In 1991, John Idler purchased a single premium whole life insurance policy. In the current year his medical expenses are $15,000 and his AGI is $75,000. What is the tax implication to John if he borrows the interest from the policy's accumulated cash value to pay his current year's medical expenses? A) John will not be required to report the amount borrowed as income and will not be allowed a medical expense deduction. B) John will not be required to report the amount borrowed as income, but he will be allowed a medical expense deduction. C) John will be required to report the amount borrowed as income, but he will not be allowed a medical expense deduction. D) John will be required to report the amount borrowed as income and will be allowed a medical expense deduction.

ans D Amounts borrowed on a single premium whole life policy issued on or after June 21, 1988 (a MEC), are taxable on a last-in, first-out basis; thus, the earnings would be taxable. A medical expense deduction will be allowed regardless of the source of the funds, since the payment would be for a valid medical expense.

A client sold an apartment building last year for $100,000, paying a sales commission of $5,000 plus $2,500 in closing costs. The building originally cost $80,000 20 years ago. Total straight-line depreciation of $40,000 had been taken. The building had a mortgage of $60,000 that was assumed by the buyer. The client is in the 24% marginal income tax bracket. What is the purchaser's cost basis? A) $100,000 B) $92,500 C) $107,500 D) $70,000 Explanation The purchaser's cost basis is simply what the purchaser paid for the property—$100,000.

ans a

Alice is age 16 and is eligible to be claimed as a dependent on her parents' tax return. She has investment income of $4,000 and earned $1,000 babysitting in 2020. How much of Alice's income will be taxed at her individual tax rate? A) $1,850 B) $1,800 C) $1,350 D) $3,650 Explanation Alice is subject to the kiddie tax rules. Alice Unearned income (UI)$4,000 Earned income (EI)1,000 Gross income$5,000 Standard deduction(1,350) (earned income plus $350) Taxable income$3,650 UI taxed at parent's marginal rate (net unearned income)(1,800) ($4,000 - $2,200) Income taxed at child's rate$1,850

ans a

All of the following are tax avoidance techniques except A) investing in zero-coupon U.S. Treasury bonds. B) investing in tax-free municipal bonds. C) owning, rather than renting, a residence to benefit from claiming a home mortgage interest deduction. D) utilizing tax credits such as qualifying child care expenses. Explanation Taxes must be paid on accrued interest on zero-coupon Treasury bonds even though no cash income is received. The other choices are legitimate tax avoidance techniques.

ans a

An office building with an adjusted tax basis of $120,000 was destroyed by fire on January 2 of last year. On January 15 of the current year, the insurance company paid the owner $200,000. The owner reinvested $190,000 in a new office building. What is the basis of the new building under Section 1033 (the involuntary conversion rules)? A) $120,000 B) $110,000 C) $190,000 D) $180,000 Explanation Gain realized equals $80,000, and gain recognized equals $10,000 ($200,000 amount realized − $190,000 amount reinvested). This is a net postponed gain of $70,000 ($80,000 − $10,000). The basis of the new building equals $120,000 ($190,000 − $70,000).

ans a

Assume that for each of the next five years, your client, Dan, will have the following: Active income: $100,000 Investment income: $50,000 Passive income: $0 Passive loss (from nonpublicly traded partnership): $25,000 Dan is considering an investment in a nonpublicly traded partnership that requires a $100,000 initial investment and will generate cash flow (pretax passive income) of $12,500 per year at the end of each year for the next five years. Upon liquidation at the end of the fifth year, Dan will receive a total of $100,000 after taxes. Dan's after-tax rate of return from other investments is 10% and his combined federal and state marginal tax bracket is 28%. What is Dan's net present value on the investment? A) $9,477 B) ($2,523) C) ($15,023) D) $21,977 Explanation This problem requires the calculation of the net present value of the cash flows as compared to the $100,000 initial cash outlay. The net present value of the $12,500 received at the end of each of the first four years is first calculated. The present value of these four payments is $39,623. The calculation of present value of the amount received at the end of year five of $112,500 ($12,500 + $100,000) using a 10% rate yields $69,854. The $69,854 combined with the $39,623 results in a combined present value of all payments of $109,477. Subtracting the initial cash outlay of $100,000 results in a net present value of $9,477. Note that no tax liability would result from the cash flows as they would be "sheltered" by the passive losses.

ans a

Bernie and Louise are married and will file a joint return for the current tax year. They each have 401(k) plans through their employers, but neither they, nor their employer, will contribute to the plan this year. They have provided you with the following information. Bernie's salary$96,000 Louise's salary$74,000 Child support payments to Lowell's ex-wife $18,000 Net short-term capital loss$8,000 Home mortgage interest$17,200 IRA contribution—Bernie$6,000 IRA contribution—Louise$6,000 Bernie's divorce was finalized in 2013. Based on the information given, what is the couple's adjusted gross income for the current tax year? A) $155,000 B) $160,000 C) $167,000 D) $150,000 Explanation The salaries of $170,000 reduced by the $3,000 of net capital losses leaves $167,000 of total income. The $12,000 of IRA contributions is also deductible, as neither spouse is an active participant in a company-maintained retirement plan. After subtracting the $12,000 of IRA contributions, they are left with an AGI of $155,000. Even though they both have a 401(k), neither they, nor their employers, made contributions to the plans during the year, so neither spouse is treated as an active participant. The home mortgage interest is an itemized deduction and does not affect the AGI. Child support payments are not deductible.

ans a

Bob passed away during the current year. He had suspended losses from a limited partnership activity of $25,000. Bob's basis in the partnership was $1,000 and the fair market value at the time of his death was $18,000. What amount of passive losses, if any, is deductible on Bob's final income tax return? A) $8,000 B) $0 C) $14,000 D) $17,000 Explanation The suspended passive losses are "freed up" and deductible only to the extent that the losses exceed the step-up in basis. In this situation, the step-up in basis equals $17,000 (from $1,000 to $18,000). The losses of $25,000 exceed the step-up amount by $8,000.

ans a

Under the installment sale method, when must a seller-lender recognize all of the gain remaining in an installment note? When an installment sale is canceled When the lender makes a gift of the installment note to the debtor When the lender sells the installment note to a third party A) I, II, and III B) III only C) I only D) II and III Explanation All of these statements are correct.

ans a

Brady and Susie, married taxpayers filing jointly, have the following itemized deductions: Home mortgage interest$17,300State income taxes$12,100Real estate taxes$6,150Charitable contributions$11,000Unreimbursed employee business expenses$7,150Tax return preparation fee$650Investment advisers fees$1,975 Their AGI for 2020 is $387,000. What is the amount of their allowable itemized deductions? A) $38,300 B) $44,450 C) $46,550 D) $40,335 Explanation The state income tax and the property tax deduction (combined—the SALT, or state and local taxes) is limited to $10,000 annually under TCJA. The home mortgage interest ($17,300), the SALT deduction, and charitable contributions ($11,000) total $38,300. The unreimbursed employee business expenses, expenses related to the determination or collection of a tax liability, and expenses related to the production of income historically have been treated as Tier II itemized deductions, but this deduction was repealed by TCJA. LO 5.2.3

ans a

Carl and Janet are married taxpayers filing a joint tax return. In 2020, their AGI is $360,000, and their net short-term capital gain and dividend income (included in the AGI) is $90,000. They have investment interest expense of $4,000 and state and local income taxes attributable to the investment income of $6,000. What is the amount of Medicare contribution tax that they must pay? A) $3,040 B) $3,800 C) $3,420 D) $4,180 Explanation They will pay the 3.8% Medicare contribution tax on $80,000. This is the lesser of the net investment income ($80,000) or the AGI in excess of the threshold amount ($360,000 - $250,000, or $110,000). The net investment income is the investment income of $90,000, reduced by the investment expenses of $10,000. In this situation, the $80,000 of net investment income is subject to the Medicare contribution tax. Carl and Janet will pay a $3,040 Medicare contribution tax (3.8% on $80,000).

ans a

During the current tax year, Jim purchased a warehouse for exclusive use in his manufacturing business. The cost of the property was $620,000, of which $100,000 was attributable to the land. Which of the following statements identify the proper treatment of the expenditure? A portion of the cost attributable to the building may be deducted under Section 179. The $100,000 attributable to the land must be capitalized and may not be depreciated. The $520,000 attributable to the building must be capitalized and depreciated. The entire $620,000 must be capitalized and depreciated. A) II and III B) II only C) I and II D) IV only Explanation Land is not a depreciable asset—only "wasting" assets are subject to depreciation. The building must be capitalized and depreciated over a period of 39 years. Section 179 generally does not apply to realty; it applies to tangible personalty used in the active conduct of a trade or business.

ans a

Eleven months ago, Lynnette received 1,000 shares of stock from her uncle, Joseph. Joseph purchased the stock eight years ago for $12 per share. The fair market value on the date of the gift to Lynnette was $9 per share, and she sold the stock today for $5 per share. What is the amount and character of Lynnette's loss from the sale of the stock? A) $4,000 short-term capital loss B) $7,000 long-term capital loss C) $3,000 long-term capital loss D) $3,000 short-term capital loss Explanation There are two components to this question. What is the basis, and is there tacking of the holding period? When the fair market value on the date of the gift is less than the donor's basis in the asset, the donee's basis in the asset for purposes of determining a loss is the asset's FMV on the date of the gift. In this situation, the $9 per-share value on the date of the gift would be Lynnette's basis. The next issue is the "tacking" of the holding period. In a situation where the donee uses the FMV as the basis, there is no tacking of the holding period. In this situation, Lynnette used the FMV; thus, she uses her own holding period of 11 months. If the donee uses the donor's basis, then the holding period is tacked. In other words, the donor's holding period is added to ("tacked") the donee's holding period.

ans a

For a taxpayer with an AGI in excess of $150,000 for the prior tax year ($75,000 if married filing separately), the estimated tax penalty safe harbor is A) 90% of the current year's tax liability or 110% of the prior year's tax liability. B) 90% of the prior year's tax liability or 110% of the current year's tax liability. C) 120% of the prior year's tax liability or 80% of the current year's tax liability. Explanation The safe harbor is 90% of the current year's tax liability or 110% of the prior year's tax liability if the taxpayer's prior-year AGI exceeded $150,000.

ans a

For which of the following transactions does immediate recognition of remaining gain on an installment sale occur? Blake entered into an installment sale with Jake when he sold Jake a motorcycle but canceled the installment sale one year into the three-year installment sale term. Sarah entered into an installment sale with her sister, Josie, when she sold her an undeveloped lot but gifted the note to Josie three months later. Josh needed cash immediately and sold his installment note he had on equipment he sold to Trevor to the bank. Les needed a loan from the bank and used the installment notes he had for cars he sold to three customers as collateral for the loan. A) I, II, III, and IV B) II and III C) I and III D) IV only Explanation Under the installment sale method, immediate recognition of remaining gain occurs with the following: At the time an installment sale is canceled When there is a gift of an installment sale to the obligor-debtor when there is a sale of the installment note to another party when an installment note is pledged as collateral for a loan If an installment obligation is canceled, it is treated as a disposition. To discourage sham transactions between related parties, when an installment sale is canceled, gain is immediately recognized.

ans a

James has suspended losses from a nonpublicly traded partnership of $17,000. In 2020, he has income from a nonpublicly traded partnership of $15,000. What amount of suspended losses, if any, may James deduct in 2020? A) $15,000 B) $2,000 C) $17,000 D) $0 Explanation He may deduct (under the at-risk limitations) his share of the partnership loss to the extent of his investment in the partnership. With a $17,000 loss and only $15,000 in income, James can only deduct $15,000 of the loss.

ans a

Upon the disposition of a passive activity interest by gift, the suspended losses are A) added to the basis of the interest. B) completely lost. C) deductible in full. D) deductible to the extent the losses exceed any increase in the fair market value of the activity. Explanation Upon the disposition of a passive activity by gift, the suspended losses are added to the basis of the activity.

ans a

Jana, age 35, is the owner of a whole life insurance policy with a face amount of $100,000. The premiums on the policy are $1,400 per year. At the end of the 12th year, it is estimated that the cash value built up within the policy will exceed the premiums paid. The beneficiary of the policy is Jana's husband, Bart. Which of these is an income tax implication of Jana's whole life insurance policy? A) There is no income tax levied on the accumulation of cash value within the policy. B) The excess of the death proceeds over the premiums paid is taxable to Bart as beneficiary. C) The death proceeds from the policy are taxable as ordinary income to Bart in his capacity as the beneficiary. D) Income tax is payable on the cash value at the time that it exceeds the premiums paid. Explanation In a whole life insurance policy, the cash value accumulation is free of current income tax. There is no taxable event, generally, unless the policy is surrendered. The proceeds payable due to death of the insured are tax-free.

ans a

Jane owns a printing business. She wants to trade her old copiers for new fax machines. In the contemplated exchange, Jane will pay $750 in cash. Additional information related to the transaction is given as follows: The copiers have an adjusted basis of $1,500. The copiers have a fair market value of $1,000. The fax machines have a fair market value of $1,750. What is Jane's recognized gain or loss in this exchange? A) ($500) B) $500 C) ($250) D) $0 Explanation Jane is paying $750 plus the adjusted basis of $1,500 ($2,250); compared to the fair market value of the property received of $1,750, thus yielding a $500 loss. There is no loss recognized in a like-kind exchange. This exchange is simply treated as a sale of the asset. A loss on a Section 1231 asset may be recognized in the year of the loss. The Tax Cuts and Jobs Act (TCJA) restricted the like-kind exchange rules to real estate only. Personalty no longer qualifies for like-kind exchange treatment.

ans a

Jeff received 100 shares of stock from his aunt, Diane. Diane purchased the stock seven years ago for $12 per share. Assume that Jeff received the stock as a gift from Diane two years ago, when the fair market value was $8 per share, and assume that he sold the stock this year for $6 per share. What was Jeff's per-share basis in the stock? A) $8 B) Basis cannot be determined C) $6 D) $12 Explanation When the fair market value on the date of the gift is less than the donor's basis in the asset, the donee's basis in the asset for purposes of determining a loss is the asset's fair market value on the date of the gift. In this situation, the $8 per share value on the date of the gift would be Jeff's basis.

ans a

Jim owns an apartment building with a fair market value of $225,000 and an adjusted basis of $85,000. He wants to acquire Frank's duplex, which has a fair market value of $240,000 and an adjusted basis of $130,000. In the exchange, Jim will pay Frank $15,000 in cash. What is Jim's substitute basis in the acquired duplex? A) $100,000 B) $225,000 C) $140,000 D) $240,000 Explanation Jim is receiving an FMV of $240,000 for the duplex. He is giving up an adjusted basis of $85,000 plus $15,000 cash. The difference between the $240,000 received and the $100,000 given up is the realized gain of $140,000. The gain recognized (the taxable amount reported on the income tax return) in a like-kind exchange is the lesser of gain realized ($140,000) or boot received ($0). The substitute basis in an asset acquired in a like-kind exchange is the FMV of the qualifying property received ($240,000) reduced by the gain realized, but not recognized ($140,000 - $0 = $140,000). Thus, $240,000 - $140,000 = $100,000.

ans a

Joe and Carter plan to combine their respective sole proprietorships to enable them to bid on a local automobile plant's contract to provide uniforms, shoes, and safety equipment to 2,300 employees. Joe currently operates a business that sells uniforms and safety equipment. Carter has a shoe store that specializes in work shoes for many occupations. Joe and his spouse have substantial assets and are in the 35% marginal income tax bracket. Carter is single and has a moderate net worth. His annual taxable income is $136,000, excluding the business income from the shoe store. Joe and Carter anticipate net operating losses over the first two years of $30,000, to be followed by substantial profits. They plan to share the management responsibilities equally. Both Joe and Carter admit that the business is risky, as neither one of them has had any experience with such large contracts. Which of these business forms would be most appropriate for Joe and Carter at this time? A) S corporation B) Limited partnership C) Partnership D) C corporation Explanation The S corporation is the best choice. This would allow the losses to flow through to the owners and would provide liability protection. The general partnership would not provide liability protection, and to form a limited partnership, there must be a general partner with unlimited liability, so neither of these options would be viable. The C corporation would not allow losses to flow through to the owners.

ans a

Larry and Sherry reside in a common law property state and recently became engaged. Larry is a retired sports executive with a considerable fortune and a son from a previous marriage. Sherry has never been married. Larry has presented Sherry with a five-carat diamond ring, contingent upon her signing of a premarital agreement. The income tax consequences of the premarital agreement depend in large part upon which of the following? The original owner of the ring Whether the transfer under the agreement is treated as a gift Local and state law Whether the transfer under the agreement is treated as a transfer for consideration A) II and IV B) I and II C) III and IV D) II and III Explanation The purpose of the premarital agreement is to limit the presumed effect of the marriage on property acquired prior to, or during, the marriage. The income tax consequences of the premarital agreement depend in large part upon whether the transfer under the agreement is treated as a gift (where income tax is avoided) or as a transfer for consideration (which will probably result in the recognition of significant income by one party). State law does not impact federal income tax liability. The origin of the ring is irrelevant.

ans a

Max is selling a truck that he uses in his business. He has taken $5,000 of depreciation on the truck and wants to use the installment sale method to sell the truck to Jerry for a down payment and an installment note over 36 months. He paid $40,000 for the truck and is selling it for $38,000. What are the tax consequences of this transaction? Max must recapture $3,000 of the Section 1245 depreciation taken as ordinary income in the year of the sale. Max has $5,000 of depreciation recapture taxed at the 25% tax rate. A) I only B) Both I and II C) II only D) Neither I nor II Explanation Statement I is correct. Gain recaptured under Section 1245 (depreciable personal property used in a trade or business) is taxed as ordinary income and is not eligible for installment sale treatment. Therefore, these amounts are fully recognized (taxable) as ordinary income in the year of sale. Unrecaptured Section 1250 depreciation occurs only on depreciable real property (real estate) used in a trade or business.

ans a

Nine years ago, Claire, age 55, purchased a deferred annuity that is estimated to pay her $850 per month for the rest of her life beginning at age 65. Her investment in the contract is a one-time payment of $50,000. The assumed rate of return on the contract is 3.5%. At this time, Claire is not sure whether she will need to withdraw any of her original investment prior to the starting date of the annuity. Which of these is an income tax implication of the deferred annuity for Claire? A) A nonperiodic (lump-sum) distribution will be treated on a last-in, first-out (LIFO) basis. B) Earnings on the investment are taxable in full each year to Claire as ordinary income. C) The distribution amount consisting of interest paid on the investment is taxed as a capital gain. D) Withdrawals in a lump sum are first allocated to the tax-free investment in the annuity (FIFO treatment). Explanation A nonperiodic distribution or withdrawal from a post-August 13, 1982 annuity contract is treated on a LIFO basis. In other words, to the extent that the cash surrender value exceeds the investment in the contract, taxable interest income is treated as being withdrawn first. The earnings on the investment in a commercial annuity are deferred—there is no current taxation on the earnings within the contract as long as an individual is the owner (or treated as an owner) of the contract.

ans a

On April 1 of the current tax year, Susan sold her principal residence for a total price of $501,000; $301,000 was in cash, with the buyer assuming a $200,000 mortgage on the house. Susan purchased the house 15 years ago for $290,000, but she has an adjusted basis of $80,000 due to a Section 1034 rollover. She has not made any improvements to the house. To assist in the sale of the residence, she incurred costs of $1,500 for repairs three weeks before the sale occurred. Realtor commissions of $31,000 resulted from the sale. On May 1 of the current tax year, Susan bought a new residence for $260,000. Assume that Susan is considering renting out her new residence for two weeks (14 days) during the upcoming tax year. However, she is unsure of the income tax consequences. Which one of the following statements is CORRECT? A) The rental income is not includible in income. B) The rental income is includible in income, but mortgage interest and property taxes allocable to the rental are deductible for AGI. C) The rental income may or may not be includible in income, depending on the amount. D) The rental income is includible in full in gross income. Explanation Income from rentals for fewer than 15 days during the year are not required to be included in gross income. However, no deductions attributable solely to the rental are allowed, either. The home mortgage interest and property taxes are still deductible in full as itemized deductions.

ans a

Paul and Mary form an equal partnership to produce hammers for the military. They each have a 50% profit and loss sharing agreement. Paul contributes cash of $50,000 and property with a fair market value of $50,000 and an adjusted basis of $20,000. Mary contributes cash of $25,000 and property with a fair market value of $75,000 and an adjusted basis of $40,000. A bank gives the partnership a recourse loan of $30,000 and a nonrecourse loan of $25,000. What is the amount at-risk for Paul and Mary, respectively? A) $85,000 for Paul, $80,000 for Mary B) $127,500 for Paul, $127,500 for Mary C) $70,000 for Paul, $65,000 for Mary D) $97,500 for Paul, $92,500 for Mary Explanation The amount at-risk is measured by the amount of cash contributed, increased by the adjusted basis of property contributed, and further increased by the partner's share of recourse financing only. Nonrecourse financing is included only if it is qualified nonrecourse financing in a real estate activity only.

ans a

Paul died recently. At the time of his death, he had $12,000 in suspended losses from a limited partnership, a passive activity. His basis in the limited partnership was zero, and the step-up in basis at death was $10,000. What is the amount of suspended losses that are deductible? A) $2,000 B) $12,000 C) $10,000 D) $0 Explanation The answer is $2,000, the suspended losses ($12,000) minus the basis ($10,000).

ans a

Paul has the following items: Carryforward of prior year passive loss from: XYZ limited partnership (publicly traded)$(10,000)ABC limited partnership (nonpublicly traded)$(6,000) Current year passive income and loss from: XYZ limited partnership (publicly traded)$12,000GHI limited partnership (publicly traded)$(9,000)JKL limited partnership (nonpublicly traded)$18,000RST limited partnership (nonpublicly traded)$(14,000) What is the total amount of passive losses that Paul may deduct during the current year? A) $28,000 B) $14,000 C) $18,000 D) $30,000 Explanation Publicly traded partnership (PTP) income may only be offset by prior year losses from the same partnership. Thus, the $10,000 XYZ carryforward is deductible. Nonpublicly traded income ($18,000) may be offset by current losses ($14,000) or carryforward losses ($6,000) from any nonpublicly traded activities. Thus, the $10,000 XYZ loss and the $18,000 nonpublicly traded loss total $28,000.

ans a

Paula purchased an interest in a master limited partnership (MLP) that produced a loss of $7,000 this year. She also purchased a real estate limited partnership (RELP) that generated $10,000 of passive income this year. How much, if any, of the passive loss from the MLP could be used to offset Paula's income from the RELP in the current year? A) $0 B) $3,500 C) $7,000 D) $3,000 Explanation The answer is $0. Paula's losses from the MLP cannot be used to offset income from the RELP. MLP losses may only be used to offset income from the same MLP.

ans a

Paula purchased an interest in a publicly traded partnership and has experienced a current loss of $7,000. If she purchased a nonpublicly traded partnership with $10,000 of passive income, how much of the passive loss may be used to offset Paula's income in the current year? A) $0 B) $7,000 C) $10,000 D) $3,000 Explanation The answer is zero. Losses from publicly traded partnerships cannot be offset against income from nonpublicly traded partnerships.

ans a

Philip, a professor, earned a salary of $140,000 from a university in the current year. He received $35,000 in dividends and interest during the year. In addition, he incurred a loss of $25,000 from an investment in a passive activity. His at-risk amount in the activity at the beginning of the current year was $15,000. What is Philip's adjusted gross income (AGI) for the current year? A) $175,000 B) $115,000 C) $160,000 D) $150,000 Explanation Philip's AGI, after considering the passive investment, is $175,000 ($140,000 active income + $35,000 portfolio income). He cannot offset the passive loss against active or portfolio income. The loss may be deducted only against passive income, which he does not have in the current year.

ans a

Phillip's personal automobile was completely destroyed in a hurricane that would be declared a federal disaster. Insurance paid $6,000 on the loss. The auto's fair market value was $16,000, and his basis in it was $23,500. Phillip's AGI is $72,500. What is the amount of Phillip's deductible casualty loss? A) $2,650 B) $17,500 C) $10,250 D) $2,750 Explanation The deductible casualty loss computation begins with the lesser of the decrease in fair market value or the adjusted basis in the property. In this situation, the decrease in fair market value of $16,000 must be used. This is reduced by the insurance coverage of $6,000, and the $100 floor per occurrence. Then it is further reduced by 10% of the AGI ($7,250). Thus, $16,000 - $6,000 - $100- $7,250 = $2,650. Lesser of decrease in FMV ($16,000) or adjusted basis ($23,500) $16,000Less insurance coverage(6,000)$10,000Less $100 floor(100)$9,900Less 10% of AGI(7,250)Deductible loss on Schedule A$2,650

ans a

Question ID: 1247950 If Marla, who is single, had $2,000 in net long-term capital gains and $4,000 in net short-term capital losses, what effect will these results have on her income taxes? A) Her net capital loss for the year will be $2,000, which may be deducted from her ordinary income. B) The $2,000 net gain will be taxed at a rate of 20%. The $4,000 net loss will be allowed to reduce her taxable income. C) Her net capital loss for the year will be $2,000, which must be carried forward to a year in which she has capital gains. D) The $2,000 net long-term gain will be taxed at Marla's ordinary income tax rate. Explanation To calculate the net capital gain/loss for the year, the taxpayer aggregates all short-term and long-term capital gains and/or losses. In this scenario, the overall net capital los

ans a

Ramone has no passive income. During January of 2018, he purchased an interest in a limited partnership that will generate a $10,000 passive loss during the current tax year. How much of this passive loss, if any, is deductible by Ramone during the current tax year? A) $0 B) $10,000 C) $5,000 D) $6,500 Explanation Under the passive activity loss rules, the general rule is that passive losses are only deductible against passive income. In this situation, there is no passive income.

ans a

Six years ago, Holly purchased an office building for her retail business at a cost of $200,000. She paid $5,000 in legal fees associated with the acquisition. She paid $10,000 to remodel the interior to make the building suit her purpose. She has paid $18,000 for property taxes and $20,000 for utilities during the five years. She has taken cost recovery deductions of $35,000. She also paid $40,000 interest on the mortgage note. What is Holly's current adjusted basis in the warehouse? A) $180,000 B) $170,000 C) $165,000 D) $220,000 Explanation The $200,000 cost is increased by the capitalized costs: legal fees of $5,000 and improvements of $10,000. The cost

ans a

Steven owned a duplex that he rented to tenants. He acquired the property several years ago for $296,000. He used the straight-line method of cost recovery, which totaled $75,000. Steven sold the property in February of the current year for $330,000. Steven is in the 32% marginal income tax bracket. What is the amount and nature of the gain on the sale? A) $34,000 regular long-term capital gain; $75,000 unrecaptured Section 1250 income B) $34,000 regular long-term capital gain; $75,000 Section 1250 recapture C) $75,000 long-term capital gain D) $75,000 Section 1250 recapture Explanation The entire gain of $109,000 is treated as Section 1231 gain. $75,000 of the gain is created by the cost recovery deductions, and is subject to a maximum rate of 25% as unrecaptured Section 1250 income. The remaining $34,000 of gain is created by actual appreciation of the duplex, and is subject to the regular long-term capital gain rate of 15%.

ans a

Taxpayers owning certain resources may recover the cost of exhausting that natural resource (cost recovery) using which of the following methods? Cost depletion Percentage depletion Amortization over 15 years A) I and II B) II only C) I and III D) II and III Explanation Statements I and II are correct. Statement III is incorrect. Amortization is used for Section 197 intangible assets.

ans a

Three years ago, Zhong purchased specialized manufacturing equipment (seven-year property) at a cost of $56,000. He paid an additional $4,000 to have the equipment installed in his plant. Cost recovery deductions total $40,935. This type of specialized equipment became more and more scarce, and this year, Zhong sold the equipment for $73,000. What is the amount and character of the gain resulting from this sale? A) $40,935 Section 1245 gain, $13,000 Section 1231 gain B) $40,935 Section 1245 gain, $17,000 Section 1231 gain C) No Section 1245 gain, $53,935 Section 1231 gain D) $57,935 Section 1245 gain, no Section 1231 gain Explanation The gain realized and recognized is the difference between the $73,000 amount realized from the sale and the adjusted basis of $19,065. Thus, the total gain is $53,935. The Section 1245 cost recovery recapture is the lesser of the cost recovery deductions taken ($40,935) or the gain realized ($53,935). Thus, the Section 1245 income (cost recovery recapture) is $40,935. The remaining $13,000 of gain is attributable to actual appreciation of the asset; therefore, there is $13,000 of Section 1231 gain. The other way to look at this—the gain created by the depreciation deductions ($40,935) is treated as ordinary income, and the gain attributable to the actual appreciation of the asset ($13,000) is potential long-term capital gain under Section 1231. Remember that the installation is a cost associated with the acquisition of an asset and must be capitalized (added to basis). Thus, the original cost basis was $60,000.

ans a

Tom and Jeanette are married taxpayers filing jointly. Jeanette is the owner of a chain of successful car washes, from which she receives a substantial portion of her income in cash. Last year, the car washes had sales of $160,000, but Jeanette intentionally failed to report $12,000 of the sales that she received in cash. Which one of the following penalties is the Internal Revenue Service most likely to apply if it determines that Jeanette underreported her income? A) Civil fraud penalty B) Underreporting penalty C) Negligence penalty D) Substantial understatement penalty Explanation This is essentially taxpayer fraud that does not rise to the level of criminal fraud (most of the income was declared), but is willful. If imposed, the penalty is 75% of the portion of tax underpayment attributable to fraud.

ans a

Under the Modified Accelerated Cost Recovery System (MACRS), real estate is depreciated under which of the following methods? A) Straight-line method B) Sum-of-the-years' digits method C) 200% declining balance method D) 150% declining balance method Explanation Under the MACRS, real estate is depreciated using the straight-line method, with a mid-month convention, while personalty is generally subject to a 200% declining balance method, with a half-year convention. Note that the taxpayer may elect the straight-line method for personalty. The MACRS system applies for all property placed in service after 1986.

ans a

Vernon owns a warehouse that has a fair market value of $125,000 and an adjusted basis of $62,000. He wants to acquire Nicole's duplex, which has a fair market value of $100,000 and an adjusted basis of $47,000. In the contemplated exchange, Nicole will pay Vernon $25,000 in cash. What is Vernon's substitute basis in the acquired duplex? A) $62,000 B) $100,000 C) $47,000 D) $63,000 Explanation We must first compute the gain realized and then the gain recognized. To compute the gain realized (the actual economic gain), use the fair market value of the property received of $125,000 ($100,000 duplex plus $25,000 cash) minus the adjusted basis of the property given up ($62,000 warehouse) to equal a gain realized of $63,000. Next, we compute the gain recognized (the gain that is subject to taxation). The gain recognized is the lesser of the gain realized ($63,000) or boot received ($25,000). Thus, the gain recognized is $25,000. To compute the substitute basis in the acquired duplex, start with the fair market value of the qualifying property received (the $100,000 duplex). This is then reduced by the amount of the gain realized but not recognized (the untaxed, or deferred, gain of $38,000). The FMV of the qualifying property received ($100,000 duplex) reduced by the deferred gain of $38,000 equals the new basis of $62,000.

ans a

When does the alternative minimum tax (AMT) apply to a taxpayer? A) When the AMT calculation results in a tax liability that is greater than that resulting from the regular income tax calculation B) When the taxpayer has not withheld or prepaid a sufficient proportion of actual income tax liability C) Anytime a person is involved in passive activities D) When the taxpayer has losses from passive activities or other business functions Explanation The AMT is paid when the AMT tax calculation produces a higher amount of tax than the regular tax calculation and the AMT payable is the difference between the two.

ans a

Which of the following are considered advantages of direct participation programs? Leverage Tax conduit Special allocations Substantial economic effect A) I, II, and III B) I and II C) II and IV D) I and III Explanation All of the characteristics are advantages of a direct participation program except option IV. Substantial economic effect is actually a disadvantage that serves to limit the potential effectiveness of a direct participation program. Leverage enables the taxpayer to use losses in excess of the amount invested. A tax conduit potentially enables the taxpayer to deduct losses of the business on her individual income tax return. Special allocations allow for the optimum use of the tax benefits and deductions of a partnership.

ans a

Which of the following dispositions of Section 1245 recapture property would result in the immediate recapture of some or all of previous depreciation deductions? A) A sale for cash and an interest-bearing note B) A distribution by a partnership to its partners C) A transfer at death D) A gift of the property Explanation A sale of Section 1245 property at a gain will result in Section 1245 recapture. None of the other choices are considered taxable dispositions that would trigger recapture of depreciation (cost recovery) deductions.

ans a

Which of the following forms of business could NOT be a direct participation program (tax conduit)? A) Closely held C corporation B) S corporation C) Limited liability company (LLC) D) Limited liability partnership (LLP) Explanation The tax advantages provided by direct participation programs are founded upon the principle that most types of business organizations function as tax conduits. Therefore, a closely held C corporation cannot qualify, as there is no flow through of gains and losses.

ans a

Which of the following items received by a taxpayer are included in gross income? Unemployment compensation Child support payments Jury duty fees Awards A) I, III, and IV B) II and III C) I, II, III, and IV D) I and III Explanation Child support received is excluded, while the other choices are all included in gross income. Unemployment compensation is essentially a replacement for wages, thus taxable.

ans a

Which of the following parties may be subject to IRS penalties? A) Both the individual taxpayer and the tax preparer B) Individual taxpayer only C) Tax preparer only D) The individual taxpayer when the tax preparer has closed their business Explanation Both clients and tax professionals alike may be held accountable for tax reporting errors. Financial planners should also be aware that a tax return preparer might be subject to certain penalties. These penalties include failure to provide a taxpayer with a copy of their return, failure to keep a copy of all returns prepared for at least the last three years or to maintain a list of returns prepared, and failure to sign a return as preparer and give their tax identification number on the return.

ans a

Which of the following rules or doctrines may limit the availability of income tax benefits from a particular investment? Tax conduit The substantial economic effect doctrine The at-risk rule The passive activity loss rule A) II, III, and IV B) II and III C) I and IV D) I and III Explanation The substantial economic effect doctrine limits the ability to use special allocations in a partnership. The at-risk rule limits the ability to use leverage by attacking the use of nonrecourse financing. The passive activity loss rule limits the ability to deduct losses from activities in which the taxpayer does not materially participate.

ans a

Which of the following statements regarding Section 1033 involuntary conversions is CORRECT? For an owner-user, the replacement property must pass the functional use test. The taxpayer use test provides less flexibility than the functional use test. A) I only B) II only C) Neither I nor II D) Both I and II Explanation Statement II is incorrect. The taxpayer use test provides more flexibility than the functional use test.

ans a

Which of the following statements regarding inventory valuation techniques are CORRECT? During a period of rising prices, LIFO increases the cost of goods sold. During a period of declining prices, LIFO increases the cost of goods sold. During a period of rising prices, FIFO increases the cost of goods sold. During a period of declining prices, FIFO increases the cost of goods sold. A) I and IV B) II and IV C) II and III D) I and III Explanation During a period of rising prices, the last-in, first-out (LIFO) method treats the higher-priced inventory items as those first sold. This therefore increases the cost of goods sold. Conversely, during a period of declining prices, the first-in, first-out (FIFO) method matches the higher-priced inventory items against income. This naturally increases the cost of goods sold.

ans a

Which of the following statements regarding passive activity losses is CORRECT? When determining the amount of suspended loss that may be used against income, the at-risk rules are applied before the passive activity loss rules. If a loss is not allowed because of the at-risk limitations, the loss is a suspended loss eligible for deduction as a disposition of a passive activity. A) I only B) Both I and II C) Neither I nor II D) II only Explanation Statement I is correct. Statement II is incorrect. If a loss is not allowed because of the at-risk limitations, the loss is a suspended loss and is not eligible for deduction as a disposition of a passive activity.

ans a

Which of the following statements regarding passive activity losses is CORRECT? When determining the amount of suspended loss that may be used against income, the at-risk rules are applied before the passive activity loss rules. If a loss is not allowed because of the at-risk limitations, the loss is a suspended loss eligible for deduction as a disposition of a passive activity. A) I only B) II only C) Both I and II D) Neither I nor II Explanation Statement I is correct. Statement II is incorrect. If a loss is not allowed because of the at-risk limitations, the loss is a suspended loss and is not eligible for deduction as a disposition of a passive activity.

ans a

Which of the following statements regarding the at-risk rules and the passive activity loss limits is CORRECT? The at-risk rules are applied before the passive activity loss rules limits. If a loss is disallowed in any year because of the at-risk rules, the loss is suspended and taken in the first year that the at-risk amount is of an amount sufficient to absorb the loss. A) Both I and II B) Neither I nor II C) I only D) II only Explanation The at-risk rules are applied before the limits of the passive activity loss rules. They operate together such that, even if the taxpayer can avoid the restrictions of the at-risk rules, the passive activity loss rules will likely still preclude an income tax loss on an annual basis.

ans a

Which of the following tax factors may limit the availability of tax benefits from a limited partnership? "Passive loss" rules At-risk rules Alternative minimum tax A) I, II, and III B) I only C) III only D) I and II Explanation The Tax Reform Act of 1986 has two major provisions that have significantly reduced the benefit of tax-shelter investments: the at-risk rules and the passive activity loss rules limits. The alternative minimum tax is another example of a tax benefit limitation.

ans a

Which of these is NOT a preference item or adjustment for purposes of the individual alternative minimum tax? A) Investment interest in excess of net investment income B) The excess of percentage depletion over the property's adjusted basis C) Excess intangible drilling costs D) Interest from a qualified private-activity municipal bond issued in 2007 Explanation "Investment interest in excess of investment income" is a made-up phrase. The other choices are all preference items or adjustments. Remember that interest from private-activity municipal bonds issued in 2009 and 2010 is not a preference item. LO 3.3.2

ans a

Which one of the following best describes the role of a special allocation in a limited partnership? A) It allows an allocation of items of income and expense that is not pro rata. B) It allocates management responsibility to the general partners. C) It requires all items to be distributed pro rata based on a partner's capital account balance. D) It establishes the standards for allocating the proceeds of non-routine or "special" items of income. Explanation A special allocation allows an allocation of items in a manner that differs from the "normal" pro rata allocation of deduction, income, credit, etc.

ans a

Which one of the following objectives of the federal taxation system would include keeping prices stable? A) Economic objective B) Writing tax code C) Social objective D) Revenue raising Explanation The first, and perhaps most important, goal of the economic objective is price stability. As a social objective, JGTRRA and TCJA significantly increased the Section 179 expense limit and the amount of depreciation deductions (bonus depreciation) that may be claimed in the first year in an attempt to stimulate purchases of business assets. Revenue raising through corporate, individual, and payroll taxes is an important objective of the federal taxation system. LO 8.1.2

ans a

Which one of the following provides the greatest federal revenue from IRS taxation? A) Individual income taxes B) Corporate income taxes C) Gift taxes D) Estate taxes Explanation The three main sources of federal tax revenue are individual income taxes, corporate income taxes, and payroll taxes. Individual income tax accounts for approximately 40% of the total tax revenue collected by the federal government. The federal estate tax and gift tax actually compose only a small percentage of annual tax revenues.

ans a

Yasamin has an apartment building that she would like to exchange. Which of the following assets could Yasamin receive in a like-kind exchange? A shopping center An interest in a real estate limited partnership A vacant lot Land-grading equipment A) I and III B) I, II, III, and IV C) II and III D) III and IV Explanation In a like-kind exchange, real estate must be exchanged for other real estate. Thus, the shopping center and the vacant lot are the only assets that would be qualifying property in a like-kind exchange of realty. The land-grading equipment is personalty and thus cannot be considered like-kind property. The interest in a limited partnership is specifically not allowed as qualifying property. Remember that like-kind exchange treatment is limited to realty for realty, as a result of TCJA.

ans a

Which of the following statements regarding the alternative minimum tax is CORRECT? If the regular income tax after credits equals or exceeds the individual AMT, then no individual AMT payment is required. If the regular income tax after credits is less than the individual AMT, then the AMT is not due. A) Neither I nor II B) Both I and II C) II only D) I only

ans a Statement II is incorrect. If the regular income tax after credits is less than the individual AMT, then the AMT must be paid with this difference, referred to as the AMT payable.

Terry and Jan are married taxpayers filing a joint tax return. In 2020, their AGI is $310,000, and their net investment income (included in the AGI) is $90,000. What is the amount of their Medicare contribution tax for 2020? A) $2,280 B) $4,180 C) $0 D) $3,420

ans a Terry and Jan will pay the 3.8% Medicare contribution tax on $60,000. This is the lesser of the net investment income ($90,000) or the AGI in excess of the threshold amount ($310,000 - $250,000, or $60,000). In this situation, only $60,000 of the net investment income is subject to the Medicare contribution tax and calculates to $2,280 (60,000 × 0.038).

You are a CFP® professional and are meeting with your client Brenda to monitor her ongoing financial status. Brenda owns a vacation home in New Mexico. She is now renting the property to others for the entire year except for 10 days during the summer when she and her family used it for their vacation. The gross rental income that Brenda received is $65,000. The expenses for the home for both the rental period and her personal use total $5,000. Brenda would like you to explain how this will change her income tax situation, in particular, how much of the rental expenses are deductible. After reviewing the documents she sent to you prior to the meeting, you have an answer for Brenda. How much of a deduction for rental expenses can Brenda take on her tax return? A) $4,863 B) $3,411 C) $4,721 D) $5,000 Explanation Brenda can deduct the cost of renting the home if she occupies it for the greater of no more than 14 days per year or for 10% of the number of days the property is rented. Because Brenda occupies the house for only 10 days during the year, this test is satisfied. Even though this rental use exception is allowed, the deductible expenses related to the rental of the house are limited. Specifically, she can only deduct a portion of the actual rental expenses, which equals the number of days during the year that the house is rented to others divided by the total number of days that the house is used by either tenants or Brenda. Given 365 days per year, Brenda's tenants occupy the house all but 10 days, for a total of 355 days. She is allowed to deduct 97.26% (355 days ÷ 365 days) of the $5,000 rental expenses, which equals $4,863.

ans a

Your client, Eva, has active income of $300,000 per year and substantial unused passive losses from a nonpublicly traded limited partnership. She would like to find an investment that would allow her to utilize her passive losses. Which of these is the most appropriate investment for Eva? A) A nonpublicly traded limited partnership generating passive income B) A portfolio asset that does not generate income but will appreciate in value over time C) Certificates of deposit generating portfolio income D) A master limited partnership generating income Explanation Eva needs a passive income generator. Only the nonpublicly traded limited partnership would qualify. The MLP, a publicly traded partnership, would not qualify, as income from a publicly traded partnership cannot be offset by passive losses arising from any other source. Passive losses may not be used to offset portfolio income.

ans a

Your clients, Jane and Mark, are contemplating the purchase of a condominium to use as a rental property. They would manage the property themselves and anticipate that it would generate losses for the first few years, at least. Which of the following statements are CORRECT with respect to active participation rental real estate? The interest may be held through a limited partnership. A deduction-equivalent tax credit of up to $25,000 is available. The taxpayer must hold a 10% or greater ownership interest in the property. The taxpayer must participate in the management of the property in a bona fide sense. A) III and IV B) I, II, and III C) I and II D) II, III, and IV Explanation III and IV are the only choices that are correct, by definition. The interest in the property may not, by definition, be held through a limited partnership, and up to $25,000 refers to losses that may be deducted, not credits that may be taken. The deduction-equivalent tax credits mentioned in option II are relevant with respect to low-income housing and historic rehabilitation passive activities. LO 7.2.3

ans a

Which of the following statements regarding the kiddie tax is CORRECT? The kiddie tax provision limits income shifting by preventing families from transferring large amounts of unearned income to children and making the shift effective for income tax purposes. If a child under the age of 19 has unearned income above a specified amount, the excess is taxed at the parents' marginal tax rates for the year, rather than at the child's marginal rate. A) Both I and II B) I only C) II only D) Neither I nor II

ans a Both statements are correct. The kiddie tax applies to unearned income for children under the age of 19 and full-time students until they reach age 24.

Which of the following are items of tax adjustments for the individual alternative minimum tax? Medical expenses in excess of 7.5% of adjusted gross income Percentage depletion deduction in excess of adjusted basis Deduction for gambling losses to the extent of gambling income Tax-exempt interest on qualified private-activity bonds issued in 2011 A) II and IV B) I, III, and IV C) II, III, and IV D) I and III

ans a By definition, the percentage depletion in excess of adjusted basis and the tax-exempt interest on qualified private-activity bonds are preference items for purposes of the AMT. Remember that interest on private-activity municipal bonds issued in 2009 and 2010 is not a preference item for the AMT. The medical expenses in excess of 7.5% of adjusted gross income (for 2020) and the gambling losses to the extent of gambling income are both treated as allowable itemized deductions / adjustments for AMT purposes.

Which of the following statements concerning alimony is CORRECT? A) Payments to maintain property used by the payee spouse but owned by the payor spouse do not qualify as alimony. B) Payments made with respect to jointly owned property are considered full alimony. C) No payments except cash can be considered alimony. D) Cash payment of the payee spouse's mortgage made by the payor spouse as required by the divorce or separation instrument qualify as one-half alimony.

ans a Cash payment of the payee spouse's mortgage, rent, tuition, or tax liability made by the payor spouse as required by the divorce or separation instrument may qualify as alimony. Payments to maintain property used by the payee spouse, but owned by the payor spouse, do not qualify as alimony, even if required under the instrument. Payments made with respect to jointly owned property are considered one-half alimony. These property-related expenditures may include mortgage payments, real estate taxes, and homeowners insurance.

Claudia, who has an AGI of $40,000, wants to donate a portrait of an ancestor who served in the American Revolution to the museum in her town that houses a collection of Revolution Era items. Her basis in the portrait is $1,750, and it has a fair market value of $2,000. How much can she potentially deduct as a charitable contribution this year, assuming it is her only donation? A) $2,000 (It is related-use capital gain property.) B) $1,750 (It is related-use capital gain property, so she must use basis.) C) $600 (The museum is a 30% organization, so she must use FMV.) D) None (It is the portrait of a relative.)

ans a The portrait is related-use, capital gain property. Claudia may deduct an amount up to 50% of her AGI if she uses the basis of the painting and 30% of AGI if she uses FMV. As long as Claudia's AGI is greater than $6,667, she can deduct the FMV of the portrait.

Ron, age 43, and Sandy, age 41, are married with two children: Michael, age 12, and Victoria, age 8, who has been blind since her birth. Ron is an architect and general partner with XYZ partnership. Sandy is self-employed as an attorney and works out of a home office. Her home office is exclusively and regularly used for business, and the home office is her principal place of business. Their information for the tax year 2020 is as follows: Adjusted gross income: $217,300 Itemized deductions (including qualified residential mortgage interest, taxes paid, and charitable contributions): $33,000 Early in the current year, Sandy's father died. Sandy is the sole beneficiary of her father's entire estate. The estate is presently in the probate process. Sandy's mother, Lisa, age 68, has moved in with them but provides her own support. She was married to Sandy's father when he died earlier this year. This is Ron's second marriage. He makes monthly support payments to his former spouse and his daughter. Because both Ron and Sandy are considered to be self-employed, they make quarterly estimated tax payments each year to cover both their income tax and self-employment tax obligations. Ron's divorce decree specifies that the payment made to his former spouse is $300 per week until his former spouse dies, at which point payments will continue to be made to her estate until her daughter is age 18. Based on the information provided in the case scenario for Ron and Sandy, what amount per week is considered to be alimony? A) $0 B) $300 C) $150 D) $50

ans a If there is an obligation to continue to make payments after the death of the ex-spouse, the payments are not treated as alimony.

Edward had the following capital transactions during the current year: Long-term capital gain$3,200Long-term capital loss($3,600)Short-term capital gain$4,500Short-term capital loss($3,400) Which of the following describes the net capital gain or loss reportable by Edward for the current tax year? A) $700 net short-term capital gain B) $1,100 net short-term capital gain; $400 net long-term capital loss C) $900 net short-term capital gain; $200 net short-term capital loss D) $7,700 net capital gain; $7,000 net capital loss

ans a Long-term transactions are netted together, as are short-term transactions. The long-term capital loss is $400 ($3,600 - $3,200). The short-term capital gain is $1,100 ($4,500 - $3,400). The short-term capital gain is netted with the long-term capital loss ($1,100 - $400) to result in a net short-term capital gain of $700.

Your client Sally, age 30, is designing an educational investment program for her 8-year-old son. She expects to need the funds in about 10 years when her AGI will be approximately $70,000. She wants to invest at least part of the funds in tax-exempt securities. Which of the following investment(s) may yield tax-exempt interest on her federal return if the proceeds were used to finance her son's education? Treasury bills EE bonds GNMA funds Zero coupon Treasury bonds A) II only B) I, III, and IV C) III and IV D) II and III

ans a Proceeds from EE savings bonds may be exempt if the proceeds are used for qualified higher-education expenses of the taxpayer, spouse, or dependent. There is an AGI phaseout, which for 2020 is approximately $82,350‒$97,350 (2020) for a single taxpayer. (The actual phaseouts are provided on the exam.) All the other options generate currently taxable income. The Treasury bills and GNMA funds both produce taxable income on the federal return (Treasury bill interest would typically be tax exempt on her state return). The zero Treasury also produces taxable income each year as the amortized discount is added to taxable income, even though no cash income is received.

Which of the following is CORRECT with respect to the alternative minimum tax? A) Charitable contributions, investment interest expense, and casualty losses are allowable itemized deductions for AMT purposes. B) The exercise of an incentive stock option always causes an AMT preference item to be added into the AMT calculation. C) Net capital gains are subject to the AMT rates of 26% or 28%. D) All itemized deductions except home mortgage interest and property taxes are added back in the AMT computation.

ans a Qualified housing interest, charitable contributions, investment interest expense, casualty losses, and gambling losses (to the extent of gambling winnings) are among the allowable itemized deductions for AMT purposes. State and local income taxes and property taxes are not allowed for AMT purposes. Net capital gains (by definition—long-term capital gains) and qualified dividends are subject to the preferential rates (15% or 20%) even for AMT purposes. The preference item for the bargain element on the exercise of an ISO may be avoided by utilizing a disqualifying disposition of stock acquired with the ISO. A disqualifying disposition is a sale of the stock in the same year as exercise of the option. LO 3.3.2

Philip's grandmother gave securities to him when he was born nine years ago. In 2020, he has dividends of $15,000 from the securities. What is his net unearned income taxed at his parents' marginal tax rate? A) $12,800 B) $12,400 C) $9,300 D) $14,100

ans a Some of Philip's unearned income is taxed at his parents' marginal tax rate and is calculated as follows: $15,000 UI - $1,100 (standard deduction) - $1,100 (greater of $1,100 for 2020 or amount of allowable itemized deductions directly connected with the production of the unearned income) = $12,800

Harriet builds remote-control cars as a hobby. Which hobby expenses must she deduct from hobby income first? A) Cost of goods sold B) Operating expenses C) Cost recovery deductions D) None are deductible

ans a Starting in 2018, due to the passage of the Tax Cuts and Jobs Act (TCJA), most hobby expenses are no longer deductible with the notable exception of cost of goods sold.

As part of the property settlement after Lori and Gordon divorced, Gordon transferred ownership of a life insurance policy to Lori. Lori is the beneficiary of the policy, and Gordon is the insured. Which of the following statements regarding the property settlement is CORRECT? The transfer of the life insurance policy is subject to the transfer-for-value rule. The death proceeds of the policy will be income tax free at Gordon's death. A) II only B) I only C) Neither I nor II D) Both I and II

ans a Statement I is incorrect; the transfer-for-value rule does not apply when a life insurance policy is transferred from one spouse to the other under a property settlement incident to divorce. Statement II is correct.

Which of the following statements regarding charitable deductions by corporations is CORRECT? The corporate statutes of most states permit corporations to make charitable contributions, and the Tax Code permits a charitable deduction for contributions by a corporation. The charitable deduction is limited to a maximum of 20% of the corporation's adjusted taxable income. In the event the contribution is in excess of 20% of the corporation's adjusted taxable income, the balance can be carried forward for up to five years. A) I only B) Both I and II C) Neither I nor II D) II only

ans a Statement II is incorrect because the charitable deduction is limited to a maximum of 10% of the corporation's adjusted taxable income.

Glen and Debbie both have significant net worth and are currently in the highest marginal income tax bracket. They have developed a process that allows them to neutralize toxic chemical waste. They want to form a business that will protect their net worth in case the business fails or it becomes involved in lawsuits. They expect the business to produce significant profits immediately, as they have agreements in the works with several large chemical companies. Also, they would like to share ownership with other family members as they get closer to retirement. Which business form would be most appropriate for Glen and Debbie at this time? A) C corporation B) Limited partnership C) S corporation D) General partnership

ans a The S corporation (and the partnership entities) are conduits, and would cause the income to be added to their other significant individual income. The use of the C corporation would allow for protection from lawsuits or business failure. Also, the general partnership and the limited partnership both would cause a flow-through of the income, and the general partnership would not provide protection from personal liability. There is no indication that either party wants to be a general partner in a limited partnership. The use of a C corporation would be appropriate since it is a separate taxable entity, and the profits would not flow through. They could receive a reasonable salary, and the qualified dividends that are paid out would be subject to long-term capital gain rates.

Carol owns and operates a retail electronics store with annual sales of approximately $2 million. The store also specializes in repairing computers and other small electronics. Approximately 30% of her sales are with extended credit terms. What method of tax accounting may be used for Carol's business? The accrual method, because inventory is such a large component of the business The cash method, because it provides flexibility in the timing of income and expenses The hybrid method, because the business involves both inventory and service The installment sale method, to spread the gain over a longer time frame A) I, II, and III B) III only C) I only D) I, II, III, and IV

ans a The cash method may be used because average annual gross receipts are under $26 million for the prior tax years. (This limit was increased by the Tax Cuts and Jobs Act, or TCJA.) The accrual method may always be used. The hybrid method may be used because the business involves both inventory and service. The installment method may not be used, as dealers are not allowed to use the installment method.

Seven years ago, Karen Price purchased U.S. EE savings bonds for $5,000. During the current year, when Karen was 27 years old, she redeemed the bonds to help pay for her graduate school tuition. The accrued value at the time of redemption was $7,000. Assume Karen incurs $11,000 of tuition expenses in the year. What are the tax consequences upon the redemption of the bonds? A) All accrued interest is taxable in the current year. B) All the interest may be excluded. C) A portion of the interest may be excluded. D) The income on the bonds is generally subject to state income taxes.

ans a The exclusion for EE bond interest redeemed to pay for qualifying higher-education expenses applies only to bonds purchased by an individual age 24 or older, and held in that person's name, or jointly with a spouse. Karen is 27 years old; the bonds were purchased 7 years ago, when Karen was approximately 20. Because Karen does not qualify for the exclusion of the interest income because she was not age 24 or older at the time of purchase. All the interest is taxable in the year the bonds are redeemed. Remember that the interest of EE bonds is deferred until maturity, unless an election has been made to have the interest taxed each year as it accrues. Also, the interest income from EE bonds (and other federal government obligations) is generally not subject to state income tax.

Which of the following would result in higher taxation of one party with a premarital agreement? A) Transfer for consideration is made B) Transfer under the agreement is treated as a gift C) Transfer under the agreement is treated as an estate D) Transfer for premarital is made

ans a The income tax consequences of the premarital agreement depend in large part upon whether the transfer under the agreement is treated as a gift (where income tax is avoided) or as a transfer for consideration (which will probably result in the recognition of significant income by one party).

Bill and Rachel have three children: Mary, Ralph, and Don. In 2020 Mary, age 12, earns $4,000 in wages from a part-time job. Ralph, age 19 and not a full-time student, earns $3,500 in interest and short-term capital gains. Don, age 12, earns $2,350 in short-term capital gains and interest. Whose income is subject to the tax at the parents' marginal tax rate? A) Don B) Ralph C) Mary, Ralph, and Don D) Ralph and Don

ans a The parents' marginal tax rate applies to unearned income above $2,200 for children subject to the kiddie tax. Only Don has income that fits this criterion. Earned income is not subject to the kiddie tax (the application of the parental rate). Ralph is not a full-time student, so the kiddie tax stops applying in the year in which he turns 19. Remember that the kiddie tax applies to children who are under 19 years of age or who are under 24 if a full-time student. The kiddie tax does not apply to a child who is married and files a joint return for the tax year or if the child has earned income that exceeds half of his support. Also, the kiddie tax applies only where the child has at least one living parent. (A full-time student is an individual who is in school full-time for at least five calendar months during the tax year.) For an individual who is not a full-time student, the kiddie tax generally stops applying in the year in which the child turns 19. For a full-time student, the kiddie tax stops applying in the year in which the child turns 24.

Francine and Marshall have three children: Bill, Curt, and Rachel. For 2020 Bill, age 11, has $1,250 of interest income. Curt, age 13, has $2,950 of salary from a part-time job. Rachel, age 19, a part-time student for four months of the year, has $5,100 of dividends and capital gains. Whose income is subject to income tax at the parents' tax brackets? A) None of the children B) Bill and Rachel C) Bill D) Curt

ans a The parents' marginal tax rates apply to unearned income above $2,200 for 2020. Rachel has unearned income above $2,200. However, because Rachel is 19 and is not a full-time student, she is not subject to the kiddie tax rules. The kiddie tax stops applying in the year that the child turns 19, if the child is not a full-time student. The parents' tax rate does not apply to earned income, so Curt is not subject to the kiddie tax. The kiddie tax applies to children who are under 19 years of age, or who are under 24 if a full-time student. The kiddie tax does not apply to a child who is married and files a joint return for the tax year, or if the child has earned income that exceeds half of his support. Also, the kiddie tax applies only where the child has at least one living parent. (A full-time student is an individual who is a full-time student for at least five calendar months during the tax year.)

Which of the following statements regarding the use of life insurance inside a retirement plan is CORRECT? A) The premiums paid are a taxable benefit to the employee. B) The premiums paid are NOT a taxable benefit to the employee. C) The premiums paid are a taxable benefit to the employer. D) If the employee dies prematurely, the survivors will receive no benefits.

ans a The premiums paid are a taxable benefit to the employee. The main benefit to the employee is in the event of their premature demise, their survivors will still receive ample retirement benefits.

Your clients, Mike and Elizabeth, have a 17-year-old dependent son, Josh, who has a part-time job during the summer months. They have heard of the kiddie tax but are unsure of how it would impact the taxation of Josh's summer wages. Which of the following statements is CORRECT regarding the taxation of Josh's income for 2020? Josh has a limited standard deduction (up to $1,100) available. Josh has a full standard deduction (up to $12,400) available. Any income in excess of the available standard deduction and $1,100 is taxable at Mike and Elizabeth's marginal income tax rate. The income is not subject to the kiddie tax rules because it is earned income. A) II and IV B) I only C) I and III D) II and III

ans a The unearned income rules (the kiddie tax rules) do not apply to earned income. Thus, Josh may utilize up to the full standard deduction ($12,400 for 2020) against earned income.

Angela, age 16, is claimed as a dependent on her parents' income tax return. During 2020, she earned $2,200 from a summer job. She also earned $2,600 in interest and dividends from investments that were given to her by her great-aunt five years ago. How much of Angela's unearned income, if any, will be taxed to her in 2020 using her great-aunt's marginal tax rate of 35%? A) $0 B) $2,200 C) $2,600 D) $400

ans a When applying the kiddie tax, the parents' marginal tax rate is always used (regardless of the source of the property generating the unearned income). Therefore, none of the income is taxed to Angela using the great-aunt tax rate. The $400 of unearned income ($2,600 − $2,200) is taxed to Amy at her parents' marginal tax rate.

Which one of the following is a characteristic of a fixed annuity contract? A) If a corporation owns the annuity contract, the earnings are not tax deferred. B) The buyer may choose among a handful of investment options. C) Fixed annuity contracts are not tax advantaged, unlike other annuity contracts. D) The annuitant pays now for future fixed or variable payments.

ans a With a fixed annuity contract, there is no ability to select the investment options; the payments are fixed. Fixed annuity contracts are generally tax advantaged (tax deferred), unless a corporation owns the annuity contract, in which case the earnings are currently taxable. Such is also the case with a variable annuity.

A taxpayer holds a lease on property that she uses as a warehouse for her manufacturing business. The interest would be classified as A) personal use property. B) property used in a trade or business. C) property held as a real estate investment. D) property held for the production of income. Explanation The building is not personal use property. Because the building is used in her manufacturing business, it is property used in a trade or business. However, property held for the production of income generally involves a lower level of activity than does property used in a trade or business. As part of a business, this building would not be property held for the production of income.

ans b

All of the following statements regarding tax deduction limits on passive activity excess losses are correct except A) excess passive activity losses are fully allowed in the year in which the taxpayer disposes of his or her entire interest in the passive activity in a taxable transaction. B) losses from one nonpublicly traded passive activity may not offset income from another nonpublicly traded passive activity. C) excess passive activity losses are the excess of otherwise allowable deductions from the taxpayer's passive activities over the amount of income from the taxpayer's passive activities. D) excess passive activity losses are disallowed on the taxpayer's current tax return, but, instead, are deferred. Explanation The taxpayer's nonpublicly traded passive losses and nonpublicly traded passive income are aggregated for purposes of the limitation, so that losses from one nonpublicly traded passive activity may offset income from another nonpublicly traded passive activity.

ans b

Blake, a sole proprietor, is selling several business assets. He has been told by a friend that the items he is selling are not capital assets and are subject to the ordinary income tax rate. You are his financial planner and tell him that the gains on Section 1231 assets can be treated as capital gains for income tax purposes subject to certain rules. Which of the assets Blake sold are Section 1231 assets? A) Accounts receivable B) The building and land sold when Blake's business moved to a new location C) Blake's inventory of electric guitars his business manufactures D) A copyright on the theme song Blake's company uses in its advertising that Blake wrote Explanation The building and land sold when Blake's business moved to a new location qualify under Section 1231 as depreciable personal or real property used in business or for the production of income. The building portion of the property was depreciable property. While they are not considered capital assets, under Section 1231 they are taxed using capital gain rates, subject to the Section 1245 and 1250 rules for depreciation recapture rules. Losses are always ordinary and not subject to the $3,000 ($1,500 for MFS) ordinary loss limitation. Accounts receivable, inventory, and copyrights and other creative works held by the creator are all ordinary assets that would result in ordinary income tax (not capital gain) if sold at a gain.

ans b

Bobby owns a rental office building that he purchased in 2017 for $275,000. He placed the property in service later that year. Recently, Bobby incurred the cost of a new air conditioning system for $11,300, replacement of the roof for $21,000, other miscellaneous repairs for $2,500, conversion of unused space to rental space for $16,700, cleaning services of $1,100, lawn services for $2,300, and construction to make the building handicap accessible for $23,800. Based on these expenditures, how much will be added to the cost basis of his building (capitalized) and will be depreciated? A) $51,800 B) $72,800 C) $77,700 D) $28,000 Explanation Expenditures that materially extend the life of an asset or adapt it to a new use are considered improvements and are added to the cost basis of the asset and depreciated in accordance with the Internal Revenue Code. As a result, these expenditures include the new air conditioning system, replacement of the roof, costs to convert unused space to rental space, and the costs to make the building handicap accessible ($11,300 + $21,000 + $16,700 + $23,800 = $72,800).

ans b

Caleb earns a salary of $190,000. This year, he also received dividends and interest of $60,000. Caleb had previously invested $50,000 to purchase a 15% interest in a passive activity. Operations of the activity this year resulted in a loss of $400,000, of which Caleb's share is $60,000. How is Caleb's loss for the current year characterized for income tax purposes? A) $60,000 is suspended under the at-risk rules. B) $10,000 is suspended under the at-risk rules, and $50,000 is suspended under the passive activity loss rules. C) $60,000 is suspended under the passive activity loss rules. D) $50,000 is suspended under the at-risk rules, and $10,000 is suspended under the passive activity loss rules. Explanation Caleb invested $50,000 in the passive activity which becomes his at-risk amount. Because his share of the loss from the activity is $60,000, Caleb will be allowed to deduct only $50,000, which is his amount at risk. In addition, $10,000 of the loss ($60,000 total - $50,000 deductible under at-risk rules) has been suspended because of the at-risk rules and must be carried forward until Caleb either has $10,000 of income from the passive activity or invests $10,000 in the activity. Caleb has a $50,000 loss after applying the at-risk rules, but he is still not permitted a deduction for the loss because he has no passive income to offset the passive activity loss.

ans b

Carol owns and operates a retail appliance store with annual sales of approximately $12 million. The store has an extensive selection of appliances. Approximately 80% of her sales are with extended credit terms. What method of tax accounting is most appropriate for Carol's business? A) The accrual method, because inventory is such a large component of the business B) The cash method, because it provides flexibility in the timing of income and expenses C) The installment sale method, to spread the gain over a longer time frame D) The hybrid method, because the business involves both inventory and service Explanation The accrual method of accounting is often mandatory when inventory constitutes a significant income-producing factor. However, Carol's business qualifies for the small business exception, as average annual gross receipts are under $26 million. The hybrid method is incorrect because there is no indication that service constitutes a significant portion of the business. Also, the installment sale method is not available for sales of inventory or sales with revolving credit terms. The accrual method may always be used, but it lacks any flexibility.

ans b

During early 2020, Bob, an individual taxpayer, purchased a principal residence, taking out a mortgage of $600,000. In late 2020, he utilizes a home equity loan to borrow $100,000 to pay off credit card balances and an automobile note. Which of these is CORRECT with respect to the deductibility of the interest on the home equity loan? A) None of the interest is deductible because the interest on a home equity loan is never deductible. B) None of the interest is deductible because it is not considered acquisition debt. C) All of the interest is deductible, as the total mortgage debt is under $750,000. D) All of the interest is deductible because the home equity loan is $100,000 or less. Explanation None of the interest on the home equity loan is deductible. After 2017, only interest on acquisition debt is deductible. Acquisition debt is debt incurred to purchase, build, or renovate (remodel) the residence.

ans b

During the current tax year, Cassandra has a long-term capital loss of $22,000 from the sale of securities. She also has a long-term capital gain from the sale of a coin collection of $10,000 and has unrecaptured Section 1250 income of $18,000. Cassandra is in the 35% marginal income tax bracket. What is the tax result of her capital transactions? A) $18,000 unrecaptured Section 1250 income, taxed at 25%, and $12,000 long-term capital loss carryforward B) $6,000 unrecaptured Section 1250 income taxed at 25% C) $7,000 unrecaptured Section 1250 income taxed at 25%, $10,000 collectibles gain, and $19,000 long-term capital loss carryforward D) $6,000 collectibles gain taxed at 28% Explanation The long-term capital loss from the securities sale is netted first against the gain that would be taxed at the highest rate. After completely offsetting the collectibles gain (potential 28% rate), the remaining $12,000 is then netted against the unrecaptured Section 1250 income, leaving $6,000 unrecaptured Section 1250 income, to be taxed at 25%.

ans b

Francisco operates a sole proprietorship from his apartment. His gross income for the current tax year is $24,000. Business expenses not associated with his home office total $22,000. Expenses associated with the home office total $2,750. How much of the home office expense, if any, may Francisco deduct for the current year? A) $0 B) $2,000 C) $275 D) $2,750 Explanation The home office expense deduction is limited to the earned income from the business. In other words, the home office expense deduction, in general, can neither create nor add to a loss. The only expenses that may create or add to a loss are the allocated amounts of home mortgage interest and property taxes. In this situation, the $24,000 of gross income is reduced by the $22,000 of business expenses not associated with the home office, to leave $2,000 of earned income. Thus, of the $2,750 of home office expenses, only $2,000 would be deductible in the current year. Note that the remaining $750 of home office expenses would be subject to a carryforward.

ans b

Fred owns a package delivery business. During the current year, he purchased and placed into service $1,150,000 of equipment. Fred is in the highest marginal income tax bracket this year, and expects to be in that bracket for two more years. After that, he plans to semi-retire, but keep the business open for another three to five years. He expects to drop into the lower marginal brackets when he semi-retires. What advice would you give Fred regarding the use of various cost recovery methods? A) Forgo Section 179 and bonus depreciation and elect the straight-line method B) Utilize the bonus depreciation provision C) Use 50% bonus depreciation and elect the straight-line method D) Forgo Section 179 and bonus depreciation and use the MACRS table Explanation The fact pattern indicates that Fred is in the highest marginal bracket for three years, and then will be in the lowest marginal bracket after that. It makes sense to maximize the depreciation deduction this year when he is in the highest marginal bracket. By utilizing the bonus depreciation provision, the entire $1,150,000 may be deducted in the year of acquisition. There is no 50% bonus depreciation provision—a taxpayer may elect 100% bonus depreciation or forgo it entirely.

ans b

Which of the following may be allowed as a like-kind exchange? A) Farmland exchanged for farming equipment B) A parking lot exchanged for a shopping center C) A heifer exchanged for a bull D) An apartment building located in San Diego exchanged for an apartment building located in Acapulco, Mexico Explanation An exchange of U.S. realty for foreign realty is not considered like-kind. The like-kind requirements were changed under TCJA and now limits exchanges to realty for realty. The like-kind requirement does not mean that the property transferred must be identical to the property received; it merely requires that realty be exchanged for realty. LO 6.2.2

ans b

Fred runs a small business that has been in the family for over 50 years. The business has always used the cash basis method of accounting. Which of the following would be considered income to Fred last year? A bonus he declared for himself on December 31 of last year that is not payable until January 2 of the following year Dividends received on December 30 of last year Salary received during last year Interest on his money market account last year, but not posted until January 2 of the next year A) I, II, and III B) II, III, and IV C) I and II D) II and III Explanation For the cash method of accounting, all income actually received during the tax year is included in gross income (e.g., dividends, interest, wages). Income received constructively during the year, even though actual receipt is delayed, is includible for the tax year. Bonuses declared but not yet payable until the next tax year are not includible for the prior tax year because there was no constructive receipt.

ans b

Gil owns a portfolio of income-producing real estate. Gil retains ownership of the real estate but directs that the rental income be paid to his son, Kevin. The income is paid directly to Kevin, who reports it as part of his taxable income. Gil does not report the income on his tax return. With which one of the following potential tax traps should Gil be most concerned? A) Ownership attribution rules B) Assignment of income C) Constructive receipt D) Substance over form Explanation The fact that Gil retains ownership of the property and merely assigns the income to someone else is a potential tax trap for him. The assignment of income doctrine serves to tax the person who actually owns the property producing the income. The income cannot merely be assigned to another in order to generate tax advantages.

ans b

If an individual taxpayer files a Form 4868 on a timely basis in 2020, the 2019 federal income tax return is due (ignoring weekends or holidays), at the latest, by A) July 15, 2020 B) October 15, 2020 C) April 15, 2020 D) August 15, 2020 Explanation The Form 4868 provides an automatic six-month extension of time to file the Form 1040. The return would be due April 15 without regard to extensions. With the six-month extension, the due date is October 15.

ans b

Jack operates a cosmetic manufacturing business as an S corporation. During 2020, the business placed in service $33,000 of new property eligible for limited expensing under Section 179. If taxable income before cost recovery is $12,750, the maximum amount that the business can elect this year under Section 179 is A) $33,000. B) $12,750. C) $25,000. D) $8,000. Explanation The expensing election is limited to taxable income before the expensing election is calculated; therefore, $12,750 is the maximum Jack's business can elect under Section 179. The remaining amount of $12,250 is carried forward until the income of the business is sufficient to use up the remaining Section 179 expense election.

ans b

Jean's mother dies and leaves her house to Jean this year. The house is valued at $40,000. Jean rents the house to tenants for $1,000 per month following her mother's death. What amount must Jean include in her annual gross income for this year? A) The lesser of the FMV of the house or the amount of rental income she receives B) The amount of rental income she receives C) $40,000 D) $40,000 plus the amount of rental income she receives Explanation Jean receives the house on a tax-free basis because inheritances and gifts are not taxable to the recipient. However, any income generated by the house, such as rental income, is subject to taxation.

ans b

Jerry owns a dry-cleaning business. During the current year, Jerry purchased and placed into service $730,000 of equipment. He had taxable income of $745,000. Jerry is in the highest marginal income tax bracket this year, and expects to be in that bracket for two more years. After that, he plans to semi-retire, but keep the business open for another five years. He expects to drop into the lowest marginal bracket when he semi-retires. What advice would you give Jerry regarding the use of Section 179, bonus depreciation, and cost recovery deductions? A) Forgo Section 179 and bonus depreciation and use the Modified Accelerated Cost Recovery System (MACRS) table. B) Use the bonus depreciation provision. C) Forgo Section 179 and bonus depreciation and elect the straight-line method. D) Elect the maximum Section 179 and elect the straight-line method. Explanation The fact pattern indicates that Jerry is in the highest marginal bracket for three years, and then will be in the lowest marginal bracket after that. It makes sense to maximize the depreciation deduction this year when Jerry is in the highest marginal bracket. By using the bonus depreciation provision, the entire $730,000 may be deducted in the year of acquisition.

ans b

Jim has the following items from four investments: Passive income from a publicly traded limited partnership$16,000Passive loss from a publicly traded limited partnership$(8,500)Passive income from a nonpublicly traded limited partnership$13,000Passive loss from a nonpublicly traded limited partnership$(20,000) What is the total amount of passive losses that may be deducted during the current year? A) $29,000 B) $13,000 C) $20,000 D) $28,500 Explanation The passive loss from a nonpublicly traded limited partnership, $20,000, is deductible, but only up to the passive income from the same partnership; thus, only $13,000 of the $20,000 is currently deductible.

ans b

Jim purchased a limited partnership interest many years ago for $50,000. He has $26,000 of suspended losses from the partnership. His basis in the partnership was $15,000 and he recently sold his entire partnership interest for $20,000. Which one of the following statements correctly describes the proper tax treatment of the sale? A) There is a $5,000 gain resulting from the sale of the partnership interest, and $11,000 of suspended losses is deductible in full. B) There is a $5,000 gain resulting from the sale of the partnership interest, and the suspended losses are deductible in full. C) There is a $30,000 loss resulting from the sale of the partnership interest, and the suspended losses are deductible in full. D) There is a $30,000 loss resulting from the sale of the partnership interest, and the $11,000 of suspended losses are deductible in full. Explanation The gain or loss is the difference between the sale price and the adjusted basis of the partnership interest—$5,000 in this instance. Upon the taxable disposition (typically the sale) of the passive activity, all suspended losses are deductible in full against any other income.

ans b

Which of the following rules regarding the sale of Section 1231 property is CORRECT? When Section 1231 property is sold for more than the purchase price, the gain is afforded capital gain treatment and taxed using capital gain tax rates. When Section 1231 property is sold at a loss, the loss is treated as a capital loss. A) II only B) I only C) Neither I nor II D) Both I and II Explanation Statement II is incorrect. When Section 1231 property is sold at a loss, the loss is treated as an ordinary loss, not a capital loss.

ans b

Jimmy will have an adjusted gross income of $275,000 for the current tax year. He would like to reduce his tax liability without exposing himself to personal liability. Which of the following investments would be appropriate for Jimmy? An investment in active participation rental real estate An investment in a newly formed low-income housing limited partnership An investment in an oil and gas working interest An investment in a newly formed historic rehabilitation limited partnership A) II, III, and IV B) II only C) II and III D) III and IV Explanation Jimmy should invest in a newly formed low-income housing limited partnership. Active participation in real estate, historic rehabilition, and investing in oil and gas are subject to personal liability.

ans b

John has had a very good year and has over $600,000 of taxable income, including a sizable amount of capital gains. He's thinking of selling a large block of stock to a neighbor at a price significantly below market value solely to recognize the loss. If a court disallows the loss on the sale of the stock because the sale was not bona fide and was made for the sole purpose of realizing a loss, which doctrine is being applied? A) Step transaction doctrine B) Sham transaction doctrine C) Assignment of income doctrine D) Clear reflection of income doctrine Explanation A sale that is not bona fide and made for the sole purpose of realizing a loss from the transaction would be considered a sham transaction.

ans b

Lowell and Thelma are married and will file a joint return for the current tax year. They are contributing to their respective 401(k) plans through their employers. They have provided you with the following information. Lowell's salary (after 401(k) contributions)$75,000Thelma's salary (after 401(k) contributions)$50,000Alimony payments to Lowell's ex-wife$24,000Net long-term capital loss$7,000Property taxes$2,000IRA contribution—Lowell$6,000IRA contribution—Thelma$6,000 Lowell's divorce was finalized in 2015. Based on the information given, what is the couple's adjusted gross income for the current tax year? A) $118,000 B) $86,000 C) $82,000 D) $98,000 Explanation The salaries of $125,000 reduced by the $24,000 of alimony payments equals $101,000. This is further reduced by $3,000 of net capital losses. Remember that only $3,000 of net capital losses are deductible in a given year, with an indefinite carryforward of the excess. The $12,000 of IRA contributions is also deductible. Even though both spouses are active participants in company-maintained retirement plans, their MAGI (AGI without the IRA contributions) is only $98,000. This is under $104,000 (for 2020)—the beginning of the phaseout range for married couples filing jointly, when both spouses are active participants. The property taxes are an itemized deduction, and do not affect the AGI.

ans b

Paul and Mary form an equal partnership to produce hammers for the military. They each have a 50% profit and loss sharing agreement. Paul contributes cash of $50,000 and property with a fair market value of $50,000 and an adjusted basis of $20,000. Mary contributes cash of $25,000 and property with a fair market value of $75,000 and an adjusted basis of $40,000. A bank gives the partnership a recourse loan of $30,000 and a nonrecourse loan of $25,000. What is the amount of Paul and Mary's basis in the partnership, respectively? A) $70,000 for Paul, $65,000 for Mary B) $97,500 for Paul, $92,500 for Mary C) $85,000 for Paul, $80,000 for Mary D) $127,500 for Paul, $127,500 for Mary Explanation The basis of a partnership interest is measured by the amount of cash contributed, increased by the adjusted basis of property contributed, and further increased by the partner's share of both recourse and nonrecourse financing.

ans b

Phillip's personal automobile was almost destroyed in an accident. The insurance company paid $6,000 on the claim. The auto's fair market value before the accident was $16,000, and the value after the accident was $1,000. His basis in the automobile was $12,000. Phillip's AGI is $42,500. What is the amount of Phillip's deductible casualty loss? A) $1,750 B) $0 C) $1,650 D) $6,000 Explanation As a result of the Tax Cuts and Jobs Act (TCJA), casualty losses are only allowed for damages sustained within a federally declared disaster area. Thus, there is no deduction for this loss. If the loss had been incurred in a federally declared disaster area (as a result of the disaster), the deductible casualty loss computation would begin with the lesser of the decrease in fair market value ($15,000 decrease in FMV) or the adjusted basis in the property. In this situation, the adjusted basis of $12,000 must be reduced by a $100 floor, the insurance of $6,000, and further reduced by 10% of the adjusted gross income. Thus, $12,000 reduced by $100, $6,000 insurance, and further reduced by $4,250, equals $1,650.

ans b

Rashida and Caroline both have significant net worth and are currently in the highest marginal income tax bracket. They have developed a process that allows them to neutralize toxic chemical waste. They want to form a business that will protect their net worth in case the business fails or it becomes involved in lawsuits, but they also would like to be able to offset income from other sources with the start-up losses. Furthermore, they want to share ownership with other family members if the business is successful. Which business form would be most appropriate for Rashida and Caroline at this time? A) Limited partnership B) S corporation C) General partnership D) C corporation Explanation The S corporation will allow the potential losses to flow through to offset other individual income. Also, the use of the S corporation would allow for protection from lawsuits or business failure. Although the general partnership and the limited partnership would both allow for the flow-through of losses, the general partnership would not provide protection from personal liability. There is no indication that either party wants to be a general partner in a limited partnership, so general partnership is not correct. The use of a C corporation would not be appropriate because it is a separate taxable entity, and start-up losses would not flow through.

ans b

Which of these is CORRECT regarding the qualified education interest deduction? A) The maximum deduction of $2,500 may be taken by a married taxpayer filing jointly with $200,000 of MAGI. B) The deduction may only be claimed by the taxpayer legally obligated to make the loan payments. C) An individual who is eligible to be claimed as a dependent may take the deduction. D) The deduction may only be taken as an itemized deduction. Explanation The deduction may only be claimed by the taxpayer legally obligated to make the loan payments. If the loan is in the student's name, the parents may not claim the deduction, even if they make the payments. The deduction is taken as an adjustment to income. There is a $140,000 to $170,000 AGI phaseout for a married couple filing jointly. An individual who is eligible to be claimed as a dependent may not take the deduction.

ans b

Reese has the following items: Prior-year passive loss carryforward amounts: ($5,000) from XYZ limited partnership (publicly traded) ($8,000) from ABC limited partnership (nonpublicly traded) Current-year passive income and loss amounts: $2,000 from XYZ limited partnership (publicly traded) ($3,000) from GHI limited partnership (publicly traded) $12,000 from JKL limited partnership (nonpublicly traded) ($14,000) from RST limited partnership (nonpublicly traded) What is the total amount of passive losses that may be deducted during the current year? A) $16,000 B) $14,000 C) $2,000 D) $12,000 Explanation The general rule is that passive losses may only be deducted against passive income. The rules for publicly-traded partnerships are more restrictive—the passive income from a PTP may not be offset by passive losses arising from any other source. In addition, the passive losses from PTPs are not currently deductible—they may only be used to offset future income from that same activity. Of the $5,000 passive loss carryforward from the XYZ limited partnership, only $2,000 may be used in the current year due to the $2,000 of current-year passive income from that same PTP. A total of $12,000 in losses from the RST limited partnership may be used against the $12,000 of income from the JKL limited partnership in the current year because both are nonpublicly traded. Thus, the total of passive losses allowed for the current year is $14,000.

ans b

Ron, age 43, and Sandy, age 41, are married with two children, Michael, age 12, and Victoria, age 8, who has been blind since her birth. Ron is an architect and general partner with XYZ partnership. Sandy is self-employed as an attorney and works out of a home office. Her home office is exclusively and regularly used for business and is her principal place of business. Their information for the tax year 2020 is as follows: Adjusted gross income: $217,300 Itemized deductions (including qualified residential mortgage interest,taxes paid, and charitable contributions): $33,000 Early in the current year, Sandy's father died. Sandy is the sole beneficiary of her father's entire estate. The estate is presently in the probate process. Sandy's mother, Lisa, age 68, has moved in with them but provides her own support. She was married to Sandy's father when he died earlier this year. This is Ron's second marriage. He makes monthly support payments to his former wife and his daughter. Because both Ron and Sandy are considered to be self-employed, they make quarterly estimated tax payments each year to cover both their income tax and self-employment tax obligations. Based on the information provided in the case scenario for Ron and Sandy, which of the following statements regarding Ron's status as a partner with XYZ partnership is CORRECT? Ron has the right to participate in the management and operation of the business. Ron has no personal liability for the acts of the partnership or the other partners. A) II only B) I only C) Both I and II D) Neither I nor II Explanation Statement I is correct. As a general partner, Ron has the right to participate in the management and operation of the business. Statement II is incorrect. As a general partner, Ron has unlimited personal liability for the acts of the partnership and other partners.

ans b

This year, Hugh sold a classic automobile to his friend, Doug, on the following terms: The price was $40,000, equal to the fair market value. Hugh's basis in the automobile was $25,000. Starting this year, Doug will pay in five annual installments of $7,000 plus accrued interest. Doug will make a $5,000 down payment. Ignoring interest income, what amount of gain will Hugh recognize for the current year? A) $7,500 B) $4,500 C) $5,000 D) $8,571 Explanation Hugh's gross profit percentage is 37.5%. This is multiplied by the $12,000 of payments received during the year. The profit on the sale was $15,000 divided by the $40,000 contract price, which equals a 37.5% gross profit percentage.

ans b

This year, Irwin sold several securities that left him with the following types of gains and losses: long-term capital gain—$18,000, short-term capital gain—$11,800, long-term capital loss—$12,200, and short-term capital loss—$12,000. What is the net capital gain or loss on Irwin's security sales? A) Net long-term gain of $5,600 and net short-term gain of $80 B) Net long-term gain of $5,600 C) Net long-term loss of $24,200 D) Net long-term gain of $5,800 and net short-term loss of $200 Explanation The long-term gain and loss are netted, leaving a long-term gain of $5,800. Short-term gains and losses are netted, leaving a short-term loss of $200. Because there is a positive and a negative, these are netted to leave a net long-term capital gain of $5,600.

ans b

What is the amount and character of the penalty for an employer who fails to withhold Social Security and federal income taxes from employee paychecks? A) 50%, but the responsible party needs only to pay back 50% of what is owed with no additional penalty B) 100%, but the responsible party needs only to pay back what is owed with no additional penalty C) 100%, and the responsible party needs to pay back what is owed plus a 100% penalty D) 50%, and the responsible party needs to pay back what is owed plus a 50% penalty Explanation Employers are required to withhold amounts from an employee's paycheck for Social Security taxes and federal income taxes. If the employer fails to do so or fails to pay such amounts to the IRS, they or any other responsible person will be subject to the 100% penalty, which is simply having to pay 100% of the amount they should have collected, accounted for, and paid; they are not subject to any additional penalty.

ans b

What percentage of current-year estimated tax payments and withholding must be paid by a married taxpayer to avoid the imposition of a penalty for underpayment of estimated tax? (Assume that the taxpayer had AGI in the prior year of $110,000.) A) 110% B) 90% C) 100% D) 80% Explanation The current-year exception for avoiding estimated tax penalties is 90% of the current year's tax. The other exceptions, 100% of the prior year's tax and 110% of the prior year's tax (for taxpayers with a prior year AGI greater than $150,000), are each based on prior liability rather than current liability. Note that the 110% requirement for 2020 estimated payments applies only if the 2019 AGI exceeds $150,000.

ans b

Which of the following activities are considered passive activities? A real estate broker spending 1,500 hours on her real estate activities, this being 80% of her personal services for the year A taxpayer who owns an apartment building using a property management to assist him in managing the property A taxpayer investing in a real estate limited partnership (RELP) A taxpayer owning 8% of an inherited condo that is rented to others during the year A) I, II, III, and IV B) II, III, and IV C) II and III D) I only Explanation Statements II, III, and IV are correct. Only the real estate professional's activities are not considered passive. In statement IV, the taxpayer owns less than 10% of the activity, so it is considered passive. This taxpayer is barred from using the real estate exception for losses because the taxpayer owns less than 10% of the activity.

ans b

Which one of the following is a tax return preparer failure that would initiate a tax penalty from the IRS? A) Failure to maintain a list of all returns prepared for the past five years B) Failure to sign a return as preparer and give their tax identification number on the return C) Failure to provide a taxpayer with a receipt for their services D) Failure to keep a copy of all returns prepared for at least the last 10 years Explanation The following actions would constitute a tax return preparer's failure subject to a tax penalty: failure to provide a taxpayer with a copy of their return, failure to keep a copy of all returns prepared for at least the last three years or to maintain a list of returns prepared, and failure to sign a return as preparer and give their tax identification number on the return.

ans b

Which one of the following is advice from the National Office of the IRS requests that can give both the taxpayer and a revenue agent an opportunity to resolve a dispute? A) Notice B) Technical Advice Memoranda C) Private Letter Rulings D) Announcement Explanation Technical Advice Memoranda normally take place during an audit or during the appeals process of the audit; they give both the taxpayer and the revenue agent an opportunity to resolve a dispute over a technical question. Private Letter Rulings are taxpayer guidance from the IRS that apply only to the particular taxpayer(s) asking for the ruling; they are not applicable to all taxpayers. A Notice is a public pronouncement that contains official guidance about regulations or interpretations of the Code. Announcements often summarize code sections in layman's terms, or notify taxpayers of impending deadlines; they have only short-term value.

ans b

Which one of the following statements correctly describes the income tax consequences upon the disposition of a passive activity? A) The suspended losses are deductible in full if more than 50% of the interest in the activity is sold. B) The suspended losses are deductible against any income upon the sale of the entire interest. C) If a taxpayer sells 20% of the interest in an activity, then 20% of the suspended losses are deductible against any other income. D) The suspended losses are nondeductible. Explanation The suspended losses are deductible against any income upon the sale of the entire interest.

ans b

Which one of the following statements is CORRECT with respect to a second, or vacation, home? A) If the residence test is not met, then any rental income is taxable, but the expenses attributable to the rental are nondeductible. B) If the home is rented for fewer than 15 days, the rental income is not taxable. C) If the residence test is met, the home mortgage interest is not deductible. D) If the home is rented for fewer than 15 days, the rental income is taxable, and the rental expenses are deductible. Explanation If property is rented fewer than 15 days per year, the taxpayer can exclude rental income from gross income, but expenses are nondeductible, except mortgage interest, taxes, and casualty losses (Schedule A).

ans b

William and his wife, Lisa, recently divorced. Under the terms of the divorce decree, William is required to pay Lisa $2,000 per month for five consecutive years. The divorce was finalized on December 15, 2018, with the first alimony payment to be made on January 15, 2019, and on the 15th of each month thereafter. There is no provision in the decree (or under state law) that payments terminate upon the death of the payee spouse, Lisa. How much of each payment, if any, is deductible, and why? A) $0, because the excess front-loading rules are violated B) $0, because the payments do not terminate at death C) $1,000, because of the limitation applied by the front-loading rules D) $2,000, because the payments are qualifying alimony Explanation Payments that do not terminate upon the death of the payee spouse are not considered qualifying alimony. This applies to all payments, even those made prior to death. Remember that, for divorces finalized prior to 2019, qualifying alimony is still deductible by the payor and taxable to the payee.

ans b

Your client, Yolanda, has active income of $300,000 per year and substantial unused passive losses from a nonpublicly traded limited partnership. She would like to find an investment that would allow her to utilize her passive losses. Which of these is the most appropriate investment for Yolanda? A) A master limited partnership (MLP) generating income B) A nonpublicly traded limited partnership generating passive income C) A publicly traded limited partnership generating income D) A portfolio asset that does not generate income but will appreciate in value over time Explanation Yolanda needs a passive income generator. Only the nonpublicly traded limited partnership would qualify. The MLP and the publicly traded partnership would not qualify, as income from a publicly traded partnership cannot be offset by passive losses arising from any other source. LO 7.2.2

ans b

In 2020, Lonnie, age 17, is claimed as a dependent on his parents' income tax return and has $3,200 of interest and short-term capital gain income from an UTMA account that was established many years ago. Lonnie's parents, who file jointly, have taxable income of $130,000 (22% MFJ tax bracket). What is Lonnie's income tax liability for 2020? A) $505 B) $330 C) $360 D) $565

ans b $3,200unearned income($1,100)standard deduction($1,100)taxed at child's rate of 10%: $1,100 × 10% =$110$1,000taxed at parents' top marginal tax rate of 22%: $1,000 × 22% =$220Total$330 The first $1,100 of unearned income is sheltered by the child's limited standard deduction. The next $1,100 is taxed to the child, at the child's marginal income tax bracket. All unearned income in excess of $2,200 (for 2020) is taxed to the child at the parents' top marginal tax rate. The parents' tax rate is 22% up to $171,050 of taxable income (rate schedule provided in textbook or downloaded from this online course dashboard). The kiddie tax applies to children who are under 19 years of age or who are under 24 if a full-time student. (A full-time student is an individual who is a full-time student for at least five calendar months during the tax year.) The kiddie tax does not apply to a child who is married and files a joint return for the tax year, or if the child has earned income that exceeds half of his support. Also, the kiddie tax applies only where the child has at least one living parent. LO 3.2.1

Which of the following statements regarding installment payments pursuant to a divorce is CORRECT? It is usually in the best interest of the payor to spread the amount of payments over as many years as possible to take advantage of the time value of money. It is usually in the payee's best interest to immediately receive as much money as possible rather than receiving payments over a longer period. A) Neither I nor II B) Both I and II C) II only D) I only

ans b A common issue that arises in the settlement of divorce proceedings is how payments are best structured in resolution of marital obligations. As a general rule, it is usually in the best interest of the payor to spread the amount of payments over as many years as possible to take advantage of the time value of money. Alternately, the payee's best interests are served by immediately receiving as much money as possible rather than receiving payments over a longer period.

Ron, age 43, and Sandy, age 41, are married with two children: Michael, age 12, and Victoria, age 8, who has been blind since her birth. Ron is an architect and general partner with XYZ partnership. Sandy is self-employed as an attorney and works out of a home office. Her home office is exclusively and regularly used for business, and the home office is her principal place of business. Their information for the tax year 2020 is as follows: Adjusted gross income: $217,300 Itemized deductions (including qualified residential mortgage interest, taxes paid, and charitable contributions): $33,000 Early in the current year, Sandy's father died. Sandy is the sole beneficiary of her father's entire estate. The estate is presently in the probate process. Sandy's mother, Lisa, age 68, has moved in with them but provides her own support. She was married to Sandy's father when he died earlier this year. This is Ron's second marriage. He makes monthly support payments to his former spouse and his daughter. Because both Ron and Sandy are considered to be self-employed, they make quarterly estimated tax payments each year to cover both their income tax and self-employment tax obligations. Based on the information provided in the case scenario, which of the following statements regarding Lisa's income tax filing status for 2020 is CORRECT?

ans b A) Lisa must file married filing separately for 2020. B) Lisa may file married filing jointly for 2020. C) Lisa may file as head of household for 2020. D) Lisa must file a single return for 2020. Explanation Because Lisa's spouse died earlier in the year, she may use married filing jointly status for 2020. She cannot use the head of household filing status because she does not maintain a household for a qualifying child or relative.

Which of the following statements regarding charitable contributions is CORRECT? The amount deductible in a tax year is limited to a percentage of the individual taxpayer's adjusted gross income (AGI). Any amount in excess of the percentage of AGI limitation for the tax year may be carried back for two years and forward for five years. The percentage of AGI limitation depends solely on the type of organization to which the donation is made. A limit is imposed after applying the percentage of AGI limitation for the amount of a charitable deduction allowed for gifts of appreciated property. A) II and IV B) I only C) I and IV D) II and III

ans b Amounts in excess of the percentage of AGI limitation may be carried forward for only five years. The percentage of AGI limitation depends on both the type of organization to which the donation is made and the type of property donated. The limit applicable to the amount of a charitable deduction allowed for gifts of appreciated property is applied before applying the percentage of AGI limitation.

Which of the following businesses may be considered a specified service business (SSB) for purposes of the qualified business income (QBI) deduction? A) An engineering firm B) A medical practice C) A furniture manufacturer/wholesaler D) An automobile repair shop

ans b An SSB is any trade or business involving the performance of services in health, law, accounting, actuarial science, performing arts, consulting, athletics, financial services, brokerage services, or any trade or business where the principal asset of such trade or business is the reputation or skill of one or more of its employees; or, any trade or business that involves the performance of services that consist of investing and investment management, trading, or dealing in securities, partnership interests, or commodities. Thus, the medical practice would be an SSB, but none of the other businesses would. Engineering and architecture are specifically excluded from the definition of SSB.

Trudy is a single taxpayer. Her Aunt Diane has come to live with her after it was determined she could no longer live independently. She is physically challenged and needs full-time care. Diane's deceased parents set up a trust for her that supplies all of her support, including paying for in-home care and all medical bills. Trudy wants to know how this will affect her own income tax situation. What does the planner tell her? A) Diane is Trudy's qualifying relative. B) Trudy must file as single. C) Trudy's filing status changes to head of household. D) Trudy may list Diane as a dependent on her tax return.

ans b Because the trust pays for all of Diane's support, Trudy's income tax return is unaffected and she will file as single.

Which of the following is a permissible method for determining the business part of a residence for purposes of the home office expense deduction? A) Relative use method of allocation B) Number of rooms in the house used for business compared to total rooms in the house C) Amount of utility use compared to that of the whole house D) Accrual method of allocation

ans b Both the relative square footage of the home office area and the relative number of rooms may be used in determining the business part of a home. The amount of utility use can be a deduction, but is not used in determining the business part of a home. There is no recognized method known as the relative use method of allocation.

The Taylors are concerned they may be subject to an alternative minimum tax liability for last year. Which of the following is NOT an adjustment added back to regular taxable income for calculating alternative minimum taxable income (AMTI)? A) Exclusion of gain from Section 1202 qualified small business stock (QSBS) purchased in August 2009 B) Charitable contributions C) Itemized deduction D) Standard deduction

ans b Charitable contributions are not added back to regular taxable income when determining AMTI.

Lois, age 29, and Clark, age 31, recently divorced. Lois is taking custody of their three children ages four, two, and one. Clark will pay alimony and child support of $1,000 per month. They live in a state that requires alimony payors to obtain a life insurance policy. Clark obtained a group term life policy for $50,000 through his employer to meet this obligation. Clark's lawyer claims this satisfies his legal obligation. Which of the following should a financial planner recommend to Lois? A) She should inquire if she can take loans against the cash value for future college tuition payments. B) She should demand an increase in coverage and request a permanent policy. C) She should demand his employer increase the coverage. D) She should accept the settlement as adequate.

ans b Clark is underinsured given his legal obligations. Lawyers are not financial planners and generally are not trained to calculate life insurance coverage needs. Consequently, they often settle for an amount of life insurance that is too low. Term policies do not have cash value. If Clark leaves his employer, the term policy will most likely be canceled, which would compel Lois to take Clark back to court.

Irwin, a young financial planner, operates his practice as an LLC. His LLC net income is $100,000, and the taxable income on his single return is $140,000. The LLC paid no wages, and has no depreciable property. What is the amount of Irwin's qualified business income (QBI) deduction? A) $0, because he has no depreciable property B) $20,000 C) $0, because he paid no wages D) $28,000

ans b For a specified service business (SSB), if the taxpayer's taxable income is below the phaseout range of $163,300 to $213,300 (single 2020), the QBI deduction is fully available. An SSB involves performance of services in the fields of health, law, accounting, actuarial science, performing arts, veterinary services, consulting, athletics, financial services, or brokerage services. For pass-through entities, the owner/taxpayer may be allowed a deduction equal to 20% of the lesser of the QBI, or 20% of the taxpayer's taxable income (reduced by net capital gain). Thus, the available deduction is 20% of the QBI of $100,000, or $20,000.

Hal and Jody created an irrevocable trust for the benefit of their dependent children. They named their attorney as trustee of the trust and authorized him to invest in stocks, bonds, and certificates of deposit. Included in the investment authority is the right to use trust income to purchase insurance on Hal and Jody's lives. All funds are currently invested in high-yielding bonds paying 4% semiannual interest on a par value of $50,000. Which taxpayer must pay tax on the income of the trust? A) The attorney, because of his broad authority as trustee B) Hal and Jody, because the income is (or may be) used to purchase insurance on their lives C) The children, because they are the designated beneficiaries D) The trust, because it is irrevocable with no benefits to the grantor

ans b If the trust income is, or may be, used to purchase insurance on the life of the grantor or the grantor's spouse, then the trust is a grantor trust.

Sam has had persistent car trouble and decided to buy a new truck to make certain he gets to work on time. He received an interest-free loan of $5,000 for the needed down payment from his employer this year on January 1. The federal interest rate was 4% this year. Sam's only income this year was his salary of $40,000 and interest income of $75. Which of the following statements regarding this loan is CORRECT? A) Because this is a compensation-related loan, it results in compensation expense for Sam's employer and compensation income for Sam of $200. B) Sam has no compensation income this year from interest imputed on the loan. C) Sam's employer must recognize $75 of interest income this year because of the loan to Sam. D) Sam has nondeductible interest expense of $200.

ans b In a compensation-related below-market loan of $10,000 or less, no interest is imputed and no compensation results.

Charley lent his friend, Richard, $17,000 for a down payment on a home in a no-interest loan early in the current year. Charley had investment income of $750, and Richard had investment income of $1,200 in the same year. The federal interest rate is 3.5%. Richard has been making payments each month. What recommendations do you make for accounting for the loan made to Richard by Charley? A) Because this is a gift loan greater than $10,000 but less than or equal to $100,000, no interest will be imputed to the loan. B) Imputed interest is calculated on the loan to Richard and is considered a gift to Richard from Charley. C) Charley must develop an amortization schedule using the federal rate of 3.5% to account for Richard's payments of principal and interest. D) Because Charley's investment income is less than $1,000 this year, no interest is imputed to the loan.

ans b In a gift loan, the amount of the imputed interest constitutes a gift from the lender to the borrower. For gift loans greater than $10,000 and less than or equal to $100,000, no interest is imputed if the borrower's investment income for the year does not exceed $1,000. For a gift loan of more than $100,000, the prevailing federal rate of interest will be imputed. For this loan, Richard's investment income exceeds $1,000 and interest will be imputed.

In the current year, Jeff makes the following charitable donations: BasisFMV Inventory used in Jeff's business (sole proprietor):$8,000, $6,000 Stock in ABC Co. (acquired 2 years ago):$10,000, $40,000 Personal coin collection (acquired 10 years ago):$1,000$7,000 The ABC stock was given to Jeff's church, and the coin collection was given to the Boy Scouts of America. Both donees promptly sold the property for the stated FMV. Ignoring percentage limitations on AGI, Jeff's maximum charitable contribution valuation for deduction purposes available for the current year is A) $55,000. B) $47,000. C) $19,000. D) $53,000.

ans b Jeff's maximum valuation for deduction purposes available for the current year is $47,000 as follows: Inventory: $6,000 (as ordinary income property, limited to lesser of basis or FMV) Stock in ABC Co.: $40,000 (the maximum possible deduction is FMV for this long-term capital gain property) Personal coin collection: $1,000 (tangible personal property depends on dedicated use of property from standpoint of the charity; this is use-unrelated property because it was given to the Boy Scouts also limited to the lesser of basis or FMV)

Nancy is a single taxpayer and 67 years old. She has the following itemized deductions: Home mortgage interest (first mortgage)$15,950State income taxes$3,120Property taxes$1,480Charitable contributions$2,000Gambling losses$1,500Unreimbursed employee business expenses$4,600Tax return preparation fee$400Medical expenses$18,980 Nancy's AGI for 2020 is $250,000. Included in the AGI is $500 of gambling winnings. What amount of Nancy's itemized deductions would be allowed for purposes of the AMT? A) $34,950 B) $18,680 C) $17,950 D) $18,950

ans b Of the itemized deductions listed, only the qualifying home mortgage interest of $15,950, the charitable contributions of $2,000, and the gambling losses to the extent of winnings of $500, and the $230 of medical expenses are allowable for purposes of the AMT. The medical expenses are deductible to the extent they exceed 7.5% of AGI for both regular and AMT purposes.

In 2020, Glen, a single taxpayer, has the following itemized deductions: Home mortgage interest (first mortgage)$7,950Home mortgage interest (home equity loan)$4,350State income taxes$4,120Property taxes$1,880Charitable contributions$2,600Gambling losses (to extent of winnings)$1,200 The home equity loan was incurred to purchase what Glen describes as a "midlife crisis" sports car. What amount of itemized deductions, if any, would be allowed for purposes of the AMT? A) $7,950 B) $11,750 C) $17,750 D) $16,100

ans b Of the itemized deductions listed, only the qualifying home mortgage interest of $7,950, the charitable contributions of $2,600, and the gambling losses to the extent of winnings of $1,200 are allowable for purposes of the alternative minimum tax. For both regular income tax and AMT purposes, the interest on the home equity loan is only deductible if used for acquisition or renovation of the principal residence and/or one other residence. The taxes are not deductible for AMT purposes

Dan and his spouse, Gina, were divorced almost four years ago. Under the terms of the decree, Dan must pay Gina 10% of his net business income each year for five consecutive years. Dan made alimony payments of $60,000 in his first post-separation year, $35,000 in the second year, and $5,000 in the third year. What is the tax result of the alimony payments made to Gina? A) None of the payments are deductible because the excess front-loading rules are violated. B) All of the alimony payments are deductible because the front-loading rules do not apply to amounts tied to business profits. C) $15,000 of the payments is deductible because of the limit imposed by the front-loading rules. D) $47,500 of the payments must be recaptured due to the front-loading rules.

ans b Payments that fluctuate because of a continuing liability to pay a fixed portion of income from business, property, or services are not subject to the excess front-loading rules.

Which of the following itemized deductions would be adjustments to regular taxable income in arriving at alternative minimum taxable income (AMTI)? Casualty losses State income taxes paid Standard deduction Charitable donation made to the local university A) II, III, and IV B) II and III C) I and III D) I and IV

ans b Statement I is incorrect because casualty losses are deductible for both regular tax and AMT; no adjustment is necessary. Statement II is correct because state taxes are not deductible for AMT purposes. Statement III is correct because the standard deduction is a positive adjustment when calculating AMT income. Statement IV is incorrect because charitable contributions are deductible for both regular income tax and AMT; no adjustment is necessary.

Eight years ago, Joan Allen, a married taxpayer filing jointly, purchased U.S. Series EE savings bonds for $6,000. She titled the bonds jointly with her husband, Hank. During the current year, when Joan was 35 years old, they redeemed the bonds to help pay for Joan's graduate school tuition. The accrued value at the time of redemption was $8,000. Their AGI for 2020 is estimated to be $100,000. Assume Joan incurs $8,000 of tuition expenses during the year. What are the tax consequences upon the redemption of the bonds? A) The interest is taxable at both state and federal levels. B) All the interest may be excluded. C) A portion of the interest may be excluded. D) All accrued interest is taxable in the current year.

ans b The EE bond exclusion (for educational purposes) is phased out (for married couples filing jointly) between $123,550 and $153,550 of AGI in 2020. There is no exclusion available when AGI exceeds $153,550. It is not necessary to memorize the exact phaseout amounts because they will be provided on the exam. To qualify for the exclusion, the bonds must be purchased by an individual age 24 or older and held in that person's name, or jointly with a spouse. EE bonds are not taxable at the state level.

Walter anticipates his adjusted gross income to be $200,000 for the current tax year. He has not made any charitable contributions during the year, but now wants to give his church a block of stock with a fair market value of $110,000. Walter paid $50,000 for the stock six years ago. What is the maximum allowable charitable deduction Walter may receive during the current tax year if he makes a contribution of the stock? A) $33,000 B) $60,000 C) $110,000 D) $50,000

ans b The answer is $60,000. The allowable deduction for a contribution of long-term capital gain property, such as stock to a public charity, is limited to 30% of AGI. The remainder ($110,000 - $60,000 = $50,000) may be carried forward up to 5 years.

Ron has the following income for the current tax year: Schedule C net income: $12,000 Flow-through of general partnership operating income: $15,000 Wages: $80,000 Net income from rental of real estate: $10,000 What is the amount of Ron's self-employment tax? A) $1,696 B) $3,815 C) $9,467 D) $5,228

ans b The items included in the computation of self-employment income are the Schedule C income and the flow-through of general partnership operating income. The total of these items is $27,000. This is then reduced by 7.65% of this amount, and then multiplied by 15.3% to give us $3,815. Remember, the 7.65% subtraction always applies. Because the total of the self-employment income and the wages received are less than the wage base, no special computation is necessary.

In 2020, which of the following is a method in calculating total self-employment tax, where net income from self-employment is above the taxable wage base? Calculate self-employment income; multiply by 0.9235 (1 − 0.0765); subtract the taxable wage base, and from that, multiply the excess over the taxable wage base by 2.9% (Medicare portion of the tax). Multiply the taxable wage base by 15.3%, and add the results of the previous two steps together to arrive at the total self-employment tax. Calculate self-employment income; multiply by 0.9235 (1 − 0.0765); and multiply the result by 15.3%. Calculate self-employment income; multiply by 14.13%. Calculate self-employment income; multiply by 15.3%; multiply the excess over the taxable wage base by 2.9% (Medicare portion of the tax); and add the results of the previous two steps together to arrive at the total self-employment tax. A) I and IV B) I only C) I and II D) II and III

ans b The steps to calculate self-employment tax for 2020 where net income from self-employment is above the taxable wage base are as follows: Step 1: Calculate self-employment income. Step 2: Subtract 7.65% or multiply by 0.9235 (1 − 0.0765). Step 3: From Step 2, subtract the taxable wage base and multiply the excess over the taxable wage base by 2.9% (Medicare portion of the tax). Step 4: Multiply the taxable wage base by 15.3%. Step 5: Add the results of Steps 3 and 4 together to arrive at the total self-employment tax.

Which of the following are characteristics of a valid and enforceable premarital agreement? It may be orally executed by the parties that are affected. There should be a full and complete disclosure of each party's net worth prior to signing. It may be used to regulate an award of alimony upon divorce of the parties. There should be a written agreement with the willingly executed signatures of both parties. A) I and II B) II and IV C) II and III D) III and IV

ans b To be valid, a premarital agreement must be in writing and contain a complete disclosure of each party's financial situation. It may not be used to regulate an award of alimony.

When spouses who are legally married use the married filing jointly filing status, what is their liability for the combined income tax owed? A) Each spouse has liability for the tax due in proportion to their earnings reported on the return. B) Each spouse has joint and several liability for payment of the entire tax. C) A nonworking spouse has no liability for the tax owed. D) Each spouse owes 50% of the tax due.

ans b When filing in this manner, spouses have joint and several liability for the payment of tax, meaning each spouse is responsible for the entire tax and not just half.

Paul was divorced from his spouse, Pat, late in 2020. As part of the property settlement agreement, Paul agreed to transfer his interest in a residential rental property to Pat in exchange for release of marital claims. Paul's cost basis in this real estate tract was $50,000. The tract was appraised at a fair market value of $100,000 at the time of its transfer to Pat. Which of the following is an income tax implication of Paul's transfer of the real estate tract to Pat? A) Paul is allowed a deduction equal to the excess of the fair market value over his basis in the property. B) Paul's basis in the real estate is carried over to Pat for income tax purposes. C) Paul must recognize the gain on the real estate at the time of transfer as ordinary income. D) Pat receives a basis in the real estate equal to the fair market value at the time of transfer.

ans b When there is a transfer of property incident to divorce, the basis simply carries over to the other spouse. The transfer is not a taxable event. Pat will likely owe capital gains tax when disposing of the property.

A client sold an apartment building last year for $100,000, paying a sales commission of $5,000 plus $2,500 in closing costs. The building originally cost $80,000 20 years ago. Total straight-line depreciation of $40,000 had been taken. The building had a mortgage of $60,000 that was assumed by the buyer. The client is in the 24% marginal income tax bracket. What is the seller's adjusted cost basis? A) $32,500 B) $37,500 C) $40,000 D) $52,500 Explanation The seller's adjusted basis is the $80,000 purchase price, decreased by the $40,000 of straight-line depreciation. The mortgage has no bearing on the basis of the property.

ans c

A taxpayer currently is being audited by the IRS, and the agent has proposed a tax deficiency with which the taxpayer does not agree. The client has asked you to research the issue. Which one of the following sources is considered to be the most authoritative and, accordingly, would have the highest precedential value in defending the taxpayer's position to the IRS? A) Revenue Ruling B) Private Letter Ruling C) Treasury Regulations D) Revenue Procedure Explanation Treasury Regulations have the full force and effect of law. A Private Letter Ruling is never precedential, and Revenue Rulings and Revenue Procedures are merely administrative interpretations of the statutory tax law with lower authority than Regulations.

ans c

A taxpayer's personal residence was severely damaged as a result of a hurricane. The area was declared a federal disaster area. A portion of the damage will be covered by insurance. Which of the following statements is accurate regarding the casualty loss deduction in this situation? It is reduced by the amount of insurance coverage. It is reduced by a $100 floor per occurrence and 20% of AGI. It is based on the greater of the decrease in the fair market value of the residence or the adjusted basis of the residence. It is treated as an itemized deduction. A) II and IV B) I and III C) I and IV D) II, III, and IV Explanation The individual casualty loss deduction, an itemized deduction, is based on the lesser of the decrease in FMV or the adjusted basis of the asset. This amount is then reduced by insurance coverage, a $100 floor per occurrence, and 10% of AGI. Remember that, as a result of the Tax Cuts and Jobs Act (TCJA), the casualty loss deduction is allowed only for damage sustained in a federally declared disaster area.

ans c

Brandon failed to file a federal income tax return for last year's tax liability by April 15 of the current year. He filed his tax return October 30 of the current year and remitted the tax that was due. Which of the following statements is CORRECT? Brandon will owe interest to the IRS on the unpaid tax liability that runs from April 15 of the current year until the tax liability is paid in full. Because he took so long to file, the 90%/100% payment criteria does not apply. Brandon will be assessed the failure-to-file penalty. Depending on the facts and circumstances surrounding the return filed, Brandon could be assessed a negligence penalty. A) I only B) I, II, III, and IV C) I, III, and IV D) III and IV Explanation Statement II is incorrect. The 90%/100% underpayment criteria will still apply even though Brandon did not file in a timely manner. All of the other statements are correct.

ans c

Camille, a successful trial attorney, also races stock cars professionally. Camille has never won a race, but because she races the professional circuit she claims to be in the business of racing and thus has been deducting the cost of maintaining her cars and crew from her income from practicing law. If the IRS disallows this loss, it will most likely be because of the A) tax benefit rule. B) substance over form doctrine. C) hobby loss rules. D) sham transaction doctrine. Explanation The tax rules most likely to be applied in this situation are the hobby loss rules, which state that the taxpayer must have a profit motive in the activity in order to deduct losses from the activity.

ans c

Devonte had $175,000 of self-employment income and $25,000 in distributive share income from an S corporation. What is Devonte's self-employment tax for 2020? Round your answer to the nearest dollar. A) $21,068 B) $22,150 C) $21,761 D) $24,726 Explanation The distributive share of income from an S corporation is not subject to self-employment tax. (Answers may vary slightly due to rounding.) Actual earnings$175,000Less 7.65%(13,388)Net earnings from self-employment$161,612LESSMaximum amount of SE earnings subject to 15.3% tax (Social Security wage base)(137,700)Excess over wage base$23,912Medicare rate× 2.9%$693($137,700 × 15.3%)21,068Total$21,761 OR Schedule C net profit (business profit)$175,000Less 7.65% of Schedule C income($13,388)Self-employment earnings subject to self-employment taxes$161,612Social Security Tax Calculation (OASDI)Medicare Tax Calculation (HI)Earnings subject to self-employment tax$137,700$161,612Times tax rate12.4%(OASDI tax rate) = 2.9% (HI tax rate) = Equal self-employment taxes $17,075 (Social Security taxes) $4,687 (Medicare taxes)Total self-employment tax$21,761 (rounded)

ans c

During 2020, your client, Bob, purchased several items of equipment with a total cost of $265,000 for use in his sole proprietorship. Bob has taxable (earned) income from his Schedule C business of $112,000 (without regard to the Section 179 expense). He also has wages from a part-time job of $10,000. What is the maximum amount of Section 179 expense that Bob may deduct in the current year? A) $265,000 B) $112,000 C) $122,000 D) $1,000,000 Explanation The Section 179 expense election is limited to the taxable (earned) income of the taxpayer. For purposes of Section 179, salary or wages received as an employee, even from a completely unrelated source, are also considered to be from the active conduct of the trade or business. Thus, the total taxable (earned) income in this situation is $122,000. The maximum Section 179 expense election is $1.04 million (for 2020), but for Bob, it is limited to his earned or taxable income of $122,000 (increased by the Tax Cuts and Jobs Act, or TCJA).

ans c

During the current tax year, Carolyn has Section 1231 gains of $14,000 and Section 1231 losses of $3,000. Four years ago, Carolyn reported a net Section 1231 gain of $6,000. Three years ago, Carolyn had a Section 1231 loss of $4,000. Which of the following correctly describes the amount and treatment of the gain? A) $7,000 treated as ordinary gain; $4,000 treated as capital gain B) $11,000 treated as ordinary income C) $7,000 treated as capital gain; $4,000 treated as ordinary income D) $11,000 treated as capital gain income Explanation The current Section 1231 gain ($11,000) is treated as ordinary income to the extent of unrecaptured Section 1231 losses during the five-year lookback period. There are $4,000 of unrecaptured Section 1231 losses during the lookback period. Thus, $4,000 is ordinary income, and the remaining $7,000 is treated as long-term capital gain.

ans c

During the current tax year, Jamie has a $13,000 short-term capital loss and a $14,000 long-term capital gain, both from the sale of securities. Jamie also has a $10,000 long-term capital gain from the sale of collectibles. Jamie, a single taxpayer, is in the 32% marginal income tax bracket. Which of these accurately describes the result of these transactions? A) $1,000 long-term capital gain, taxed at 15%, and $10,000 collectibles gain, taxed at 28% B) $14,000 long-term capital gain and a $3,000 net capital loss carryforward C) $11,000 long-term capital gain taxed at 15% D) $11,000 long-term capital gain, taxed at 20% Explanation The $13,000 short-term capital loss is first used against the collectibles gain—the gain that would be taxed at the highest rate (28%). This eliminates the collectibles gain. The remaining $3,000 short-term capital loss is then used against the $14,000 long-term capital gain from the sale of securities. This leaves $11,000 of long-term capital gain, taxed at 15%. We know the long-term capital gain is taxed at 15%, as the top of the 32% MITB is $207,350 (2020) for a single taxpayer, and the 20% LTCG rate doesn't kick in until $441,450 for a single taxpayer.

ans c

During the current tax year, Jamie sold several securities that resulted in the following types of gains and losses: a long-term capital loss of $6,700; a short-term capital loss of $7,000; a long-term capital gain of $1,900; and a short-term capital gain of $9,200. What is the net capital gain or loss on Jamie's security sales? A) Net short-term gain of $7,300; net long-term loss of $300 B) Net long-term loss of $4,800; net short-term gain of $2,200 C) Net long-term loss of $2,600 D) Net short term loss of $3,800 Explanation The long-term items are netted, leaving a long-term capital loss of $4,800. The short-term items are netted, leaving a short-term capital gain of $2,200. These are netted, leaving a net long-term capital loss of $2,600.

ans c

During the current tax year, Rod purchased a building for exclusive use in his manufacturing business. The cost of the property was $422,000, of which $122,000 was attributable to the land. Which of the following statements identifies the proper treatment of the expenditure? A) The cost attributable to the building may be deducted under Section 179. B) The $300,000 attributable to the building may be currently deductible. C) The $122,000 must be capitalized and may not be depreciated. D) The $122,000 must be capitalized and may be depreciated. Explanation The land may not be depreciated, as only "wasting" assets are subject to depreciation. The Section 179 expense election generally applies to personalty only, and is not available for most real estate. The cost of the building may not be currently deducted; it must be capitalized and depreciated because it has a useful life of over one year.

ans c

Dwight has an active participation rental real estate activity. In 2019, he had losses of $20,000 from the active participation real estate and his AGI was $140,000. He was allowed to deduct $5,000 of the losses against other income. The remaining $15,000 loss was carried forward into 2020. In 2020, Dwight has an AGI of $90,000 and only $6,000 of current losses from his real estate rental activity. What amount of loss, if any, may Dwight deduct in 2020? A) $0 B) $15,000 C) $21,000 D) $6,000 Explanation Dwight is allowed to deduct $21,000 of losses in 2020. The $15,000 carryforward losses are treated as if they occurred in 2020. The $6,000 current losses plus the $15,000 carryover losses total $21,000. This loss would be fully deductible as the AGI is under $100,000.

ans c

Ethel had the following from securities transactions during the current year: Long-term capital gain: $6,400 Long-term capital loss: $2,200 Short-term capital gain: $2,300 Short-term capital loss: $5,500 Which of the following describes the net capital gain or loss reportable by Ethel for the current tax year? A) $900 net long-term capital gain; $100 net short-term capital loss B) $1,000 net long-term capital loss C) $1,000 net long-term capital gain D) $4,200 net long-term capital gain; $3,200 net short-term capital loss Explanation Long-term transactions are netted together, as are short-term transactions. The net long-term capital gain is $4,200 ($6,400 - $2,200). The net short-term capital loss is $3,200 ($2,300 - $5,500). The net short-term capital loss is netted with the net long-term capital gain ($4,200 - $3,200) to result in a net long-term capital gain of $1,000.

ans c

Fred, age 59, is a single taxpayer. He has wage income of $90,000 for the current tax year. Fred is not an active participant in a company-maintained retirement plan. In addition, he has the following: Long-term capital gains$4,000Short-term capital losses$9,000Loss from active participation rental real estate$3,700Alimony paid to ex-wife$5,200Gambling winnings$7,100Gambling losses$4,100Interest income$3,500Sole proprietorship (Schedule C) income$2,000Self-employment tax liability$283Qualified home mortgage interest$11,890Real estate tax paid$1,840Investment interest expense$4,925Charitable contributions (cash)$2,975Total medical expenses$4,217State and local income taxes$1,625Consumer interest$2,180Unreimbursed employee business expenses$1,560IRA contribution$6,000 Fred's divorce was finalized in 2017. What is the amount of Fred's allowable itemized deductions? A) $26,647 B) $35,712 C) $25,930 D) $28,607 Explanation The itemized deductions total $20,930. This is composed of the qualified home mortgage interest of $11,890, the real estate tax paid of $1,840, the investment interest expense of $3,500, the charitable contributions of $2,975, the state and local income taxes of $1,625, and the gambling losses to the extent of gambling winnings ($4,100). Note that the unreimbursed employee business expenses are not deductible because of TCJA. Also, the medical expenses are not deductible because they do not exceed 7.5% of adjusted gross income. Also note that in this situation the investment interest expense of $4,925 is deductible only up to the amount of net investment income, which in this situation is $3,500 (the interest income). Consumer interest (interest on personal auto loans, credit card debt, etc.) is nondeductible. Home mortgage interest$11,890Property taxes$1,840Investment interest expense$3,500Charitable contributions$2,975State and local income taxes$1,625Gambling losses (to extent of winnings)$4,100Allowable itemized deductions$25,930

ans c

Which of the following is NOT one of the methods by which an S corporation can be terminated? A) When it fails to meet the requirements of a small business corporation B) If gross income for 3 years in a row is of a certain type that exceeds a certain share of total income C) If it earns more than $10 million in gross receipts in any one year D) A majority vote of the shareholders Explanation The only time that earnings can revoke the S status is when more than 25% of gross receipts for three successive years come from certain types of passive income and the corporation has accumulated earnings and profits from its operations prior to the S election.

ans c

Harry has a vacation home and is considering renting it to others when he is not using it. It appears that his property will be a mixed-use rental. Which of the following statements regarding mixed-use rentals is CORRECT? A) The rental income and expenses are reported on Schedule A of IRS Form 1040. B) The mortgage interest and property taxes allocable to the personal use of the vacation home are deductible on Schedule E of Form 1040. C) Rental expenses can be deducted only to the extent of rental income. D) Up to $25,000 of rental losses can be deducted against income. Explanation In mixed use rentals, where the residence test is met, rental expenses can be deducted only to the extent of rental income. The mortgage interest and property taxes allocable to the personal use of the vacation home are deductible on Schedule A of Form 1040 as itemized deductions. The rental income and expenses are reported on Schedule E of IRS Form 1040. There is no possibility of a $25,000 ordinary loss against income.

ans c

Helen purchased an antique cabinet as an investment for $30,000 a few years ago. On January 15 of this year, she sold the cabinet to an art museum for $120,000 in an installment sale. She received a down payment of $12,000 and a note requiring monthly principal payments (to begin in March of this year) of $5,000. What amount of gain must Helen recognize for the current year? A) $62,000 B) $42,500 C) $46,500 D) $50,000 Explanation Step 1: Calculate gross profit percentage: profit divided by sale price. $90,000$120,000=75%$90,000$120,000=75% Step 2: Calculate payments received in current year. $12,000 down payment + (10 × $5,000) = $12,000 + $50,000 = $62,000 (payments received) Step 3: Calculate gain recognized for current year. gross profit percentage×payments received=gain recognized75%×$62,000=$46,500

ans c

In 2020, Jim had an AGI of $160,000. What percentage of prior-year income tax must be paid by Jim in 2020 to avoid the imposition of a penalty for underpayment of estimated tax? A) 90% B) 100% C) 110% D) 80% Explanation The common exception for avoiding estimated tax penalties generally is 90% of the current year's tax or 100% of the prior year's tax. However, if the prior year's AGI exceeds $150,000, then the requirement is 90% of the current year tax or 110% of the prior year's tax liability.

ans c

In February, Bryan purchased a new high-speed copier for use in his printing business. The cost of the copier was $8,250, sales taxes were $550, and installation charges totaled $1,200. Assume that Bryan opts out of the bonus depreciation provision. What is the first-year cost recovery deduction using the straight-line method? A) $880 B) $945 C) $1,000 D) $2,000 Explanation The installation charges of $1,200 and the sales taxes of $550 must be capitalized—that is, added to the cost of the copier to give a total basis of $10,000. A copier is five-year property. (Copiers, cars, computers, and computer peripherals are five-year properties; furniture and other equipment are seven-year properties.) The straight-line rate for five-year property is 20% (100% divided by five), but the half-year convention limits the deduction to half of a full year's depreciation in the year of acquisition. Thus, $10,000 times 10% equals $1,000. If Bryan had not opted out of bonus depreciation, the entire $10,000 would be depreciated in the first year.

ans c

In September, Eric purchased a new computer for use in his business. The cost of the computer was $4,300, while freight and setup charges totaled $600. Assume that Eric opts out of the bonus depreciation provision. What is the cost recovery deduction using the Modified Accelerated Cost Recovery System (MACRS)? A) $790 B) $430 C) $980 D) $490 Explanation A computer is five-year property. The first-year deduction using the MACRS table is 20%. Thus, 20% times $4,900 equals $980. Please note that the MACRS table has all applicable conventions already built into the table—thus, no adjustment is necessary for the half-year convention. If the MACRS table is not available, remember that the table reflects a 200% declining balance method, with a half-year convention. A computer is five-year property. (Copiers, automobiles, computers, and computer peripherals are five-year properties; furniture and other equipment are seven-year properties.) The straight-line rate for five-year property is 20% (100% divided by five). Twice the straight-line rate is 40%. So, 40% times the basis of $4,900 is $1,960, but the half-year convention limits the deduction to half of a full year's depreciation in the year of acquisition. Thus, half of $1,960 equals $980. If Eric had not opted out of the bonus depreciation, the entire $10,000 would be depreciated in the first year.

ans c

In the current year, William has passive losses of $21,000 from an equipment-leasing limited partnership purchased many years ago. He also has passive income of $11,000 from an older real estate limited partnership. Both limited partnerships are not publicly traded. What is the total amount of passive losses that may be used to offset active, passive, and portfolio income in the current year? A) $13,000 B) $21,000 C) $11,000 D) $12,000 Explanation The passive losses of $21,000 are deductible, but only up to the current year's income ($11,000).

ans c

Jane has the following income and losses for the current year: $1,500 loss from a 30% interest in Laminate Partnership in which she does not materially participate $500 loss from a .2% limited partnership interest in Venture, a limited partnership $2,400 loss from a 12% interest in an S corporation in which she manages one of the departments $32,000 salary as manager with an S corporation $600 of dividend income from Higher Mutual Fund What is Jane's adjusted gross income? A) $28,200 B) $32,600 C) $30,200 D) $32,000 Explanation She has $32,600 of income from salary and dividends. The S corporation loss is deductible because the information provided leads us to the conclusion that she is materially participating in the S corporation. The Laminate Partnership and the Venture LP are both passive activities; therefore, the losses would be deductible only against passive income.

ans c

Jasper invested $20,000 for a 30% interest in a nonpublicly traded limited partnership. Jasper is not a material participant. The partnership has a loss this year and Jasper's share is $15,000. This is the only nonpublicly traded limited partnership Jasper owns an interest in, but Jasper did have portfolio income of $9,000 in the same year in addition to his salary of $150,000. How much of the loss can Jasper deduct this year? A) $15,000 B) $9,000 C) $0 D) $6,000 Explanation Because Jasper has no passive income to offset the passive loss, the loss is suspended under the passive activity loss rules.

ans c

Ron established a revocable trust for the benefit of his destitute sister, Barbara. The trust is to last until Barbara's death, or until Ron revokes it. Ron would revoke the trust only in the event of his own financial difficulty. Ron named a local bank as the trustee. Which of the following parties will be taxed on the income generated by the trust? A) The bank B) Barbara C) Ron D) The trust

ans c Ron will be taxed on the trust assets because he has retained the right to revoke the trust.

John has the following items from four separate investments during the current tax year: Passive income from a publicly traded limited partnership: $8,000 Passive loss from a publicly traded limited partnership: $10,000 Passive income from a nonpublicly traded limited partnership: $17,000 Passive loss from a nonpublicly traded limited partnership: $9,000 What is the total amount, if any, of passive losses that may be deducted during the current year? A) $19,000 B) $17,000 C) $9,000 D) $0 Explanation Losses from a non-PTP may be deducted up to the amount of income from a non-PTP. In this situation, the passive loss of $9,000 may be deducted in full against the $17,000 of passive income. The income from a PTP may not be offset by passive losses arising from any other source, and the losses from a PTP must be held in suspense until that same partnership generates income.

ans c

Judy's rental real estate was completely destroyed by a fire in June of 2020, and she received the reimbursement check from the insurance company later that month. If there were a gain on the conversion, when would the replacement period end? A) December 31, 2023 B) June 30, 2023 C) December 31, 2022 D) June 30, 2022 Explanation The replacement period for a casualty or theft begins on the date the property was damaged, destroyed, or stolen and ends on the last day of the second taxable year following the year in which the taxpayer realizes a gain with respect to the property. If this was a condemnation of business or rental real estate, the replacement period would end on the last day of the third taxable year following the year in which the taxpayer realizes a gain from the property.

ans c

Kevin is a college professor who has an accounting business on the side that he runs as a C corporation. He lives entirely off his teaching salary and has never withdrawn any salary or dividend from the accounting practice (a personal service corporation), preferring to save the money within the corporation (from which it will be withdrawn upon retirement). The corporation currently has retained earnings and profits of $500,000. With which of these should Kevin be most concerned? A) Assignment of income B) Unreasonable compensation C) Accumulated earnings tax D) Ownership attribution Explanation In addition to the income tax, corporations are taxed on earnings that are accumulated and not distributed to shareholders when a valid business purpose does not exist for the accumulation. An exemption of $150,000 is allowed to C corporations that are considered personal service corporations. An accumulation of $500,000 would potentially subject the corporation to an accumulated earnings tax of 20% on the $350,000 accumulated in excess of the exemption amount.

ans c

Kris anticipates adjusted gross income of $100,000 for the current tax year. She is considering making a gift of appreciated real estate to her church, a qualified charitable institution. Kris's adjusted basis in this real estate is $20,000. The real estate has a current fair market value of $50,000. Kris has owned the real estate for 15 years. If Kris does gift the real estate to her church, what is the maximum allowable charitable deduction she can receive in the current tax year? A) $100,000 B) $50,000 C) $30,000 D) $20,000 Explanation The current deduction for a contribution of long-term capital gain property to a 50% organization is based on FMV but is limited to 30% of AGI. This results in a $30,000 current year deduction with a $20,000 carryforward.

ans c

Last month, Amanda received 100 shares of stock from her aunt, Martha, as an inheritance. Martha purchased the stock 10 years ago for $150 per share. The fair market value on the date of Martha's death was $65 per share, and the fair market value six months after the date of death was $70 per share. Assume that the administrator did not elect the alternate valuation date. Amanda sold the stock this month for $90 per share. What is Amanda's per-share gain or loss in the acquired stock? A) $20 per share long-term capital loss B) $60 per share long-term capital loss C) $25 per share long-term capital gain D) $25 per share short-term capital gain Explanation The basis of property acquired by inheritance is simply the fair market value on the date of the decedent's death. In this case, that value was $65 per share. The holding period of an asset acquired from a decedent is presumed to be long term. Thus, even though Amanda held the stock for only a month or so, she still has a long-term capital gain.

ans c

Mary has owned her principal residence for over six years. Two years ago, she married John, who immediately moved into the residence. John has never used the Section 121 exclusion. If Mary sells the residence this year and John and Mary file a joint return, which of the following statements is CORRECT with respect to the availability of the Section 121 exclusion? A) The maximum exclusion is $500,000 because that is the amount always available for married taxpayers who file jointly. B) The maximum exclusion is $250,000 because John is not an owner of the residence. C) The maximum exclusion is $500,000 because Mary has at least two years of ownership, and both spouses meet the use requirement. D) The maximum exclusion is $250,000 because that is the maximum exclusion for an individual who was single when the residence was purchased. Explanation Currently, Section 121 allows for a gain exclusion, of up to $500,000 for taxpayers married filing jointly, to any taxpayer who satisfies certain tests, known as the ownership test and the use test. To satisfy the ownership test, the home must have been owned and used as a principal residence for at least two of the five years preceding the date of sale. (Note: These years do not have to be consecutive; they only have to add up to at least two years.) Either spouse can meet the ownership test, but both must meet the use (two-out-of-five-year) test. This is likely not difficult for most married couples (applies even to those living in the house and then getting married), but it can be burdensome for individuals who are divorced or in the process of a divorce.

ans c

Which of the following are requirements for a taxpayer who materially participates in a real property trade or business to be able to deduct any losses from the business? More than 50% of the individual's personal services during the tax year are performed in the real property trades or businesses in which the individual materially participates. More than 10% of the individual's net assets at the end of the tax year are invested in the real property trades or businesses in which the individual materially participates. The individual performs more than 750 hours of service in the real property trades or businesses in which the individual materially participates. A) II and III B) I only C) I and III D) I and II Explanation Options I and III are specific requirements that must be met for a taxpayer to be able to deduct any losses from a real estate trade or business. The amount of the individual's net assets used in the business is not relevant.

ans c

Which one of the following is NOT a goal of the federal income tax system? A) Inflation control B) Price stability C) Monetary policy D) Economic growth Explanation The Federal Reserve System controls monetary policy. Income tax policy is used to influence all of the other goals.

ans c`

On December 20, 2003, Jody moved into a condominium that she owns and had rented to tenants since July 1, 1996. Her cost basis in the condo was $238,440. Jody took depreciation deductions totaling $54,000 for the period that she rented the property. After moving in, she used the residence as her principal residence. Jody sells the property on August 1, 2020, for $538,000. Jody is in the highest marginal income tax bracket for the current year. What is the amount and character of the recognized gain resulting from the sale? A) $54,000 of unrecaptured Section 1250 income; $299,560 of "regular" long-term capital gain B) $353,560 "regular" long-term capital gain C) $54,000 of unrecaptured Section 1250 income; $49,560 of "regular" long-term capital gain D) $54,000 of ordinary income; $49,560 of "regular" long-term capital gain Explanation Jody's gain realized (the actual economic gain) from the sale is $353,560 ($538,000 of sales proceeds reduced by the adjusted basis of $184,400). Of this $353,560 of gain, the first $54,000 is recognized as unrecaptured Section 1250 gain, taxed at 25%. Unrecaptured Section 1250 gain is the gain created by the straight-line depreciation. This leaves $299,560 of gain to account for. Jody used the condo as her principal residence for two full years—thus, she is eligible to exclude $250,000 under Section 121. This leaves $49,560 of long-term capital gain to be recognized at a 20% rate (because she is in the highest marginal income tax bracket, her taxable income exceeds the $425,800 breakpoint for the 20% LTCG rate). The recognized gain is the gain on which Jody will pay taxes. Note that the nonqualified use provision does not come into play here as there was no nonqualified use after 2008.

ans c

Sandra and Colby, a married couple, ask you to explain how taxable income is calculated after adjusted gross income has been determined. Which of the following is a deduction from adjusted gross income (AGI) to arrive at taxable income? Additional standard deduction Itemized deductions Exclusions Tax credits A) I, II, III, and IV B) IV only C) I and II D) II and III Explanation To compute the taxable income, we subtract the greater of the standard deduction (including the additional standard deduction for elderly or blind) or the itemized deductions from AGI. We also subtract the qualified business income (QBI) deduction, if any, to arrive at the taxable income. Exclusions are not reported on the return, so they don't need to be subtracted. Tax credits are deducted from the tax liability.

ans c

Several years ago, Ben purchased equipment at a cost of $11,000 to use in his dry-cleaning business. He used the straight-line method of cost recovery. He deducted $7,074 of cost recovery. He sold the equipment earlier this year for $12,000. What is the amount and nature (character) of the gain resulting from this disposition? A) No Section 1245 gain; $8,074 Section 1231 gain B) $8,074 Section 1245 gain; no Section 1231 gain C) $7,074 Section 1245 gain; $1,000 Section 1231 gain D) $1,000 Section 1245 gain; $7,074 Section 1231 gain Explanation The gain realized and recognized is the difference between the $12,000 amount realized from the sale, reduced by the adjusted basis of $3,926. Thus, the total gain is $8,074. The Section 1245 cost recovery recapture is the lesser of the cost recovery deductions taken ($7,074) or the gain realized ($8,074). Thus, the Section 1245 recapture is $7,074. The remaining $1,000 of gain is attributable to actual appreciation of the asset; therefore, there is $1,000 of Section 1231 gain. The Section 1245 income is treated as ordinary income, while the Section 1231 income generally receives long-term capital gain treatment. For Section 1245 property, it makes no difference whether straight-line or accelerated depreciation were used. LO 6.1.4

ans c

The following summarizes several financial events in the life of George during the current tax year. Received $100,000 from a life insurance policy due to the death of his brother Had gambling winnings of $45,000, while incurring gambling losses of $20,000 Received net royalties of $10,000 from an oil and gas investment Received $5,000 of unemployment compensation Had job-related moving expenses of $4,000 Contributed $6,000 to an IRA Assuming George is NOT a professional gambler, what is his total income for the current tax year? A) $34,500 B) $155,000 C) $60,000 D) $45,500 Explanation Total income is basically the starting point of the income tax calculation. The gambling winnings of $45,000, the unemployment compensation of $5,000, and the royalties of $10,000 are all included in income. Gambling losses are an itemized deduction, to the extent of gambling winnings; thus, they do not affect the total income. The life insurance proceeds received by reason of death of the insured are excluded from income. The IRA contribution is a potential adjustment to income, and does not affect the total income. Job-related moving expenses are only deductible for active duty military personnel who are undergoing a change of station.

ans c

Three years ago, Sam received a gift of 100 shares of common stock from his uncle. The fair market value of the stock on the date of the gift was $12 per share. His uncle had purchased the stock four years earlier at $5 per share. Sam sold this stock for $17 per share last week. What was Sam's per-share basis in the stock when it was sold? A) $17 B) $12 C) $5 D) $22 Explanation If the fair market value on the date of the gift is greater than the donor's adjusted basis, the donor's adjusted basis is used as the recipient's basis. Note that the donor's holding period would be tacked to the donee's holding period.

ans c

Under the passive activity loss rules, passive losses can be deducted against which of the following? Active income Passive income Portfolio income A) I and III B) III only C) II only D) I, II, and III Explanation Statement II is correct. Statements I and III are incorrect. Passive losses cannot be deducted against either active income or portfolio income.

ans c

What, if any, is the primary difference in tax treatment between a general partnership and a limited partnership? A) Limited partnerships are taxed as corporations, while general partnerships are taxed as partnerships. B) The limited partners are treated only as capital investors, whereas the active partners receive both ordinary and capital distributions. C) None of these. D) The limited partners only receive capital distributions, while the general partners receive only ordinary income distributions. Explanation As long as the limited partnership is classified as a partnership (and not a C corporation) for tax purposes, the taxation is no different than it would be if the organization were a general partnership. That is, the partnership issues a K-1 to all of the partners for their distributive share of items of income and loss. Limited partners generally must treat net income or loss from the partnership as passive.

ans c

Which of the following statements concerning the passive activity loss rules is NOT correct? A) Losses from one master limited partnership activity may only offset income from that particular activity; they cannot be used to offset income from any other passive activities. B) Losses from passive activities may not offset portfolio or active income except under limited circumstances. C) The passive activity loss rules limitation is a permanent disallowance rule. D) The $25,000 offset allowance for the small real estate investor is not available to taxpayers whose AGI is $150,000 or more. Explanation The passive activity loss limitation is not a permanent disallowance rule. When a taxpayer disposes of his entire interest in a fully taxable transaction to an unrelated purchaser (not a related party), his suspended losses from that activity, including any losses incurred in the year of disposition, are generally deductible in full.

ans c

Which of the following statements regarding material participation is CORRECT? A) It only applies to portfolio income and losses. B) It is a much less rigorous standard than active participation. C) It usually refers to a taxpayer's main business. D) It only applies to real estate and associated rentals. Explanation Material participation is a much more rigorous standard than active participation. The statute provides that a taxpayer is materially participating only if he or she is involved in the operations of the business on a regular, continuous, and substantial basis.

ans c

Which of these best describes a tax benefit associated with an active participation real estate investment? A) The first $25,000 of taxable income from the investment is tax-exempt. B) Losses may be used to offset active or portfolio income. C) Up to $25,000 in losses may be used to offset active or portfolio income. D) There are no income limitations associated with this investment. Explanation Active participation rental real estate allows for losses of up to $25,000 on an annual basis. The full deduction is only available if the AGI is under $100,000. There is a phaseout between $100,000 and $150,000 of AGI. It is true that the losses may be used to offset active or portfolio income, but up to $25,000 in losses may be used to offset active or portfolio income is the most complete answer.

ans c

Which of these is a characteristic of a policyholder dividend paid by an insurance company? A) It is received as a result of a partial or complete liquidation. B) It represents the right to subscribe to a new issue of stock. C) It generally is exempt from taxation. D) It is taxed the same as a dividend from a corporation. Explanation A policyholder dividend is generally tax exempt, as a return of unused premium. It will be taxable to the extent that the dividends paid exceed the investment in the contract. Also, if the dividend is from a MEC, it is taxable if received as cash or used to pay a loan.

ans c

Which of these is most often considered an advantage of an S corporation? A) Two classes of shareholders are permitted. B) The business may be a foreign corporation. C) There is a flow-through of ordinary loss. D) The number of shareholders is limited. Explanation The ability to have losses flow through to the shareholders to offset other income is one of the primary advantages of the S corporation. The limitation on the number of shareholders is not considered an advantage. Only one class of stock is permitted (although there may be differences in voting rights within that single class of stock), and the business must be a domestic corporation.

ans c

Which one of the following is CORRECT regarding an active participation rental real estate activity? A) A deduction-equivalent tax credit of up to $25,000 is available. B) A cumulative deduction of up to $25,000 is available over the life of the activity. C) A deduction of up to $25,000 is available annually. D) A cumulative deduction-equivalent tax credit of up to $25,000 is available over the life of the activity. Explanation In meeting several tests, an individual with active participation in real estate may deduct up to $25,000 of rental real estate losses against active and portfolio income in any one year.

ans c

Which one of the following is NOT a type of tax audit performed by the IRS? A) A correspondence audit B) A field audit C) A home audit D) An office audit Explanation The three types of tax audit performed by the IRS are as follows: a correspondence audit, which is usually performed through the mail because the disputed tax issue is minor; an office audit, which is usually restricted in scope to a specific item or items and is performed at the IRS office by an office auditor; and a field audit, which is an examination of numerous items and is usually performed on the premises of the taxpayer (such as a business office) by a revenue agent. LO 8.2.2

ans c

Which one of the following is a publication of specific taxpayer guidance from the IRS? A) Regulations B) Revenue Rulings C) Private Letter Rulings D) Revenue Procedures Explanation Private Letter Rulings are taxpayer guidance from the IRS that apply only to the particular taxpayer(s) asking for the ruling; they are not applicable to all taxpayers. The primary purpose of the regulations is to explain and interpret particular IRS Code sections.

ans c

Which one of the following is an application of the administrative powers of the Internal Revenue Service and not of the powers of Congress? Regulations Revenue Rulings A) Neither I nor II B) Both I and II C) II only D) I only Explanation Regulations are a direct extension of the lawmaking powers of Congress, whereas revenue rulings are an application of the administrative powers of the Internal Revenue Service

ans c

Your client has a salary of $80,000, dividends of $20,000, and limited partnership income of $15,000. This year, she invested in an equipment-leasing partnership. Her initial investment included $50,000 cash and a nonrecourse note for $60,000. What is the maximum tax deduction your client may take from this equipment-leasing investment this year? A) $50,000 B) $35,000 C) $15,000 D) $45,000 Explanation Passive losses are only deductible against passive income. She has passive income of $15,000 from the limited partnership, thus she could deduct up to $15,000 of passive losses from the equipment-leasing limited partnership.

ans c

Jim is planning to make a charitable contribution to a local university, a qualifying charitable organization. He is going to contribute a piece of real estate that he has owned for six years. The fair market value of the property is $80,000 and his basis in it is $55,000. He has an AGI of $120,000. What can you accurately tell Jim about the effect of a 50% election? A) The current-year deduction is $60,000 with a $20,000 carryforward. B) The current-year deduction is $40,000 with a $15,000 carryforward. C) The current-year deduction is $55,000 with no carryforward. D) The current-year deduction is $55,000 with a $25,000 carryforward.

ans c A 50% election allows a deduction based on the property's basis with a 50% of AGI limitation. Remember that the election applies to gifts of long-term capital gain property to a 50% organization only.

Which of the following must be true for someone to be claimed as a dependent for another taxpayer? A dependent may not have more than $4,300 (2020) of gross income. The taxpayer must provide over 75% of the dependent's support. A person who dies during the year may be identified as a dependent. Social Security payments are always included in the dependent's gross income. A) I and II B) III and IV C) I and III D) II and IV

ans c A dependent may not have more than $4,300 (2020) of gross income. Social Security income is excluded from the test if that is the elder's only source of income. The taxpayer must also provide over 50% of the dependent's support to claim them. Coincidentally, as long as all the tests are met, a person who dies during the year may still be identified as a dependent.

Chris Burdick anticipates adjusted gross income of $200,000 for the current tax year. He contributed appreciated stock to the United Way. Chris's adjusted basis in this stock is $50,000. The stock has a current fair market value of $140,000. Chris has owned the stock for 12 years. If Chris does gift the stock to the United Way, what is the maximum allowable charitable deduction he can receive in the current tax year? A) $140,000 B) $100,000 C) $60,000 D) $50,000

ans c A gift of long-term capital gain property to a 50% organization is based on the FMV of the property, with the deduction for the current year limited to 30% of AGI.

Adrian Brown owned 500 shares of XYZ growth and income fund. She has become increasingly dissatisfied with the performance of the fund and, upon the advice of a friend, decided to execute a "telephone transfer" and switch the balance in the fund to the XYZ intermediate bond fund. Which one of the following describes the tax effect of such a strategy? A) No gain or loss will be recognized by the taxpayer, and the basis in the new fund will be the same as that of the old fund. B) Any loss will be recognized by the taxpayer, but any gain will be deferred through a reduction in the basis of the new fund. C) Gain or loss will be recognized by the taxpayer on the redemption of the old fund. D) No gain or loss will be recognized by the taxpayer, but the basis of the new fund will be reduced by any deferred gain or increased by any unrecognized loss.

ans c A telephone transfer is the same as a sale or other taxable redemption of the fund. Therefore, gain or loss will be recognized based on the difference in the redemption proceeds and the basis in the shares redeemed. This is true even if the transfer is made between two funds in the same fund family.

Which of the following statements about S corporations is CORRECT? A) S corporations are prohibited from earning passive income. B) S corporation status is automatic if there are fewer than 100 shareholders. C) S corporations are prohibited from having more than one class of stock. D) S corporations may have nonresident aliens as shareholders.

ans c An S corporation may have only one class of stock, although differences in voting rights are allowed within that one class of stock. An S corporation may have no more than 100 shareholders; however, the corporation must elect S status by filing a Form 2553—the treatment is not automatic. An S corporation may have passive income, although excess passive income may trigger the sting tax if the corporation had been a C corporation with earnings & profits (E&P). Nonresident aliens may not be shareholders in an S corporation.

Macy, age 22 and disabled, has been living with her older brother, Leon, since last December when their parents died in an auto accident. Leon has been providing all of her support as her share of the life insurance benefit was put into a trust for her college education. Leon is asking his planner about Macy's status as a qualifying relative because her life insurance benefit was $250,000. He hopes to use the head of household status this year. What does the planner tell him? A) Leon cannot list Macy as a dependent and claim head of household status. B) Even though Leon supported Macy, Leon cannot list her as a dependent because her insurance benefit was so large. C) Because Leon provided all of Macy's support this year, he may claim her as a dependent. D) Only the trust can list Macy as a dependent on its return.

ans c Because Leon provided all of Macy's support this year, he can claim her as a dependent and claim head of household status.

Ron, age 43, and Sandy, age 41, are married with two children, Michael, age 12, and Victoria, age 8, who has been blind since her birth. Ron is an architect and general partner with XYZ partnership. Sandy is self-employed as an attorney and works out of a home office. Her home office is exclusively and regularly used for business, and the home office is her principal place of business. Their information for the tax year 2020 is as follows: AGI: $217,300 Itemized deductions (including qualified residential mortgage interest, taxes paid, and charitable contributions): $33,000 Early in the current year, Sandy's father died. Sandy is the sole beneficiary of her father's entire estate. The estate is presently in the probate process. Sandy's mother, Lisa, age 68, has moved in with them but provides her own support. She was married to Sandy's father when he died earlier this year. This is Ron's second marriage. He makes monthly support payments to his former spouse and his daughter. Because both Ron and Sandy are considered to be self-employed, they make quarterly estimated tax payments each year to cover both their income tax and self-employment tax obligations. Based on the information provided in the case scenario for Ron and Sandy, which of the following statements regarding Sandy's home office deduction is CORRECT? A) Sandy is not eligible to deduct her home office expenses. B) The net income from the business use of Sandy's home must equal or exceed the business expenses (including depreciation). C) Sandy may deduct her qualifying home office expenses when calculating AGI. D) The amount Sandy may deduct this year for qualifying home office expenses is unlimited.

ans c Because Sandy is self-employed, she can deduct her qualifying home office expenses when calculating AGI (as a line item on Schedule C). The amount of her home office expense deduction this year is limited to the net income from her business. A profit from the business is not one of the tests for qualification of a home office for tax deduction purposes. Rather, it is one of the limitations on the deductible amount after it qualifies.

Which one of the following statements is incorrect regarding investment interest expense? A) Interest paid or accrued to purchase or carry tax-exempt investments is not deductible. B) Investment interest expense is deductible up to the amount of the net investment income. C) Excess investment interest expense cannot be carried forward into succeeding tax years. D) Net investment income is the taxpayer's investment income—typically interest, nonqualified dividends, and short-term capital gains.

ans c Excess investment interest expense can be carried forward into succeeding tax years. Investment interest expense is deductible up to the amount of net investment income. The interest on funds borrowed to purchase tax-exempt investments is not deductible. The net investment income is typically interest, nonqualified dividends, and short-term capital gains. Long-term capital gains and qualified dividends may be included at the taxpayer's election, but the taxpayer must forgo the preferential tax rates on these items.

Which of the following statements regarding charitable gifts is CORRECT? The total deduction of a cash gift is the fair market value of the cash, limited to an annual deduction of 50% of the donor-taxpayer's AGI. If the donor elects a total maximum deduction equal to the FMV of appreciated property (long-term capital gain property), the donor is limited to an annual deduction of no more than 30% of AGI if the recipient of the gift is a public charity. If the recipient of a gift of appreciated property (long-term capital gain property) is a private charity or foundation, the donor is limited to an annual deduction of no more than 20% of AGI. If a donor elects a total maximum deduction equal only to the tax basis in appreciated property (long-term capital gain property), the donor is permitted an annual deduction of up to 50% of AGI if the recipient of the gift is a private charity. A) I and IV B) III and IV C) II and III D) I only

ans c For a cash gift, the amount of the total deduction is the FMV of the cash, limited to an annual deduction of either 60% or 30% of the donor-taxpayer's AGI, depending on the recipient of the gift. If the donor elects a total maximum deduction equal to the FMV of appreciated property (long-term capital gain property), the donor is limited to an annual deduction of no more than 30% of AGI if the recipient of the gift is a public charity. If the recipient of a gift of appreciated property is a private charity or foundation, the donor is limited to an annual deduction of no more than 20% of AGI. If a donor elects a total maximum deduction equal only to the tax basis in appreciated property, the donor is permitted an annual deduction of up to 50% of AGI if the recipient of the gift is a public charity. The donor is still limited to a 20% of AGI annual deduction if the recipient is a private charity.

Kris anticipates adjusted gross income of $100,000 during the current tax year. She is considering making a gift of appreciated real estate to the university she attended, Sinton Tech, a qualified charitable institution (a public charity). Kris's adjusted basis in this real estate is $40,000. The real estate has a current fair market value of $50,000. Kris has owned the real estate for six months. If Kris does gift the real estate to Sinton this year, what is the maximum allowable charitable deduction Kris can receive for the current tax year? A) $100,000 B) $30,000 C) $40,000 D) $50,000

ans c For a gift of ordinary income (short-term capital gain) property to a 50% organization, the deduction is based on the basis of the property with a 50% of adjusted gross income limitation. 50% of her AGI is $50,000. However her adjusted basis in the property is $40,000. Thus her deduction is limited to $40,000.

Jim made charitable contributions of cash to a local university, a qualifying charitable organization. Jim's cash contributions for the current year totaled $80,000. Jim has an AGI of $120,000. What is the amount of charitable contribution deduction that Jim may claim in the current year? A) $36,000 B) $24,000 C) $72,000 D) $60,000

ans c Gifts of cash to a 50% organization are limited to 60% of AGI under TCJA. Sixty percent of the $120,000 AGI equals $72,000. Jim would also have an $8,000 carryforward.

As a planner, you have grown increasingly concerned about the impact of the alternative minimum tax. Which of the following clients is least likely to have exposure to the alternative minimum tax? A) The client who is heavily invested in private-activity municipal bonds B) The client who has exercised incentive stock options during the tax year C) The client who has no itemized deductions D) The client who is invested in oil and gas activities

ans c Investment in oil and gas activities often causes AMT exposure due to the intangible drilling costs and percentage depletion. The bargain element on the exercise of an ISO and the interest from private-activity municipal bonds are preference items for AMT purposes. The client least likely, in this case, to have an AMT exposure is the one with no itemized deductions. Clients with state and local income taxes and property taxes may have an AMT problem because those itemized deductions are not allowed for AMT purposes. Note that interest on private-activity municipal bonds issued in 2009 and 2010 is not a preference item for the AMT.

Jeffrey and Karen have given cash gifts to their children over the years. In addition, in 2020 Mark, age 13, earns $2,500 in salary. Jennifer, age 19, who attends community college for approximately three months per year, earns $2,200 in dividends and capital gains. Nancy, age 12, earns $2,950 in dividends and interest. Steven, age 10, earns $900 in dividends and interest. Whose income is subject to the tax at the parents' marginal rate? A) Nancy's and Mark's B) Steven's C) Nancy's D) Jennifer's and Nancy's

ans c Nancy is the only child up to and including age 18 with unearned income in excess of $2,200 for 2020. Earned income is not subject to taxation at the parental rate. Jennifer is not subject because she is not a full-time student. The kiddie tax applies to children under 19 years of age. It also applies to children under age 24 if they are full-time students. The kiddie tax does not apply if the child's earned income exceeds one-half of the child's support. A full-time student is an individual who is a full-time student for at least five calendar months during the tax year.

Which of the following are techniques of income shifting or splitting? An installment sale of an income-producing asset from a parent to a child Transfer of income-producing property from the grantor to a grantor trust Valid employment of a child in the parent's business A) II and III B) I, II, and III C) I and III D) I and II

ans c Of the listed choices, only option II is not an example of income shifting or splitting. The transfer of property to a grantor trust, by definition, involves the income being taxed back to the grantor. Thus, there would be no income shifting or splitting.

Which of the following statements regarding alimony paid under a 2020 divorce agreement is CORRECT? A) Alimony may be paid in either cash or property. B) Alimony is deductible by the payor spouse, and includible in income by the payee spouse, to the extent that the payment is contingent on the status of the divorced couple's children. C) Alimony payments must terminate on the death of the payee spouse. D) The divorced couple may be members of the same household at the time the alimony is paid.

ans c Payments to former spouses are no longer deductible and are considered alimony only if the payments are made in cash (and not property); the decree does not specify that the payments are not alimony for federal income tax purposes; the payor and payee are not members of the same household at the time that the payments are made; and there is no liability to make the payments for any period after the death of the payee.

Max is a widower who provides a home for himself and his dependent six-year-old daughter, Lucy. He has hired an individual to pick his daughter up from school each day, bring her home, cook dinner, and perform some housekeeping services until he gets home four hours later. He pays $1,600 per month for the service. How will this affect Max's income tax return? Max may be entitled to a child and dependent care credit. Max qualifies to list Lucy on his income tax return as a dependent. Because Lucy attends school during the day, the child or dependent care credit is not available. Max must allocate the $1,600 per month between child care and housekeeping services. A) II only B) II, III, and IV C) I and II D) I and IV

ans c School attendance does not affect the availability of the credit. The $1,600 in expenses incurred each month to enable Max to work outside the home do not have to be divided between child care and housekeeping services.

Peter, a CPA, has a standard package of services for which he normally charges $600. Peter donated this package to a charitable auction. The package contains three hours of Peter's services for which the CPA normally charges $200 per hour. How much, if any, can Peter take as a charitable contribution deduction? A) $600 B) $400 C) $0 D) $200

ans c Services donated to charity are not deductible. Therefore, Peter cannot deduct any amount under the charitable contribution income tax rules

Now that their parents have died, Joseph is assuming the responsibility for the care of his 25-year-old disabled sister. Joseph has told his financial planner there is a trust for his sister to provide funds sufficient for her to live in an assisted-living facility or to reimburse him for the cost of an at home caregiver and his sister's other expenses if she stays in his home. He would prefer for his sister to live with him but does not want to cause her any problems with tax issues on any income she gets if she does. His sister is adamant that she does not want to be anyone's dependent. What advice can the planner give to Joseph? If his sister's income from the trust is paying for all of her needs, then she is not a dependent on anyone's income tax return. If Joseph's sister lives in an assisted-living facility, she cannot be a dependent of Joseph. A) II only B) Both I and II C) I only D) Neither I nor II

ans c Statement I is correct. If all of his sister's support is paid for by funds she receives from the trust, she cannot be a dependent of Joseph. Statement II is incorrect. Simply not living with Joseph does not preclude him from listing his sister as a dependent. If Joseph provided more than half of the support for his sister, he would be allowed to list her as a dependent and take any tax credits that may be available.

Which of the following would be a planning strategy to limit the imposition of the alternative minimum tax in a given tax year? Avoid purchasing most private-activity bonds. Move deductions into an AMT year. Move income into a non-AMT year. A) I and III B) II only C) I only D) II and III

ans c Statement I is correct. Taxpayers should time the recognition of certain AMT adjustments and tax preference items. For example, a taxpayer should avoid the purchase of most private-activity bonds unless the bonds were issued in 2009 and 2010. Deductions should be moved into a non-AMT year and income to an AMT year, if possible.

It is January and Baxter is meeting with his financial planner to discuss planning strategies for the AMT for the year. He will be including the bargain element for the incentive stock options he will be exercising this year in his AMTI. He current tax bracket is 32%. What does the planner tell Baxter he can do to try to minimize his AMT liability? Baxter should prepay as many itemized expenses, such as medical expenses, taxes, and other miscellaneous itemized deductions, to offset the ISO bargain element. Baxter should accelerate other items of income to this tax year. A) Neither I nor II B) Both I and II C) II only D) I only

ans c Statement I is incorrect. Moving deductions to an AMT year reduces regular taxable income and makes for a greater exposure to the AMT. Statement II is correct. Increasing regular income decreases the possibility that the AMTI will be greater than the regular taxable income.

Gary has just divorced. He is asking his planner, Ruth, for recommendations of amending his financial plan given his newly single status. Which of the following recommendations should Ruth make? Gary should review the beneficiary designations on his life insurance policies to be certain the beneficiaries are in line with his wishes post-divorce. Ruth should ask for any documentation on property settlements and other court-ordered financial transactions. Ruth should inquire whether a qualified domestic relations order was issued by the court and obtain a copy to ascertain its effect on Gary's financial plan. Ruth should inform Gary that the transfer-for-value income tax rule makes the transfer of any life insurance policies a taxable event. A) I and IV B) III only C) I, II, and III D) II and IV

ans c Statement IV is incorrect. The transfer-for-value income tax rule does not apply in situations when a life insurance policy is transferred from one spouse to another as a result of a property settlement.

Which of the following statements regarding the kiddie tax is CORRECT? The standard deduction for a child with both earned and unearned income is always earned income plus $350. The child's tax rate is 10% for all income received. The excess of unearned income above $2,200 is tax to the child at the parents' top marginal tax rate. The kiddie tax provision limits the effectiveness of income shifting, wherein families are prevented from transferring large amounts of unearned income to children and making the shift effective for income tax purposes. A) I, II, and III B) I only C) III and IV D) III only

ans c Statements I and II are incorrect. The standard deduction for a child with both earned and unearned income is the greater of $1,100 (2020) or earned income plus $350, but it is limited to the standard deduction for a single taxpayer, $12,400 in 2020. The child's tax rate is determined by the amount of taxable income the child has. Statements III and IV are correct.

Which of the following taxpayers may owe the additional Medicare tax in 2020? Brad and Jane file jointly and have combined wages of $288,000. Terry's only income in 2020 is from his investments and totals $290,000. Jack has earned $150,000 in compensation from his employment at Bland Foods Inc. Lisa, whose filing status is head of household, is self-employed and has self-employment income of $225,000. A) IV only B) I only C) I and IV D) I, II, and III

ans c Statements I and IV are correct. The additional Medicare tax rate is .9%. An individual is liable for the additional Medicare tax if the individual taxpayer's wages, other compensation, or self-employment income (combined with a spouse if filing as married filing jointly) exceeds the thresholds for the taxpayer's filing status of a combined income greater than $200,000 if single and $250,000 if married filing jointly.

Jena owns and operates a string of retail electronics stores with approximately $30 million of sales annually. Approximately 20% of her sales are with extended credit terms. What method of tax accounting is most appropriate for Jena's business? A) The installment sale method, to spread the gain over a longer time frame B) The hybrid method, because the business involves both inventory and service C) The accrual method, because inventory is such a large component of the business D) The cash method, because it provides flexibility in the timing of income and expenses

ans c The accrual method of accounting generally is mandatory when inventory constitutes a significant income-producing factor. Thus, the cash method is incorrect. The hybrid method is incorrect because there is no indication that service constitutes a significant portion of the business. Also, the installment sale method is not available for sales of inventory, or sales with revolving credit terms. With annual sales of $30 million, the accrual method exception does not apply. If she had annual sales of under $26 (2020) million, she could still use the cash method, even though she has inventory.

Kappa Corporation has the following items of income and expense: Taxable income: $310,000 Federal income tax paid: $80,000 Dividends paid: $20,000 Accumulated earnings and profits at the end of the preceding year: $90,000 Kappa is an engineering firm with 100% of the stock owned by its three employee-shareholders. The corporation cannot establish a valid business purpose for excess accumulations. How much accumulated earnings tax is payable by Kappa? A) $42,000 B) $46,000 C) $30,000 D) $34,000

ans c The accumulated earnings tax applies to corporate accumulated earnings in excess of $250,000, generally, for which there is no valid business reason for accumulating the funds. For a personal service corporation (PSC), the accumulations limit is $150,000. Engineering is one of the personal services specifically listed for determining a PSC. Expenses otherwise not allowed in computing the corporate income tax are allowed in computing the accumulated earnings tax, as is summarized here. (Note that the accumulated earnings credit of $60,000 is the difference between the $150,000 safe harbor and the $90,000 of accumulations at the beginning of the year.) The accumulated earnings tax rate is 20%, the highest rate that may apply to qualified dividends. Taxable income$310,000Federal income tax(80,000)Dividends paid(20,000)$210,000Less: accumulated earnings credit(60,000)Excess accumulations$150,000× 20%$30,000

Larry Gibson will have an adjusted gross income of $100,000 for the current tax year. His only charitable contribution during the year is a gift of a painting to the United Way. Larry had paid $40,000 for the painting several years ago, and the painting had a fair market value of $65,000 at the time of the gift. The painting is use-unrelated tangible personal property. What is Larry's charitable contribution deduction for the current tax year? A) $50,000 B) $65,000 C) $40,000 D) $30,000

ans c The answer is $40,000. Because this is use-unrelated property, Larry's charitable deduction is limited to his basis of $40,000.

Marion donated a truck to the local food bank to use for picking up food donations. Marion had purchased the truck several years ago for $15,000, and it currently has a value of $3,400. Which of the following statements regarding the documentation Marion must have to support his charitable contribution of the truck is CORRECT? A) A noncash contribution under $5,000 needs no documentation to support the donation. B) An appraisal must be attached to Marion's income tax return for the year of the donation. C) The documentation must have the description of the property, the name of the receiving charitable organization, the date of the contribution, and the amount of the donation. D) A letter from the food bank thanking him for the donation of the truck is sufficient documentation.

ans c The donor-taxpayer must have a canceled check, bank record, or a receipt from the donee organization to substantiate the deduction. The documentation must have the amount of cash or description of property, the name of the receiving charitable organization, the date of the contribution, and the amount of the donation. An appraisal is not required for noncash property over $500 and less than or equal to $5,000. However, taxpayers may wish to get an independent appraisal to support the deduction claimed.

Jim is planning to make a charitable contribution to a local university, a qualifying charitable organization. He is going to contribute a piece of real estate that he has owned for six years. The fair market value of the property is $80,000, and his basis in it is $55,000. He has an AGI of $120,000. Jim wants to maximize the amount of charitable contribution deductions from the donation of the real estate. What is the amount of charitable contribution deduction that Jim may claim in the current year? A) $55,000 B) $60,000 C) $36,000 D) $40,000

ans c The gift of long-term capital gain (LTCG) property is generally based on the fair market value of the property. The university is a 50% organization, a public charity. LTCG property contributed to a 50% organization involves a 30% of AGI limitation, and 30% of $120,000 is $36,000. There is also a $44,000 carryforward for up to five years. Jim could have made a 50% election to maximize the current-year deduction, but that would have reduced his overall deductions. If Jim had made a 50% election, he could have deducted $55,000 in the current year. By forgoing the 50% election, he is allowed to deduct the full $80,000 fair market value—$36,000 this year and $44,000 over the next several years.

Carla anticipates adjusted gross income of $90,000 during the current tax year. She is considering making a gift of real estate to the United Way. Carla's adjusted basis in this real estate is $20,000. The real estate has a current fair market value of $55,000. Carla has owned the real estate for three years. If Carla gifts the real estate to the United Way this year, what is the maximum allowable charitable deduction she can receive for the current tax year? A) $27,000 B) $55,000 C) $45,000 D) $20,000

ans c The gift of long-term capital gain property to a 50% organization is limited to 30% of AGI, which is $27,000. The deduction is based on the FMV of the property contributed. There would be a five-year carryforward of $28,000. The use-related or use-unrelated provisions do not apply here, as the distinction is relevant only with respect to tangible personalty, not realty.

Andrew and Olivia want to maximize their charitable deduction for this year. They are interested in several charities. The couple's AGI is $70,000. They have received an inheritance of $100,000 in cash they wish to use to further their charitable interests. What amount may the couple donate to maximize their charitable deduction this year, without using the carry forward option? A) $100,000 to a 50% charity B) $35,000 to either a 30% or 50% charity C) $42,000 to a 50% charity D) $21,000 to a 30% charity

ans c The maximum amount of charitable deduction the couple may take would be for a $42,000 donation to a 50% charity (allowing a donation of up to 60% of AGI for a cash donation in a given tax year).

John and Karen will spend $7,000 on day care for their two children (ages 9 and 10) in the current tax year. These expenses were incurred to allow both John and Karen to work outside the home. Their adjusted gross income is estimated at $138,000. What is the amount, if any, of child care credit to which they are entitled? A) $960 B) $1,400 C) $1,200 D) $480

ans c The maximum amount of qualifying expenditures on which the credit may be based is $3,000 per child, or $6,000 for two or more children. This is multiplied by 20% for taxpayers with an AGI greater than $43,000. Thus, $6,000 × 20% = $1,200.

Ruth and Doug divorced last year. They have two children ages seven and nine. Their divorce decree states that Ruth has custody of both children. There is no written agreement for listing the children as dependents on Ruth's or Doug's income tax returns. However, Doug provides 75% of the child support, amounting to $15,000 per year. Based on this information, which parent is entitled to list the children as dependents for income tax purposes? A) Doug, because he provides over one-half of the child support B) Doug, because he provides at least $1,200 per year for the children's support and Ruth cannot not prove she contributes more than this amount C) Ruth, because she has custody and there is no written agreement stating Doug could list the children on his return D) Ruth, because the court awarded her custody of the children

ans c The parent with custody for a greater portion of the year is treated as providing more than one-half of a child's support. In these circumstances, however, Ruth could potentially sign IRS Form 8332 which would constitute a written agreement that would allow Doug to claim their two children as dependents on his tax return in the current year.

Francine and Marshall have three children: Bill, Curt, and Rachel. For 2020 Bill, age 11, has $1,250 of interest income. Curt, age 13, has $2,950 of salary from a part-time job. Rachel, age 19 and not a full-time student, has $5,100 of dividends and capital gains. Whose income is subject to application of the parents' marginal rate? A) Bill and Rachel B) Curt C) None of the children D) Bill

ans c The parents' marginal rates apply to unearned income above $2,200 received by an individual under the age of 19 at the close of the tax year, if the individual is not a full-time student. Bill has unearned income under $2,200. Curt has earned income, so the parental rate does not apply. Rachel is 19 and is not a full-time student, therefore the parental rate no longer applies.

John, an accountant, is considering forming an S corporation for his practice. He will be the sole employee of the corporation. Which of the following statements accurately describes the income tax consequences of the proposed arrangement? A) The corporation would not be considered a PSC; therefore, the income would be subject to the graduated corporate income tax rates. B) The corporation would be considered a PSC, subject to graduated tax rates. C) The corporation would not be considered a personal service corporation (PSC), and the income would be subject to John's personal income tax rates. D) The corporation would be considered a PSC, subject to a flat 21% tax rate.

ans c The personal service corporation (PSC) classification for a C corporation was effectively repealed with the Tax Cuts and Jobs Act (TCJA). S corporations do not have corporate income tax rates. As an S corporation, all income would flow through to John to be taxed at his individual rates.

Bob and Bonnie were divorced in 2016. As a result of a court order, Bob pays Bonnie $700 per month in alimony. He makes each month's payment with a money order. Earlier this year, Bonnie moved in with Bob, and they now share a two-bedroom apartment. Which of the following statements is accurate concerning the alimony payments by Bob? A) The payments are deductible because they are being made as a result of a court order. B) The payments are not deductible because the payments must be in cash. C) The payments are not deductible because the taxpayers are living together at the time of payment. D) The payments are deductible because they are equal each year of the agreement.

ans c The requirement that the payments be in cash means that the payments may not be in the form of property. Payments are not necessarily deductible just because they are a result of a court order; there are other requirements that must be met as well. One of those requirements to have qualifying alimony is that the taxpayers must not be living together at the time of payment. The payments do not need to be equal each year. However, unequal and declining payments can trigger the alimony recapture (excess front-loading) rules.

Which of the following children have income subject to federal income tax at the parent's marginal tax rate 2020? Brittney, age 16, earned $5,000 in salary. Kate, age 14, received $2,800 in mutual fund dividends. Tony, age 5, received $951 in savings account interest. Amanda, age 19, a part-time college student, received $2,400 in dividends and interest. A) Kate, Tony, and Amanda B) Brittney C) Kate D) Kate and Tony

ans c This question involves application of the kiddie tax, which applies to unearned income in excess of $1,100 (for 2020) by a child under the age of 19 before the close of the tax year or a full-time student who has not attained age 24 before the close of the taxable year. Brittney, although under age 18, has only earned income. Amanda is age 19 and only a part-time college student, so she is not subject to the kiddie tax. Finally, Tony, age 5, has unearned income, but it is less than $1,100. Only Kate is subject to the tax.

Which of the following methods of accounting is mandatory for businesses that maintain inventory and have gross receipts in excess of $26 million? A) Hybrid method of accounting B) IRS method of accounting C) Accrual method of accounting D) Cash method of accounting

ans c Under the cash method of accounting, a taxpayer generally reports income when any cash is collected (or the constructive receipt income tax doctrine applies) and reports expenses when any cash payment is made. The accrual method is the conceptual opposite of the cash method and requires recognition of taxable income in the same tax year it is reported on the taxpayer's financial statements; it is mandatory for businesses maintaining inventory and have gross receipts in excess of $26 million. The hybrid method of accounting is a combination of the accrual method and the cash method of accounting. There is no IRS method of accounting.

Alternative minimum taxable income (AMTI) is calculated using which of the following methods? A) Modified adjusted gross income (from IRS Form 1040) + positive AMT adjustments − negative AMT adjustments + AMT preference items = AMTI B) Regular taxable income (from IRS Form 1040) − positive AMT adjustments + negative AMT adjustments + AMT preference items = AMTI C) Regular taxable income (from IRS Form 1040) + positive AMT adjustments − negative AMT adjustments + AMT preference items = AMTI D) Adjusted gross income (from IRS Form 1040) + positive AMT adjustments − negative AMT adjustments + AMT preference items = AMTI

ans c regular taxable income (from IRS Form 1040) + positive AMT adjustments − negative AMT adjustments + AMT preference items = AMTI

During 2020, Judy, a sole proprietor, purchased new equipment (seven-year property) for her manufacturing business at a cost of $600,000. Judy is in a 12% marginal income tax bracket this year, and expects to be in that bracket for two more years. She is extremely confident that she will be in the highest marginal bracket after that. What advice would you give Judy regarding the use of bonus depreciation and cost recovery deductions? A) Use the maximum bonus depreciation and use the Modified Accelerated Cost Recovery System (MACRS) table. B) Use the maximum bonus depreciation and elect the straight-line method. C) Forgo bonus depreciation and use the Modified Accelerated Cost Recovery System (MACRS) table. D) Forgo bonus depreciation and elect the straight-line method. Explanation The fact pattern indicates that Judy is in the lowest marginal bracket for three years, and will be in the highest marginal bracket after that. It makes no sense to maximize the depreciation deduction in years when Judy is in the lowest marginal brackets. By forgoing bonus depreciation and using straight-line, more deductions are pushed into the last five years of the depreciation schedule, when Judy will be in the highest marginal bracket. Remember that because of the half-year convention, seven-year property is depreciated over eight years. Under TCJA, 100% bonus depreciation is allowed for all personalty. In other words, 100% of the cost is deducted in the first year.

ans d

During the current tax year, Jim purchased a fully finished small warehouse for exclusive use in his manufacturing business. The cost of the property was $122,000, of which $32,000 was attributable to the land. Which of the following statements identifies the proper treatment of the expenditure? A) The $90,000 attributable to the building may be deducted under Section 179. B) The entire $122,000 is currently deductible. C) The $90,000 attributable to the building may be currently deductible. D) The $32,000 must be capitalized and may not be depreciated. Explanation The cost associated with the land must be capitalized (establishes basis) and may not be depreciated. Only so-called wasting assets may be depreciated; thus, the land is not depreciable. Because the warehouse has a useful life of greater than one year, it must be depreciated. Section 179 is not an option for the warehouse, because Section 179 generally applies primarily to personalty, not realty.

ans d

Eric purchased a painting for $200,000. He sold the artwork 5 years later for $500,000 in an installment sale, receiving $50,000 as the first-year payment. How much gain must Eric recognize in the year of sale? A) $500,000 B) $20,000 C) $50,000 D) $30,000 Explanation Eric would recognize a capital gain of $30,000 in the year of sale, calculated as follows: Gross profit percentage (60%) = profit ($300,000) ÷ total contract price ($500,000). Gain recognized ($30,000) = gross profit percentage (60%) × installment payment ($50,000).

ans d

Erma, age 40, anticipates an adjusted gross income of $177,000 for the current tax year. All her income is attributable to active and portfolio income. She would like to acquire an investment that would reduce her tax liability without exposing her to personal liability. Which one of the following investments is the most appropriate for Erma? A) An oil and gas working interest that will produce losses B) An "active participation" investment in rental real estate that will produce losses C) A master limited partnership that will produce passive losses D) A historic rehabilitation real estate limited partnership that will produce rehabilitation tax credits Explanation Erma's AGI is too high to claim the active participation real estate exception. Losses from a master limited partnership can only be deducted against income generated by the same partnership in another tax year. The oil and gas working interest will generate unlimited liability. Thus by process of elimination, the correct answer is historic rehabilitation credit.

ans d

Frank, a single taxpayer, owned a warehouse that he rented as commercial property. He acquired the property several years ago for $196,000. He used the straight-line method of cost recovery, which totaled $35,000. Frank sold the property in February of the current year for $230,000. Frank is single, and has taxable income (not including the real estate gain) of $475,000. What is the amount and nature of the gain on the sale? A) $34,000 Section 1231 gain; $35,000 ordinary income B) $7,000 ordinary income C) $69,000 ordinary income D) $35,000 unrecaptured Section 1250 gain; $34,000 long-term capital gain Explanation The entire gain of $69,000 is treated as Section 1231 gain, because there is no excess depreciation on the use of the straight-line method. So, $35,000 of the gain is subject to a maximum rate of 25%, as unrecaptured Section 1250 income, and the remaining $34,000 of gain is subject to the maximum regular long-term rate of 20%. The 20% long-term capital gain rate applies, as his taxable income is over the $425,800 breakpoint for the 20% rate. Note that Section 1250 recapture (ordinary income treatment) applies only to excess depreciation—in other words, the excess of an accelerated method over what would have been deducted if straight-line had been used. All realty placed in service after 1986 is depreciated using straight-line, and there is NO recapture (ordinary income) where straight-line depreciation was used.

ans d

How can passive activity losses from an ongoing nonpublicly traded partnership, such as a RELP, be deductible from other taxable income? A) Passive activity losses are deductible against portfolio gains. B) Passive losses from one partnership can only be offset by passive gains from the same partnership. C) Passive losses can only be carried forward against future passive gains. D) Passive losses can offset passive gains. Explanation Generally, passive activity losses can only be used to offset passive activity income. Losses from nonpublicly traded partnerships (non-PTP) can be used to offset gains from other nonpublicly traded partnerships. Losses from master limited partnerships can only offset gains from the same master limited partnership. Losses from a non-PTP may be carried forward, but it is not a requirement. Passive losses cannot be used to offset portfolio income.

ans d

If a vacation home is rented for 14 days or less during the year, which one of the following statements is CORRECT? A) Repair expenses attributed to the rental activity are deductible. B) Typically, only a small amount of cost recovery deductions is allowed for the year. C) A portion of the rental income may be nontaxable. D) The full amount of home mortgage interest is permitted as an itemized deduction. Explanation If property is rented fewer than 15 days per year, the full amount of home mortgage interest, taxes, and casualty losses are permitted as an itemized deduction (not expenses, though); in addition, rental income can be excluded from gross income.

ans d

If the Taylors rented their condo for 300 days last year, how many days could the Taylors have used it personally and still maintained the classification of the condo as primarily a rental use property? A) 0 days B) 14 days C) 3 days D) 30 days Explanation The Taylors could have used the property personally for a total of 30 days (300 rental days × 10%). If rental property is rented at least 15 days a year and is not used for personal use more than the greater of 14 days per year or 10% of the rental days, it is classified as primarily rental use. This treatment permits the Taylors to deduct expenses associated with the rental (not personal) use on Schedule E of IRS Form 1040.

ans d

In June of the current year, Mindy sold her principal residence for a total price of $185,000—she received $100,000 in cash and the buyer assumed an $85,000 mortgage on the house. Mindy purchased the house six years ago for $120,000 and has made $80,000 in improvements to the house. Real estate commissions of $9,200 resulted from the sale. What amount of gain or loss, if any, must be recognized on the sale of Mindy's residence? A) $35,800 B) ($24,000) C) ($15,000) D) $0 Explanation This is computed as follows: Gain realized:Amount realized:Cash$100,000Mortgage assumed by buyer85,000Selling expenses(9,200)Total amount realized$175,800Less adjusted basis ($120,000 + $80,000)(200,000)Loss realized(24,200)If a loss is realized on the sale of a principal residence, it is not deductible (recognized).

ans d

In March of the current year, Susan sold her principal residence for $408,000. Susan sold the house due to a job transfer out of state. Susan received the house as a gift 18 months ago when its fair market value was $375,000, and the donor's basis was $50,000. She made $20,000 of improvements to the house. She paid real estate broker commissions of $23,000. What amount, if any, must be recognized on the sale of Susan's residence? A) $85,000 B) $10,000 C) $65,000 D) $127,500 Explanation $127,500 is the correct answer, and is computed as follows. Note: We use the donor's basis of $50,000 as Susan's basis, because the fair market value on the date of the gift was greater than the donor's basis. The $50,000 was increased by the $20,000 of improvements to equal the $70,000 basis. Gain realized:Amount realized: Sale price $408,000 Less selling expenses (23,000)Total amount realized$385,000Less adjusted basis(70,000)Gain realized$315,000Gain recognized:Gain realized$315,000Less allowable exclusion(187,500)Gain recognized$127,500The allowable exclusion is the full exclusion of $250,000 multiplied by 75%—18 months of use divided by the 24-month requirement.

ans d

In the current tax year, Fay has short-term capital loss carryovers of $5,000 and long-term capital loss carryovers of $40,000, both carried over from the previous year. Her net short-term gain for this year is $6,000, and her net long-term gain for this year is $5,000. How much of her gain for this year will be taxable? A) $6,000 B) $5,000 C) $1,000 D) $0 Explanation Fay can apply her short-term capital loss carryover to all current short-term capital gains, which results in a net short-term capital gain for this year of $1,000 ($6,000 gain − $5,000 carryover). She is then left with a net long-term capital loss of $35,000 ($5,000 gain − $40,000 carryover). To calculate net capital gains for the year, aggregate the long-term and short-term gains or losses, which in this case equals $35,000 long-term loss − $1,000 short-term gain, or a $34,000 net capital loss. She has no net gain and, as such, pays no taxes on any of the capital transactions she made this year.

ans d

Jean leases a house from Nan. The rent is normally $850 per month. Nan accepts reduced monthly rent of $500 per month for the current year in addition to Jean providing repairs to the property. The repairs are worth $2,500 and cost $3,200. How much must Nan include in income due to the house rental? A) $6,000 B) $10,200 C) $9,200 D) $8,500 Explanation The actual rent received of $6,000 plus the fair market value of the services provided, $2,500, are included in income.

ans d

Joe has $60,000 in wages, $13,000 in deductions, and $5,000 in passive losses this year. The passive loss did not arise from rental property or real estate. What is Joe's taxable income this year? A) $48,000 B) $44,000 C) $60,000 D) $47,000 Explanation Joe's taxable income is $47,000 ($60,000 − $13,000). Passive losses may only offset passive income so his $5,000 loss is not allowed, except for rental and real estate property. Rental and real estate passive losses are allowed up to $25,000 and are subject to the AGI phase out limitations.

ans d

Kurt anticipates adjusted gross income of $100,000 for the current tax year. He is considering making a gift of appreciated stock to his alma mater, Regis University. His basis in this stock is $48,000. The stock has a current fair market value of $60,000. Kurt has owned the stock for four years. If Kurt gifts the stock to Regis, what is the maximum allowable charitable deduction that Kurt can receive in the current tax year? A) $60,000 B) $50,000 C) $30,000 D) $48,000 Explanation The deduction for a donation of long-term capital gain property to a 50% organization typically is restricted to 30% of AGI, with the deduction based on the fair market value of the asset. However, in this case, the 50% election, using the basis of the property, results in a larger current-year deduction. If the 50% election is made, the deduction amount may be up to 50% of AGI, but the basis of the property must be used.

ans d

Lana filed her current income tax return three months late. The return showed a balance due of $10,000. What is Lana's penalty, if any, for late filing of her income tax return? A) $2,000 B) $150 C) $0 D) $1,500 Explanation The penalty for failing to file an income tax return is 5% of the amount due for each month, or part thereof, that the return is late. Lana has filed her return three months late, which results in a 15% penalty for late filing. In this case, $10,000 × 15% (5% per month for three months) results in a penalty of $1,500.

ans d

Marcus purchased a diamond ring for $15,000 10 years ago. It was stolen in March this year. The ring was purchased to celebrate achieving a significant promotion at work. The FMV at the time of the theft was $20,000. The ring was insured, and after the deductible, Marcus received $19,000 from the insurance company. Marcus replaced the ring with a new one for $20,000. Under Section 1033, what is Marcus's new basis in the replacement ring? A) $19,000 B) $15,000 C) $20,000 D) $16,000 Explanation Marcus's deferred gain on the new ring is $4,000. His new basis is the FMV of the property at acquisition minus the deferred gain ($20,000 − $4,000 = $16,000).

ans d

Martha and Max own Louisiana Properties, in which they manage their rental investment properties, both commercial and residential. Martha spends around 1,800 hours annually actively managing the business and it is her only business activity. Max spends 1,500 hours annually managing the business, usually during their summer and holiday rental seasons, and it is 70% of his business activity for the year. Which of the partners can fully deduct any real estate losses against active and/or portfolio income? Martha Max A) Neither I nor II B) I only C) II only D) Both I and II Explanation Both Martha and Max are considered real estate professionals because the activity comprises more than 50% of their personal services and they both participate in more than 750 hours annually. As real estate professionals, the partners may deduct any loss against active and/or portfolio income.

ans d

Michelle has interest and short-term capital gain income of $9,000 during the current tax year. She paid brokers' commissions of $1,000, investment advisers' fees of $2,200, and had $7,700 of interest expense on funds borrowed to purchase securities. Michelle has an AGI of $105,000. What amount of investment interest expense may be deducted as an itemized deduction? A) $5,800 B) $8,200 C) $6,700 D) $7,700 Explanation The investment interest expense ($7,700) is deductible up to the amount of the net investment income. The net investment income is simply the investment income of $9,000. Investment interest expense may be deducted up to the amount of investment income—$7,700 in this situation. The broker's commissions do not enter into this calculation, nor do the advisor's fees. They are not a deductible expense, and they are not part of the investment income.

ans d

Pam is considering renting out her vacation home in Vail for two weeks during ski season. She lives in the home for approximately two months of the year. She has asked you to explain the income tax consequences. Which one of the following statements is CORRECT? A) The rental income may or may not be includible in income depending on the amount. B) The rental income is includible in full in gross income, and the mortgage interest and property tax are 2/52 deductible for AGI. C) The rental income is includible in income, but mortgage interest and property taxes allocable to the rental are not deductible for AGI. D) The rental income is not includible in income, and the mortgage interest and property taxes are fully deductible as itemized deductions. Explanation Rentals for 14 days or fewer (fewer than 15 days) during the year are not required to be included in gross income. However, no rental deductions are allowed either. The mortgage interest and taxes are fully allowed as itemized deductions because the residence test is met.

ans d

Policyholder dividends from a whole-life insurance policy are generally tax exempt. In which of these situations would the policyholder dividend be tax exempt? A) The dividend is from a MEC and is received in cash. B) The dividend is from a MEC and is used to pay a loan. C) The policyholder receives dividends greater than the investment in the contract. D) The policyholder receives dividends that are less than the investment in the contract. Explanation The dividend from a life insurance policy is typically tax exempt. However, the dividend is taxable to the extent that the dividends received exceed the investment in the contract. The dividend is also taxable if it is from a MEC and received in cash, or is used to pay a loan. LO 2.2.2

ans d

Rami, an Augusta, Georgia taxpayer, is considering renting out his home for one week (seven days) during a golf tournament in the summer. He is unsure of the income tax consequences related to this rental income. Which of these statements is CORRECT? A) The rental income is includible in full in gross income. B) The rental income is includible in income, but mortgage interest and property taxes allocable to the rental are deductible for AGI. C) The rental income may or may not be includible in income, depending on the amount. D) The rental income is not includible in income. Explanation Rentals for 14 days or less (fewer than 15 days) during the year are not required to be included in gross income. However, no deductions specific to the rental are allowed either. The mortgage interest and property tax deductions are unaffected—they are still deductible as itemized deductions.

ans d

Ron, age 43, and Sandy, age 41, are married with two children, Michael, age 12, and Victoria, age 8, who has been blind since her birth. Ron is an architect and general partner with XYZ partnership. Sandy is self-employed as an attorney and works out of a home office. Her home office is exclusively and regularly used for business, and the home office is her principal place of business. Their information for the tax year 2020 is as follows: Adjusted gross income: $217,300 Itemized deductions (including qualified residential mortgage interest, taxes paid, and charitable contributions): $33,000 Early in the current year, Sandy's father died. Sandy is the sole beneficiary of her father's entire estate. The estate is presently in the probate process. Sandy's mother, Lisa, age 68, has moved in with them but provides her own support. She was married to Sandy's father when he died earlier this year. This is Ron's second marriage. He makes monthly support payments to his former wife and his daughter. Because both Ron and Sandy are considered to be self-employed, they make quarterly estimated tax payments each year to cover both their income tax and self-employment tax obligations. Ron's investment in his XYZ partnership interest on December 31, 2018, was $12,000. As the result of a serious downturn in business, the partnership calculates Ron's share of the partnership's losses for 2020 will be $14,500. How much of this projected 2020 loss, if any, may Ron deduct on his income tax return for 2020? A) $0 B) $14,000 C) $2,500 D) $12,000 Explanation Because this is not a passive activity, he may deduct (under the at-risk limitations) his share of the partnership loss to the extent of his investment in the partnership assuming he has at least that much in other income to offset the loss ($12,000).

ans d

Sally is a waitress who makes a significant amount of her income from tips. During the current year, Sally willfully underreported by 50% the amount of tips that she received. Which of these penalties, potentially applicable to Sally, would be the most costly? A) Late payment B) Underpayment of estimated tax C) Negligence D) Civil fraud Explanation The penalty for civil fraud is 75% of the amount underreported due to fraud by the taxpayer. Because she willfully underreported her income, the civil fraud penalty is likely to apply. This is considerably more than the other penalties, which are as follows: PenaltyConsequenceUnderpayment of estimated taxBased on the applicable federal rate (currently, approximately 5%/year of the underpayment)Late filing5% per month up to 25% maximumLate payment0.5% per month up to 25% maximumNegligence20% of the underpayment amount

ans d

Sarah has two dependent children who attend Sun Valley Day Care while she is at work. She will claim a $1,200 credit for child and dependent care expenses in the current tax year. What amount of deduction would be necessary to provide a tax benefit that is equal to that provided by the child care credit if Sarah is in the 24% marginal income tax bracket? A) $892 B) $1,579 C) $288 D) $5,000 Explanation It would take $5,000 of deductions to equal the benefit of a $1,200 tax credit. This is determined by simply dividing the $1,200 of credit (savings) by the marginal income tax bracket of 24%.

ans d

Susan received 100 shares of stock as a gift from her uncle, Carl. Carl purchased the stock 15 years ago for $12 per share. Susan received the stock from Carl two months ago, when the fair market value of the stock was $15 per share, and she sold the stock this week for $19 per share. What is the amount and character of Susan's gain from the sale of the stock? A) $400 short-term capital gain B) $700 short-term capital gain C) $400 long-term capital gain D) $700 long-term capital gain Explanation In the case of an asset received as a gift, where the fair market value on the date of the gift is greater than the donor's adjusted basis, the recipient has a carryover basis. In this case, Uncle Carl had purchased the stock for $12 per share and had gifted it to Susan when the fair market value was $15 per share. Susan subsequently sold the stock for $19 per share. Thus, the carryover basis from Uncle Carl would be $12 per share. In a situation where the recipient of the gift takes the donor's basis, the holding period is tacked. In other words, the donor's holding period is added to the donee's holding period. Thus, Susan is treated as holding the stock for over 15 years.

ans d

Three years ago, Lydia purchased specialized manufacturing equipment (seven-year property) at a cost of $66,000. She paid an additional $4,000 to have the equipment installed in her plant. She used the straight-line method, and cost recovery deductions total $30,000. Earlier this year, Lydia sold the equipment for $25,000. What is the amount and character of the gain or loss resulting from this sale? A) $15,000 Section 1245 loss, treated as an ordinary loss B) $11,000 Section 1231 loss, treated as an ordinary loss C) $15,000 Section 1231 loss, treated as a capital loss D) $15,000 Section 1231 loss, treated as an ordinary loss Explanation The loss realized and recognized is the difference between the $25,000 amount realized from the sale and the adjusted basis of $40,000. The adjusted basis is the $70,000 cost basis, reduced by the $30,000 of depreciation deductions taken. Thus, the loss is $15,000. The Section 1231 loss is treated as an ordinary loss, deductible in full against any other income. Remember that the installation is a cost associated with the acquisition of an asset and must be capitalized. Thus, the original cost basis was $70,000. There is no such thing as a Section 1245 loss.

ans d

Which of the following applies to the at-risk rules, as related to passive loss restrictions for partners? It is the maximum deductible loss for an investment limited to the amount of risk that the taxpayer has at the end of the current year. Determining the amount at risk includes the adjusted basis of other property contributed to the partnership. The inclusion of nonrecourse financing is essentially the only difference between the basis in a partnership and the amount at risk. A) II only B) I and II C) I only D) I, II and III Explanation In the Tax Code, the at-risk rules are defined as the maximum deductible loss for an investment limited to the amount that the taxpayer-investor has at risk at the end of the current year (i.e., the amount of potential economic loss). A partner may deduct losses only to the extent of the amount that they have "at risk." The amount at risk equals the sum of the following: The money invested (except to the extent the money invested was borrowed and was secured only by the investment) The adjusted basis of other property contributed to the partnership Amounts borrowed for use in the activity, but only to the extent that the partners are personally liable for repayment of the debt (recourse indebtedness) The partner's share of income, less the partner's share of losses or withdrawals from the partnership The proportionate share of qualified nonrecourse financing in a real estate activity ONLY Essentially, the only difference between the amount at risk and the basis in a partnership interest is the treatment of nonrecourse financing.

ans d

Which of the following are characteristics of a C corporation but are NOT characteristics of a sole proprietorship? Limited personal liability Tax-free formation Perpetual life Separate taxable entity A) II and III B) I, II, and IV C) I and IV D) I, III, and IV Explanation The C corporation involves limited personal liability for the shareholders, potential tax-free formation, and perpetual life, and is clearly a separate taxable entity. The sole proprietorship enjoys potential tax-free formation. The other three options are not characteristics of a sole proprietorship. The sole proprietor has unlimited personal liability. The sole proprietorship does not have perpetual life, nor is it a separate taxable entity.

ans d

Which of the following are requirements in order for a taxpayer who materially participates in a real property trade or business to be able to deduct any losses from the business? More than 50% of the individual's personal services during the tax year are performed in the real property trades or businesses in which the individual materially participates. More than 10% of the individual's net assets at the end of the tax year are invested in the real property trades or businesses in which the individual materially participates. The individual performs more than 750 hours of service in the real property trades or businesses in which the individual materially participates. A) I and II B) II and III C) I only D) I and III Explanation Options I and III are specific requirements that must be met in order for a taxpayer to able to deduct any losses from a real estate trade or business. The amount of the individual's net assets used in the business is not relevant.

ans d

Which of the following may enable a direct participation program to provide specific tax advantages to the investors? A) At-risk rules B) Partnership basis rules C) Passive activity loss rules D) Special allocations Explanation The benefits that flow through from a partnership entity may be enhanced by the potential, under certain circumstances, "special allocation" of certain items of income, expense, gain, or loss. The passive activity loss rules state that passive losses may only be deducted against passive income (not a tax advantage); there are no partnership basis rules; and the at-risk rules are defined as the maximum deductible loss for an investment limited to the amount that the taxpayer-investor has at risk at the end of the current year. None of the other answers are direct participation programs, which concern business organizations that function as tax conduits.

ans d

Which of the following statements describes the constructive receipt doctrine? Darrell was issued notification in December of this year that his bonus for the current year would be $10,000 and the check would be issued in January next year. Martha was notified that a check would be issued to her on December 31 for royalties on her song used in a local commercial; the check was available for pick up that day. A) I only B) Neither I nor II C) Both I and II D) II only Explanation Darrell was only notified in December of the amount of the bonus that would be paid. No check was issued and the funds were not available until January of the following year. The check to Martha constitutes constructive receipt, whether or not she picks up the check in December.

ans d

Which of the following statements regarding replacement property in a Section 1033 exchange is CORRECT? In the taxpayer use test, the taxpayer's use of the replacement property and of the involuntarily converted property must be the same. For the functional use test, the owner-investor's properties must be used in similar endeavors as the previously held properties. A) Both I and II B) II only C) I only D) Neither I nor II Explanation Neither statement is correct. In the functional use test, the taxpayer's use of the replacement property and of the involuntarily converted property must be the same. In the taxpayer use test, the owner-investor's properties must be used in similar endeavors as the previously held properties.

ans d

Which of the following statements regarding the disposition of passive activities with suspended losses is NOT correct? After using losses to offset gain on the sale of the activity, any remaining losses are still passive losses. Suspended losses from the activity are first offset against any gain on the sale of the activity. A) Both I and II B) Neither I nor II C) II only D) I only Explanation Statement II is a correct statement. Statement I is incorrect. After using losses to offset gain on the sale of the activity, any remaining losses are classified as nonpassive losses.

ans d

Which of these below-market-interest loans would result in imputed interest to the lender? An employee borrows $12,000 from his employer, New Media, Inc., to pay medical bills, and the employee has investment income of $330 in the same year. John lends his friend, Mel, $8,000 to buy a boat, and Mel has investment income of $2,200 for the year. Faith borrows $120,000 from her father for an addition to her home. Faith has $4,400 of investment income in the same year. A) I, II, and III B) I and II C) III only D) I and III Explanation Statement I describes a compensation-related loan between an employer-lender and an employee-borrower. Because the loan is in excess of $10,000 and is between a corporation and an individual, it is not eligible for treatment as a loan between individuals would be. New Media, Inc. will have interest income and compensation expense in the amount of the imputed interest. In Statement III, Faith's father will have imputed interest income because the loan is in excess of $100,000 and Faith's investment income for the year exceeds $1,000. In Statement II, the loan is not in excess of $10,000, so no interest is imputed.

ans d

Which of these best describes the role of the at-risk rules? A) They limit the ability to use special allocations. B) They require the substantial economic effect doctrine to be satisfied. C) They allow the IRS to tax certain partnerships as if they were corporations. D) They limit the advantage gained through leverage. Explanation The at-risk rule limits the deductibility of losses to the amount at risk. Only certain debt establishes an amount at risk. Specifically, recourse financing and qualified nonrecourse financing (in a real estate activity only) establish amounts at risk. Conceptually, at risk is very similar to basis. A taxpayer may only deduct losses to the extent that he is at risk. The primary difference between at risk and basis is that basis includes all nonrecourse financing.

ans d

Which of these is CORRECT regarding the Lifetime Learning credit? A) Qualifying expenses include tuition, books, supplies, and room and board. B) The credit is equal to 100% of the first $2,000 and 25% of the next $2,000 of qualifying higher education expenses. C) The credit applies during the first four years of postsecondary school. D) There is a phaseout between $118,000 and $138,000 of AGI for married taxpayers filing jointly. Explanation Qualifying expenses generally include only tuition. Amounts paid for books and supplies may be included only if required to be paid to the education institution as a condition of enrollment. The credit is available annually for an unlimited number of years. The credit is equal to 20% of qualified tuition expenses up to $10,000. All of the listed options, except for the phaseout limits, accurately describe the American Opportunity tax credit.

ans d

Which one of the following best describes the role of a special allocation in a limited partnership? A) It requires all items to be distributed pro rata based on a partner's capital account balance. B) It allocates management responsibility to the general partners. C) It establishes the standards for allocating the proceeds of non-routine or "special" items of income. D) It allows an allocation of items of income and expense that is not pro rata. Explanation A special allocation allows an allocation of items in a manner that differs from the "normal" pro rata allocation of deduction, income, credit, etc.

ans d

Which one of the following statements is CORRECT with respect to capital gains and losses? A) Net capital gains are always taxed at a maximum rate of 28%. B) Excess capital losses are carried forward for up to five years. C) Net capital gains are always taxed at a flat rate of 15%. D) Net capital losses are deductible up to $3,000 annually.

ans d

Which one of the following types of audits is conducted on a random basis? A) Discriminant Functions System Program audit B) Document Matching Program audit C) Targeted Program audit D) National Research Program audit Explanation The National Research Program audit is the only random audit that the IRS conducts. The NRP is, in effect, the replacement for the old Taxpayer Compliance Measurement Program audit. The Discriminant Functions System Program (DIF) compares the information on each return to a set of norms, weighs each item, and ranks the return for audit-worthiness. The Document Matching Program allows the IRS to detect discrepancies between the amounts reported on a tax return and the amounts shown on information documents (e.g., W-2 forms, 1099 forms). Under the Targeted Program Audit, the IRS "targets" particular taxpayers in particular situations.

ans d

Your client is contemplating the exchange of two parcels of investment land for two similar parcels. Given the following details of the proposed transactions, compute the amount of recognized gain and loss, if any, on both parcels if your client does the exchanges. Parcel A: There were 10 acres of land acquired 15 years ago with a current basis of $50,000. In exchange, your client will receive 8 acres of land (FMV $80,000) and $20,000 of cash. Parcel B: There were 20 acres of land acquired 2 years ago with a current basis of $100,000. In exchange, your client will receive 12 acres of land (FMV $75,000) and $10,000 of cash. A) Parcel A recognized gain: $20,000; Parcel B recognized loss: $15,000 B) Parcel A recognized gain: $20,000; Parcel B recognized loss: $10,000 C) Parcel A recognized gain: $50,000; Parcel B recognized loss: $10,000 D) Parcel A recognized gain: $20,000; Parcel B recognized loss: $0 Explanation The realized gain in Parcel A is $50,000 and the recognized gain (the lesser of the gain realized or the boot received) is $20,000. The realized loss in Parcel B is $15,000. However, there is no loss recognized (deducted) in a like-kind exchange.

ans d

Your client, Sally, is considering investing in real estate as a way to diversify her investments. She has heard of active participation rental real estate, but is unsure of the requirements that must be met. What can you accurately tell her with respect to active participation rental real estate? The interest may not be held through a limited partnership. A deduction-equivalent tax credit of up to $25,000 is available. The taxpayer must hold a 10% or greater ownership interest. The taxpayer must make the major management decisions. A) I and II B) I and IV C) III and IV D) I, III, and IV Explanation The deduction for active participation rental real estate requires that the taxpayer participate in management in a bona fide sense; making the major management decisions. To qualify, the interest may not be held through a limited partnership, and the taxpayer must have at least a 10% or greater ownership interest in the property. The loss is deductible when computing the gross income. Active participation rental real estate generates an above-the-line deduction of up to $25,000 annually, not a credit.

ans d

Which of the following statements is CORRECT regarding a minority non-employee shareholder in an S corporation? Reports income when the corporation has net income for a tax year Votes for the board of directors at the annual shareholders meeting Receives a K-1 annually to prepare a personal income tax return Reports on a personal income tax return a pro rata share of corporate profit or loss A) I and III B) I, II, and III C) II and IV D) I, II, III, and IV

ans d All of the statements are correct. The fact that the shareholder has a minority interest in the corporation has no bearing on the correct answers. The flow-through of a proportionate share of net income is reported on the K-1. The shareholder is allowed to vote for the board.

Several years ago, Allison Colbert purchased a deferred fixed annuity. The cost of the annuity was a single payment of $40,000. The annuity will provide monthly payments of $275. At the time the annuitized distributions are to begin, Allison's life expectancy will be 25 years. How much of each payment will be excluded from taxation? A) $57 B) $142 C) $206 D) $133

ans d Allison is expected to receive $82,500 ($275 × 12 × 25). Her investment in the contract ($40,000) is then divided by the total expected return ($82,500) to determine the excludable portion of each payment. The exclusion ratio is the $40,000 divided by $82,500, which equals 48.48%. 48.48% of $275 = $133 excludable from each payment.

As part of their divorce decree, Judy, age 44, was forced to split her IRA with Alex, age 36. Alex received a check in April for $100,000 from the IRA custodian. Alex put the proceeds into a one-year CD account at the local bank. As a result, Alex will A) have to pay ordinary income tax on the $100,000 plus a 10% early withdrawal penalty. B) not have to pay income tax, but Judy will owe gift tax. C) have to pay ordinary income tax on the $100,000. D) not have to pay ordinary income tax on the $100,000.

ans d Although the transfer itself is tax free, Alex should roll the assets distributed from Judy's IRA within 60 days into his own IRA or retirement plan. Because he failed to do so, he owes ordinary income tax on the entire distribution. When incident to a divorce, domestic relations orders (DROs) are however not subject to gift tax or early withdrawal penalties.

Ron, age 43, and Sandy, age 41, are married with two children: Michael, age 12, and Victoria, age 8, who has been blind since her birth. Ron is an architect and general partner with XYZ partnership. Sandy is self-employed as an attorney and works out of a home office. Her home office is exclusively and regularly used for business, and the home office is her principal place of business. Their information for the tax year 2020 is as follows: Adjusted gross income: $217,300 Itemized deductions (including qualified residential mortgage interest, taxes paid, and charitable contributions): $33,000 Early in the current year, Sandy's father died. Sandy is the sole beneficiary of her father's entire estate. The estate is presently in the probate process. Sandy's mother, Lisa, age 68, has moved in with them but provides her own support. She was married to Sandy's father when he died earlier this year. This is Ron's second marriage. He makes monthly support payments to his former spouse and his daughter. Because both Ron and Sandy are considered to be self-employed, they make quarterly estimated tax payments each year to cover both their income tax and self-employment tax obligations. Ron gave a painting to the local art museum in the current year. The painting had a fair market value of $34,000. He paid $16,500 for it five months ago. The museum will display the painting among its collection. Based on the information provided for Ron and Sandy, what is Ron's charitable contribution deduction? A) $34,000 B) $13,800 C) $16,500 D) $23,000

ans d Because the artwork is appreciated property held for less th

Your clients, Joseph and Jane, have read many articles in financial publications about the alternative minimum tax (AMT) and are concerned that some of their investments and activities may cause AMT problems. Which of the following are preference items or adjustments for purposes of the individual AMT? Interest from qualified private-activity municipal bonds issued in 2008 Bargain element on the exercise of an incentive stock option Excess of percentage depletion over the property's adjusted basis Cost depletion deductions A) I and IV B) II, III, and IV C) I, II, III, and IV D) I, II, and III

ans d By definition, the only listed item that is not an AMT preference item or adjustment is the cost depletion deduction. Note that interest on private-activity municipal bonds issued in 2009 and 2010 is not a preference item for the AMT.

What important information could you tell your nonresident client (married to a U.S. citizen) about electing to be treated as a resident alien for tax purposes? You will need to pay taxes only on your U.S. income. You will need to pay taxes on your worldwide income. This will create an immigration benefit. You should obtain an individual taxpayer identification number (ITIN) or Social Security number. A) I only B) II, III, and IV C) I and IV D) II and IV

ans d If married to a U.S. citizen or resident alien, the nonresident alien can elect to be treated as a resident alien for tax purposes only. Tax status does not necessarily reflect immigration status. If this election is made, the couple must pay U.S. taxes on their worldwide income. In this situation, the nonresident alien spouse should obtain an Individual Tax Identification Number (ITIN). Depending on their individual situation and intentions, they may apply for a Social Security number with the Social Security Administration.

Which of the following statements regarding below-market loans is CORRECT? A) In a compensation-related loan, the employer has interest income and compensation expense in the amount of the imputed interest, but the employee has received a gift from the employer in the same amount. B) For gift loans greater than $10,000 and less than or equal to $100,000, a minimum of $1,000 of interest must be imputed. C) If the borrower's net investment income for the year does not exceed $1,000, no interest is imputed on loans of more than $100,000. D) For a gift loan, the amount of the imputed interest is treated as a gift from the lender to the borrower.

ans d In a gift loan, a gift has been made by the lender to the borrower in the amount of the imputed interest. In a compensation-related loan, the employer has interest income and compensation expense for the amount of the imputed interest. The borrower will have compensation income and interest expense, which may or may not be deductible, for the same amount. For gift loans greater than $10,000 and less than or equal to $100,000, if the borrower's net investment income for the year does not exceed $1,000, no interest is imputed. For a gift loan of more than $100,000, the prevailing federal rate of interest will be imputed.

For the current tax year, Bob Phillips, an individual taxpayer filing a joint return, has $50,000 of investment interest expense and $20,000 of net investment income (interest and dividends). Bob's AGI is $200,000. How much investment interest expense, if any, may Bob deduct in the current tax year? A) $0 B) $50,000 C) $21,000 D) $20,000

ans d Investment interest expense is deductible up to the amount of net investment income. The problem tells us that the net investment income is $20,000; thus that is the maximum deduction. The fact that the dividends are included in the net investment income indicates that the taxpayer elected to include them in investment income and is forgoing the preferential rates associated with qualified dividends. The AGI has no bearing on the answer.

Which one of the following is the best description of itemized deductions? A) Personal expenses deductible in arriving at total income B) Trade or business expenses deductible in arriving at total income C) Trade or business expenses deductible from adjusted gross income D) Personal expenses deductible from adjusted gross income

ans d Itemized deductions are generally personal expenses (e.g., home mortgage interest, medical expenses) that are specifically allowed as a deduction from AGI in arriving at taxable income. They are not deductible in arriving at total income.

Jim, a single taxpayer, sells Section 1244 stock for a $250,000 loss during the tax year. The stock had been held for two years. Which one of the following is CORRECT regarding the treatment of the loss? A) $100,000 is treated as an ordinary loss; $150,000 is treated as long-term capital loss. B) $50,000 is treated as an ordinary loss; $200,000 is treated as a Section 1244 carryforward. C) $250,000 of the loss is fully deductible as a capital loss. D) $50,000 is treated as an ordinary loss; $200,000 is treated as long-term capital loss.

ans d Loss on the sale of Section 1244 stock is deductible as an ordinary loss up to $100,000 per year on a jointly filed return, or $50,000 on any other return (which applies to Jim as a single taxpayer). Any Section 1244 loss in excess of the annual limits is treated as a capital loss—long term or short term, depending on the holding period. The $100,000/$150,000 option would be correct if Jim were a married taxpayer filing jointly. LO 5.2.1

Which of the following charitable contributions is NOT deductible on Mary's 2020 income tax return? A) A contribution charged to her credit card October 10, 2019, which she did not pay until the end of 2019. B) All of these are not deductible. C) A contribution charged to her credit card on December 10, 2019, even though she did not pay the bill until January 2020. D) A pledge made on December 31, 2020, to pay the United Way $50 before April 15, 2021.

ans d One requirement for a charitable contribution deduction is that the gift must be paid in cash or property before the end of the tax year. Contributions made by credit cards are deductible the year in which the credit card is actually charged, regardless of when the taxpayer pays the credit card bill.

Gwen is discussing her charitable contribution plans with her financial advisor, Rick. Her AGI is $40,000, and she has two charities to which she is considering making a cash donation of $15,000: The Hebert Family Foundation, established by her grandfather, or the American Cancer Society. A donation to either would be in honor of her aunt who succumbed to cancer this year. Gwen would like to maximize her charitable deduction this year. Which of the following statements Rick makes is CORRECT? A donation split 50/50 between the two charities would allow Gwen to deduct the entire $15,000 this year. Donating to The Hebert Foundation would allow Gwen a donation of up to 50% AGI and please her grandfather. A) Both I and II B) Neither I nor II C) II only D) I only

ans d Only statement I is correct. If she split the donation and gave $7,500 to each one, her entire donation would be deductible this year. The Hebert Family Foundation is a 30% organization and her maximum available deductible donation this year would be $12,000. If she donated only to it, she would have to carry forward $3,000 until next year. The American Cancer Society is a 50% organization, and she could donate up to $24,000 (60% of AGI for cash) to it in a cash contribution this year. Halving the donation will still allow her to deduct the full $15,000 this year.

Your client, Elaine Dell, is near the highest tax bracket and is contemplating several investments. She is, however, concerned about minimization of her federal income tax liability on the income from the investment. Which of the following investments would produce income that would be taxed at the lowest potential tax rate? A) A corporate bond fund B) A zero coupon bond C) A certificate of deposit D) A utility stock with a high dividend yield

ans d Qualified dividends are generally taxed at a 15% rate (or 20% for taxpayers with higher income levels). All of the other options produce interest income, which is taxable as ordinary income, at the marginal rate of the taxpayer. LO 2.1.3

Clare is a single taxpayer. In 2020, her AGI is $235,000, including a net long-term capital gain of $50,000. What is the amount, if any, of Medicare contribution tax that she must pay? A) $0 B) $570 C) $1,900 D) $1,330

ans d She will pay the 3.8% Medicare contribution tax on $35,000. This is the lesser of the net investment income ($50,000) or the AGI in excess of the threshold amount ($235,000 - $200,000, or $35,000). In this situation, only $35,000 of the net investment income is subject to the Medicare contribution tax. Clare will pay a $1,330 Medicare contribution tax (3.8% on $35,000).

Jack bought publicly traded stock seven years ago for $6,000. Its current value on the securities market is $11,000. He has donated this appreciated stock to a charity that provides housing for the homeless. What must Jack do to take the donation as a charitable deduction? Jack must have documentation from the charity substantiating the amount of the donation, the date donated, and the name of the charity. All donations of stock must have a qualified appraisal of the stock attached to the donor's income tax return. A) II only B) Neither I nor II C) Both I and II D) I only

ans d Statement I is correct. In addition, the taxpayer must be in receipt of this documentation by the due date of the return or when the return is filed. Statement II is incorrect because it reads "all" donations of stock must have a qualified appraisal. Per the textbook, a qualified appraisal is not required for closely held stock if the amount donated is less than $10,000. The appraisal itself is not attached to the return.

Jack bought publicly traded stock seven years ago for $6,000. Its current value on the securities market is $11,000. He has donated this appreciated stock to a charity that provides housing for the homeless. What must Jack do to take the donation as a charitable deduction? Jack must have documentation from the charity substantiating the amount of the donation, the date donated, and the name of the charity. All donations of stock must have a qualified appraisal of the stock attached to the donor's income tax return. A) Both I and II B) Neither I nor II C) II only D) I only

ans d Statement I is correct. In additional, the taxpayer must be in receipt of this documentation by the due date of the return or when the return is filed. Statement II is incorrect because it reads "all" donations of stock must have a qualified appraisal. A qualified appraisal is not required for closely held stock if the amount donated is less than $10,000. The appraisal itself is not attached to the tax return.

Five years ago, Tom bought 10,000 shares at $10 per share in an intermediate-term bond fund. Today, the shares are worth $200,000 and are paying a nonqualified dividend of $8,000 per year. Tom feels that the stock will continue to appreciate at a rate of 5% per year, including the dividend. Tom wants to establish a college education fund for his two daughters, ages 18 and 9. Neither child has any earned income. Which of the following statements is true? If Tom gives 2,500 shares to his 18-year-old daughter, all income from the 2,500 shares will be taxed in her income tax bracket. If Tom gives 2,500 shares to his 9-year-old daughter, all dividends from the 2,500 shares will be taxed at her marginal rate. Two years from now, if Tom's older daughter sells her 2,500 shares at $30 per share, Tom will need to report the gain as a long-term capital gain on his personal income tax return. All interest income received by his 9-year-old daughter that exceeds $2,200 in 2020 will be taxed at the parents' marginal tax rate. A) I and IV B) II and III C) I and II D) IV only

ans d Statement IV is the only correct statement. The kiddie tax applies to children under 19 years of age. It also applies to children under age 24 if they are full-time students. The kiddie tax does not apply if the child's earned income exceeds one-half of the child's support. Thus, I and II are incorrect. There is no requirement that the proceeds of a future sale be reported on the donor's return. As a result of TCJA, the net unearned income is taxed at the parents' marginal tax rates.

Which of the following tax preference items are used in calculating the alternative minimum tax (AMT) for an individual? Tax-exempt income from a State of Louisiana general obligation municipal bond Percentage depletion in excess of basis on a mining property Tax-exempt interest on a private-activity bond issued in 2012 Exclusion of gain on the sale of certain qualified small business corporation stock A) I only B) I, II, and III C) I, II, III, and IV D) II, III, and IV

ans d Tax-exempt income from a general obligation municipal bond is not a preference for AMT. All of the other items are tax preference items for AMT purposes.

Larry and Pam are married and will file a joint return for the 2020 tax year. Pam is an active participant in a company-maintained retirement plan, but Larry is not. They have provided you with the following information. Pam's divorce was finalized in 2018. Larry's salary$70,000Larry's IRA contribution$6,000Pam's salary$65,000Pam's IRA contribution$6,000Alimony payments to Pam's ex-spouse$9,600Itemized deductions$15,000 Based on the information given, what is their taxable income for the 2020 tax year? A) $100,900 B) $89,900 C) $107,400 D) $94,600

ans d The $135,000 in salaries is reduced by the alimony payment of $9,600 and Larry's IRA contribution of $6,000 to give an AGI of $119,400. The AGI is reduced by the greater of the itemized deductions ($15,000) or the standard deduction ($24,800 in 2020) to leave $94,600. Pam's IRA contribution is nondeductible because the MAGI exceeds the phaseout range of $104,000 to $124,000. Because the MAGI is less than $196,000, Larry may still deduct his full IRA contribution. Note that the MAGI ($125,400 in this situation) is the AGI computed without regard to the IRA deduction itself. The AGI phaseout limits for active participants will be provided on the exam.

Brent and Sheila are married and will file a joint return. They have provided you with the following information. Brent's salary$80,000Sheila's salary$65,350Alimony payments to Brent's ex-spouse$20,000Brent's child support paid$12,000Itemized deductions$13,000 Brent's divorce from his prior marriage was finalized in 2016. Based on the information given, what is their taxable income for the 2020 tax year? A) $112,250 B) $111,550 C) $88,550 D) $100,550

ans d The $145,350 in salaries is reduced by the alimony payment of $20,000 to give an AGI of $125,350. The AGI is reduced by the greater of the itemized deductions ($9,000) or the standard deduction ($24,800 in 2020) to equal $100,550. The child support paid is not a deductible item.

Carl Taylor has an AGI of $200,000. He donated vacant land valued at $130,000 that was purchased eight years ago to the local church. His basis in this land was $110,000. What is Carl's maximum allowable charitable contribution for the current year? A) $39,000 B) $33,000 C) $60,000 D) $100,000

ans d The 50% election would provide Carl with the maximum current-year deduction. If Carl makes a 50% election, he must utilize the basis of the property but may deduct up to 50% of AGI. This yields a $100,000 current-year deduction with a $10,000 carryforward. If no 50% election were made, the deduction would be based on the fair market value of the property but would be limited to 30% of AGI, which is $60,000 with a $70,000 carryforward.

Your client, Mary, is contemplating the formation of an S corporation for her small manufacturing business. What can you accurately tell Mary about forming an S corporation? Mary would have limited liability. The number of shareholders is limited to 100. Mary's death would require reorganization of the corporation. The corporation's items of income, deductions, and tax credits would flow through to Mary's income tax return. A) III only B) II and III C) I and IV D) I, II, and IV

ans d The S corporation form is similar to the partnership in operating as a conduit for tax purposes. It also is similar to a C corporation, since it features limited liability for shareholders. In addition, the death of a shareholder in an S corporation does not create a need to reorganize the business. Finally, unlike a C corporation, an S corporation may have no more than 100 shareholders.

Mike is involved in the business of breeding show horses on a part-time basis as a sole proprietor. He has shown a significant net loss in this business every year for the last six years, and he has used these losses to reduce the tax liability on the salary he has earned from his law practice. With which one of the following potential tax traps should Mike be most concerned? A) Passive activity B) Substance over form C) Assignment of income D) Hobby loss

ans d The fact that this business has not been profitable and is merely a side business increases the likelihood it could be reclassified as a hobby loss. Mike has operated the business only part time, and he has not demonstrated that he is attempting to make it a profitable business. The fact that huge losses have occurred every year overrides the presumption that Mike is in the business to make a profit.

Which of the following individuals will be subject to the kiddie tax at the parents' marginal tax rate in 2020? Kyle, age 19, and a full-time freshman at State University, who earned $5,000 from his summer employment. He is listed as a dependent on his parents' income tax return. Devin, age 22, has $900 in dividend income this year. She is a full-time student and her parents' dependent. Jordan, age 14, received $1,200 of interest income this year. Lisa, age 19 and living at home but not attending school, is listed as a dependent on her parents' income tax return and received $2,400 interest and dividends in 2020. A) IV only B) I, II, and III C) III and IV D) None of these

ans d The kiddie tax applies to any unearned income (such as dividends and interest) of a child under age 19 (under age 24 if a dependent full-time student providing less than 50% of own support). None of these individuals are subject to the kiddie tax. Kyle has only earned income. Devin and Jordan received less than $2,200. Lisa is not a full-time student under age 24.

Kris Swenson anticipates adjusted gross income of $100,000 for the current tax year. She is considering making a gift of a painting to the American Red Cross in the current tax year. Kris's basis in the painting is $35,000. The painting has a current fair market value of $50,000. Kris has owned the painting for 15 years. If Kris does gift the painting to the American Red Cross this year, what is the maximum allowable charitable deduction she can receive in the current tax year? A) $50,000 B) $20,000 C) $30,000 D) $35,000

ans d The painting would be considered use-unrelated tangible personalty. The deduction for use-unrelated tangible personalty is limited to basis, with a 50% of AGI limitation. Thus, the current-year deduction is $35,000. If the painting had been donated to an art museum, for example, the contribution would be of use-related tangible personalty. Since the painting had been held for the long-term holding period, the deduction would have been $30,000 (long-term capital gain property to a 50% organization uses FMV with a 30% of AGI limitation) with a $20,000 carryforward.

Which one of the following is CORRECT regarding the Coverdell Education Savings Account? A) Distributions may be tax free only if made for a full-time student. B) Room and board may be covered with a tax-free distribution only if the student is full-time. C) Deductible contributions of up to $2,000 may be made per beneficiary. D) Distributions may be tax free even if made for K-12 expenses.

ans d The predominant benefit of the Coverdell ESA is distributions may also be used to pay for K-12 expenses. This is unlike the 529 plan which is designed primarily to pay for college expenses (Note: a limited amount of $10,000 may now be withdrawn from a 529 for K-12 expenses per the TCJA).

Which of the following forms of business would file a Schedule C to report the income or loss from the business? A) Limited partnership B) S corporation C) C corporation D) Sole proprietorship

ans d The sole proprietorship is the business that files a Schedule C with the individual's Form 1040. The limited partnership files a Form 1065, and the C corporation files a Form 1120. The S corporation files an 1120-S. Note that a single-member LLC would also file a Schedule C, as it would be treated as a disregarded entity for income tax purposes.

Your client has a small business that is operated as a sole proprietorship. He is considering employing his 15-year-old son in the family business. The child, who has no other income, will provide services such as dusting, sweeping, and vacuuming. Which of the following is CORRECT with respect to the wages paid to the son? Because there is no valid business purpose for the son's employment, the wages would not be deductible. Because the wages are paid to a family member under the age of 18, they are not subject to FICA. Any wages paid to the son in excess of $2,200 annually are subject to tax at the parental rate. Up to $12,400 of wages could be received free of income taxes by the son. A) II and III B) I only C) I and III D) II and IV

ans d The wages paid to a child under the age of 18 from an unincorporated business are not subject to FICA or unemployment taxes. Earned income is not subject to the kiddie tax, and the child's full standard deduction of up to $12,400 (for 2020) may be used to shelter his earned income.

Tony made a $10,000 pledge to his church building fund that he would like to fulfill before the end of the year. He owns stock with a basis to Tony of $15,000 that currently has a FMV of $10,000. He is meeting with his planner to discuss year-end planning and tells the planner about the stock he wants to use as payment of the donation to the church building fund. What should the planner tell Tony? A) Tony should hold on to the stock and gift other property or cash instead. B) Tony should donate the stock with the stipulation that the church sells it immediately and take advantage of the capital loss. C) Tony cannot donate loss property. D) Tony should sell the stock and make a cash donation to his church.

ans d Tony should sell the stock and take advantage of the capital loss on his income tax return. He can donate the cash from the stock sale to fulfill his pledge to the church building fund.

Which of the following benefits that Claudia has received from her employer can be excluded from taxation? A) A company car that she uses for personal vacations. B) A year-end bonus. C) An athletic membership at a local club valued at $1,500 per year. D) $5,000 of graduate education assistance.

ans d Undergraduate and graduate education assistance is excluded from an employee's income in any one year period, up to a maximum of $5,250. The other options are fully taxable.

Which of the following statements is CORRECT regarding the credit for adoption expenses? A) All of these. B) The adoption credit is a nonrefundable credit. C) An eligible adoptee is an adoptee who is not yet age 18 at the time of adoption or who is physically or mentally incapable of caring for herself. D) A tax credit of $14,300 of qualified adoption expenses (for 2020) for each eligible adoptee is available.

ans: a A tax credit of $14,300 of qualified adoption expenses for each eligible adoptee is available in 2020. An eligible adoptee is an adoptee who is not yet age 18 at the time of adoption or who is physically or mentally incapable of caring for herself. The adoption credit is nonrefundable. LO 1.5.1

Which one of the following steps occur in the tax calculation process? A) Tax liability minus tax credits equals refund or tax owed B) Total withholding is adjusted on Form I-9 C) Total tax liability equals refund or tax owed D) Total tax liability minus tax credits and plus additional taxes owed, equals total tax liability

ans: a Tax liability minus tax credits plus additional taxes owed equals total tax liability. Then, total tax liability minus withholding and/or estimated tax payments made equals refund or tax amount owed. Witholding is adjusted on form W4 and is not considered part of the 1040 calculation.

Todd is employed at Wow Industries as an accountant. His employer deducted $8,500 from his paycheck in 2020 for federal income taxes. Todd also has a side practice for which he paid another $4,300 to the IRS in estimated federal income taxes for 2020. When he filed his return, he had a tax liability of $11,600 before a child and dependent care credit of $400. Which of the following statements is CORRECT? A) Todd has an income tax refund of $1,600 for 2020. B) Todd's refund is $5,200 for 2020. C) Because the child and dependent care credit is a nonrefundable credit, Todd's refund is $1,200. D) He cannot receive a refund in 2020.

ans: a Todd's refund is $1,600 [$12,800 in total tax deposits - ($11,600 tax liability - $400 child and dependent care credit)]. The child and dependent care credit only reduces Todd's tax liability to $11,200 and he has deposited $12,800. This entitles him to a refund of his excess tax payments of $1,600.

Cash value life insurance is often structured like an investment vehicle. However cash value life insurance contains important features that shelter the inside buildup from taxation. Which of the following will NOT be considered when determining whether a policy can maintain its tax favored status? A) The premium value test B) The cash guideline premium test and corridor test C) The cash value accumulation test D) The death benefit

ans: a Without a death benefit, a contract does not meet the legal definition of life insurance. There are currently two tests—only one of which must be met—in order to classify a product as life insurance for federal income tax purposes: (1) the cash value accumulation test and (2) the cash guideline premium test and corridor test. There is no premium value test.

Which of the following is an incentive provision as it relates to federal income taxation? A) Exclusion for life insurance death benefits B) Residential solar energy credits C) Casualty loss deduction D) Capital loss deduction

ans: b Congress has deemed it socially desirable to provide solar energy, so there is a credit provided to taxpayers who invest in such activities.

Which one of the following is NOT deductible as a miscellaneous itemized deduction? A) Gambling losses to the extent of gambling winnings B) Appraisal fee for a charitable contribution C) Deduction for unrecovered basis in a commercial annuity D) Impairment-related work expense of a handicapped individual

ans: b The appraisal fee to determine the value of a piece of artwork being donated to charity is not a deductible miscellaneous itemized deduction. An expense related to the determination of, or collection of, a tax liability is no longer deductible. Impairment-related work expenses of a handicapped individual, gambling losses to the extent of gambling winnings, and the deduction for unrecovered basis in a commercial annuity are deductible miscellaneous itemized deductions.

Personal expenses deductible from adjusted gross income most accurately describes which one of the following? A) Adjustments to income B) Schedule C expenses C) Standard deduction D) Itemized deductions

ans: d Itemized deductions are generally personal expenses (e.g., home mortgage interest, medical expenses) that are specifically allowed as a deduction from AGI. Schedule C (sole proprietorship) expenses and adjustments to income are both deductions for AGI, or above-the-line deductions.

Sally Franklin has AGI of $300,000. In addition, she currently has passive income of $150,000 and passive losses of $175,000—$150,000 of which she uses to offset the passive income and $25,000 of which is subject to disallowance. Which one of the following investments has the greatest potential for reducing Sally's tax liability? A) A limited partnership involved in a historic rehabilitation project that is producing passive losses and credits B) An equipment-leasing limited partnership producing passive losses C) "Active participation" rental real estate that is producing a loss D) A working interest in an oil and gas general partnership

ans: d A working interest in an oil and gas partnership can provide unlimited loss deductions against other income. Congress considers this socially desirable to encourage investment in the industry. The caveat is investors in these instruments must also assume unlimited liability. Therefore, a working interest must be a general partnership rather than a limited partnership. The other answer choices cannot offset passive income.

George, whose wife died last November, filed a joint tax return for last year. He did not remarry after his wife's death and has continued to maintain his home for his two dependent children. In the preparation of his tax return for this year, what is George's filing status?

qualifying widower George filed a joint return in the year of his wife's death. He can file as a qualifying widower (also known as surviving spouse) for the two years following his wife's death if he continues to maintain a home for his dependent children.


Kaugnay na mga set ng pag-aaral

ECON: Chapter 3: Supply: Think Like a Buyer

View Set

net125 chapter 8 quiz, net125 chapter 9 quiz, net125 chapter 8 exam, net125 chapter 9 exam

View Set

Astronomy Homework Questions (Ch. 6-14)

View Set

Unit 6: The Enlightenment and French Revolution

View Set